You are on page 1of 167

Contents

Mathematics
Paper Term I 2014 1—13
Paper Outside Delhi 2014 14—33
Paper Delhi 2014 34—53
Paper Term I 2015 54—69
Paper Outside Delhi 2015 70—87
Paper Delhi 2015 88—107
Paper Term I 2016 108—123
Paper Outside Delhi 2016 124—141
Paper Delhi 2016 142—161
Paper Outside Delhi 2017 162—180
Paper Delhi 2017 181—197
Paper 2018 198—215
Paper Outside Delhi 2019 216—242
Paper Delhi 2019 243—268
Basic Paper Outside Delhi 2020 269—290
Basic Paper Delhi 2020 291—312
Standard Paper Outside Delhi 2020 313—339
Standard Paper Delhi 2020 340—363

Science
Paper Term I 2014 364—375
Paper Outside Delhi 2014 376—398
Paper Delhi 2014 399—421
Paper Term I 2015 422—445
Paper Outside Delhi 2015 446—461
Paper Delhi 2015 462—478
Paper Term I 2016 479—488
Paper Outside Delhi 2016 489—509
Paper Delhi 2016 510—527
Paper Outside Delhi 2017 528—545
Paper Delhi 2017 546—562
Paper 2018 563—576
Paper Outside Delhi 2019 577—597
Paper Delhi 2019 598—613
Paper Outside Delhi 2020 614—638
Paper Delhi 2020 639—658
Social Science
Paper Term I 2014 659—669
Paper Term I 2015 670—680
Paper Outside Delhi 2015 681—695
Paper Delhi 2015 696—711
Paper Term I 2016 712—721
Paper Outside Delhi 2016 722—736
Paper Delhi 2016 737—752
Paper Outside Delhi 2017 753—766
Paper Delhi 2017 767—778
Paper 2018 779—791
Paper Outside Delhi 2019 792—807
Paper Delhi 2019 808—822
Paper Outside Delhi (Term-II) 2020 823—835
Paper Delhi (Term-II) 2020 836—848

English
Paper Term I 2014 849—857
Paper Term I 2015 858—866
Paper Outside Delhi 2015 867—880
Paper Delhi 2015 881—894
Paper Term I 2016 895—903
Paper Outside Delhi 2016 904—917
Paper Delhi 2016 918—932
Paper Outside Delhi 2017 933—944
Paper Delhi 2017 945—954
Paper 2018 955—964
Paper Outside Delhi 2019 965—979
Paper Delhi 2019 980—992
Paper Outside Delhi 2020 993—1003
Paper Delhi 2020 1004—1019
Hindi Course A
Paper Term I 2014 1020—1028
Paper Term I 2015 1029—1037
Paper Outside Delhi 2015 1038—1051
Paper Delhi 2015 1052—1064
Paper Term I 2016 1065—1074
Paper Outside Delhi 2016 1075—1093
Paper Delhi 2016 1094—1113
Paper Outside Delhi 2017 1114—1133
Paper Delhi 2017 1134—1153
Paper 2018 1154—1160
Paper Outside Delhi 2019 1161—1172
Paper Delhi 2019 1173—1188
Paper Outside Delhi 2020 1189—1204
Paper Delhi 2020 1205—1219

Hindi Course B
Paper Term I 2015 1220—1227
Paper Outside Delhi 2015 1228—1237
Paper Delhi 2015 1238—1248
Paper Term I 2016 1249—1255
Paper Outside Delhi 2016 1256—1266
Paper Delhi 2016 1267—1276
Paper Outside Delhi 2017 1277—1287
Paper Delhi 2017 1288—1298
Paper 2018 1299—1307
Paper Outside Delhi 2019 1308—1325
Paper Delhi 2019 1326—1342
Paper Outside Delhi 2020 1343—1356
Paper Delhi 2020 1357—1371
https://www.arundeepselfhelp.info/index.php?route=product/product&path=180&product_id=395

MATHEMATICS 2014 TERM I


SET I
Time allowed : 3 hours Maximum marks : 90

Section – A
1. In the given figure if DE || BC, AE = 8 cm, EC = 2 cm and BC = 6 cm, then find DE. [1]
Sol. In DADE and DABC,
A
ÐDAE = ÐBAC [Common]
ÐADE = ÐABC [Corresponding angles] 8 cm
By AA axiom of similarity
D E
DADE ~ DABC
2 cm
AE DE
\ = [CPCT] B 6 cm C
AC BC
8 DE
Þ = Þ 10 × DE = 48
8+2 6
Þ DE = 4.8 cm Ans.

1 - cot 2 45º
2. Evaluate : 10 · . [1]
1 + sin2 90º

1 - cot 2 45° 1 - (1) 2


Sol. 10· = 10·
1 + sin 2 90° 1 + (1)2

0
= 10. = 0 Ans.
2
5
3. If cosec q = , find the value of cot q. [1]
4
Sol. We know that,
cot2 q = cosec2 q – 1
2
æ5ö 25 25 - 16 9
= ç ÷ -1 = -1 = =
è4ø 16 16 16

9
Þ cot2 q =
16
3
i.e. cot q = Ans.
4
Arundeep’s Solved Papers 1 Mathematics 2014 (Term I)
https://www.arundeepselfhelp.info/index.php?route=product/product&path=180&product_id=395

Arundeep’s Solved Papers 2 Mathematics 2014 (Term I)


4. Following table shows sale of shoes in a store during one month :

Size of shoe 3 4 5 6 7 8
Number of pairs sold 4 18 25 12 5 1
Find the model size of the shoes sold. [1]
Sol. Maximum number of pairs sold = 25 (size 5)
\ Model size of shoes = 5 Ans.
Section – B

5. Find the prime factorisation of the denominator of rational number expressed as 6.12 in
simplest form. [2]
Sol. Let x = 6.1212........... ...(i)
Þ 100x = 612.1212........... ...(ii)
Subtracting eq. (i) from (ii), we get
99x = 606 3 33
606 202 11 11
Þ x = =
99 33 1
\ Denominator = 33
and Prime factorisation of 33 be = 3 × 11 Ans.
1
6. Find a quadratic polynomial, the sum and product of whose zeroes are 3 and 3
respectively. [2]
Sol. Given, sum of zeroes, (S) = 3

1
and Product of zeroes, (P) =
3
Thus, quadratic polynomial is given as f(x) = x2 – Sx + P

1 3x 2 - 3x + 1 1
\ f(x) = x 2 - 3x + = = ( 3x 2 - 3 x + 1) Ans.
3 3 3
7. Complete the following factor tree and find the composite number x. [2]

3 585
3 195
5 65
13 13
Sol. y = 5 × 13 = 65
1
x = 3 × 195 = 585 Ans.
https://www.arundeepselfhelp.info/index.php?route=product/product&path=180&product_id=395

Arundeep’s Solved Papers 3 Mathematics 2014 (Term I)


8. In a rectangle ABCD, E is middle point of AD. If AD = 40 m and AB = 48 m, then find EB.
[2]
Sol. Given, E is the mid-point of AD
40 m
\ AE = = 20 m
2
Also ÐA = 90º [Angle of a rectangle]
\ In rt. DBAE, we have
EB2 = AB2 + AE2 [Pythagoras’ theorem]
= (48)2 + (20)2 = 2304 + 400 = 2704
EB = 2704 = 52 ´ 52 = 52 m Ans.
9. If x = p sec q + q tan q and y = p tan q + q sec q, then prove that x2 – y2 = p2 – q2. [2]
Sol. L.H.S. = x2 – y2
= (p sec q + q tan q)2 – (p tan q + q sec q)2
= (p2 sec2 q + q2 tan2 q + 2pq sec q tan q) – (p2 tan2 q + q2 sec2 q + 2pq sec q tan )
= p2 sec2 q + q2 tan2 q + 2pq sec q tan q – p2 tan2 q – q2 sec2 q – 2pq sec q tan q
= p2 (sec2 q – tan2 q) – q2 (sec2 q – tan2 q)
= p2 – q2 [Q sec2 q – tan2 q = 1]
= R.H.S. Hence Proved.
10. Given below is the distribution of weekly pocket money received by students of a class.
Calculate the pocket money that is received by most of the students. [2]

Pocket Money (in ` ) 0–20 20–40 40–60 60–80 80–100 100–120 120–140
No. of Students 2 2 3 12 18 5 2

Sol. Pocket Money Number of


(in ` ) Students
0–20 2
20–40 2
40–60 3
60–80 12 f0
80–100 18 f1 (Maximum)
100–120 5 f2
120–140 2

Here maximum frequency is 18 which lies in class 80 – 100.


\ Modal class = 80 – 100
f1 - f0 18 - 12
Thus, mode = l + ´ h = 80 + ´ 20
2 f1 - f0 - f 2 36 - 12 - 5
https://www.arundeepselfhelp.info/index.php?route=product/product&path=180&product_id=395

Arundeep’s Solved Papers 4 Mathematics 2014 (Term I)

6 120 On subtracting eq. (i) and (ii), we get


= 80 + ´ 20 = 80 + = 80 + 6.32
19 19 3x – y = 5
= 86.32 (approx.) 5x – y = 11
\ Required pocket money = ` 86.32 (approx.) – + –
Ans. – 2x = –6
Section – C Þ x=3
Putting the value of x in eq. (i) ; we have
11. Prove that 3 + 2 3 is an irrational 3 (3) – y = 5
number. [3] Þ 9–5=y Þy=4
Sol. Let us assume to the contrary, that 3 + 2 3 \ x = 3, y = 4 Ans.
is rational. 13. A man earns ` 600 per month more than
So that we can find co-prime positive his wife. One-tenth of the man’s salary
integers a and b (b ¹ 0), such that and one-sixth of the wife’s salary amount
to ` 1,500, which is saved every month.
a
3+2 3 = Find their incomes. [3]
b Sol. Let wife’s monthly income = ` x
Rearranging the equation, we get Then man’s monthly income = ` (x + 600)
a a - 3b According to the question, we have
2 3 = b -3= b 1 1
( x + 600) + ( x ) = 1,500
a - 3b a 3b 10 6
Þ 3 = = -
2b 2b 2b 3( x + 600) + 5 x
Þ = 1,500
a 3 30
Þ 3 = 2b - 2 Þ 3x + 1,800 + 5x = 45,000
Since a and b are integer, and b ¹ 0, Þ 8x = 45,000 –1,800

a 3 43, 200
So - is rational number and hence Þ x = = 5, 400
2b 2 8
Thus, wife’s income = ` x = ` 5,400
3 is rational.
and Man’s income = ` (x + 600) = ` (5400 + 600)
But this contradicts the fact that 3 is an = ` 6,000 Ans.
irrational number. 14. Check whether polynomial x – 1 is a
So we conclude that 3 + 2 3 is an factor of the polynomial x3 – 8x2 + 19x – 12.
Verify by division algorithm. [3]
irrational number. 3 2
Sol. Let P (x) = x – 8x + 19x – 12
Hence Proved.
12. Solve by elimination : Put x = 1, we have
3x = y + 5 P (1) = (1)3 – 8 (1)2 + 19 (1) – 12
5x – y = 11 [3] = 1 – 8 + 19 – 12
Sol. Given equations are, = 20 – 20
3x = y + 5 ...(i) =0
5x – y = 11 ...(ii) \ (x – 1) is a factor of P (x).
https://www.arundeepselfhelp.info/index.php?route=product/product&path=180&product_id=395

Arundeep’s Solved Papers 5 Mathematics 2014 (Term I)


Verification : 16. In the figure if DE || OB and EF || BC,
then prove that DF || OC. [3]

Sol. Given, In DABC, DE || OB and EF || BC


To Prove : DF || OC
Since remainder = 0. Proof : In DAOB, we have
Thus, (x – 1) is a factor of P (x). DE || OB
Hence Verified. AE AD
15. If the perimeters of two similar triangles \ = ...(i)
EB DO
ABC and DEF are 50 cm and 70 cm [Thales’ Theorem]
respectively and one sideof DABC = 20 cm, Similarly, in DABC, we have
then find the corresponding side of EF || BC
DDEF. [3]
Sol. AE AF
\ = ...(ii)
EB FC
D [Thales’ Theorem]
From (i) and (ii), we have
AD AF
=
DO FC
\ DF || OC
[By Converse of Thales’ Theorem]
Given, DABC ~ DDEF,
Hence Proved.
Perimeter of DABC = 50 cm 17. Prove the identity :
Perimeter DDEF = 70 cm (sec A – cos A) · (cot A + tan A) = tan A · sec A.
One side of DABC = 20 cm [3]
Let AB = 20 cm Sol. L.H.S. = (sec A – cos A) (cot A + tan A)
DABC ~ DDEF [Given] æ 1 ö æ cos A sin A ö
=ç - cos A ÷ ç + ÷
Peri (DABC) AB è cos A ø è sin A cos A ø
\ =
Peri (DDEF) DE
æ 1 - cos2 A ö æ cos 2 A + sin 2 A ö
50 20 =ç ÷ç ÷
Þ = è cos A ø è sin A cos A ø
70 DE
\ 50 × DE = 1400 sin 2 A 1
= ´
Þ DE = 28 m cos A sin A cos A
The corresponding side of DDEF = 28 cm. Ans. [Q cos2 A + sin2 A = 1]
https://www.arundeepselfhelp.info/index.php?route=product/product&path=180&product_id=395

Arundeep’s Solved Papers 6 Mathematics 2014 (Term I)

sin A 1
= ´
cos A cos A
= tan A · sec A = R.H.S. Hence Proved.
18. Given 2 cos 3q = 3, find the value of q. [3]

Sol. Given, 2 cos 3q = 3

3
Þ cos 3q = Þ cos 3q = cos 30º Þ 3q = 30º Þ q = 10º Ans.
2
19. For helping poor girls of their class, students saved pocket money as shown in the following
table :

Money saved (in ) 5–7 7–9 9–11 11–13 13–15


Number of students 6 3 9 5 7
Find mean and median for this data. [3]
Sol. The table of values is given as under :

Money No. of xi x i – 10 fidi c.f.


di =
saved Students 2
(in ` ) (fi) where h = 2
5–7 6 6 –2 –12 6
7–9 3 8 –1 –3 9
9–11 9 a=10 0 0 18
11–13 5 12 1 5 23
13–15 7 14 2 14 30
Sf i = 30 Sfidi = 4

(i) Then by step deviation method, we have,


å fi di
Mean = a + ´h
å fi

4
Þ x = 10 + ´ 2 = 10 + 0.27 = ` 10.27Ans.
30
(ii) Now, N = Sfi = 30
N 30
\ = = 15, Clearly the cummlative frequency just greater than 15 be 18 which lies in class
2 2
9 – 11.
https://www.arundeepselfhelp.info/index.php?route=product/product&path=180&product_id=395

Arundeep’s Solved Papers 7 Mathematics 2014 (Term I)


\ Median class is 9–11.

N
-F
15 - 9 6
Thus, median = l + 2 h =9+ ´2 =9+ ´2
f 9 9

= 9 + 1.33 = ` 10.33 Ans.


20. Monthly pocket money of students of a class is given in the following frequency distribution:
[3]

Pocket money (in ) 100–125 125–150 150–175 175–200 200–225


Number of students 14 8 12 5 11
Find mean pocket money using step deviation method.
Sol. The table of values is given as under :

Pocket No. of
Money Students xi x i – 162.5 fidi
di =
(in ` ) (fi) 25

100–125 14 112.5 –2 –28


125–150 8 137.5 –1 –8
150–175 12 0 0
175–200 5 187.5 1 5
200–225 11 212.5 2 22
Sfi = 50 Sfidi = –9

Then big step deviation method, we have


å fi di
Mean = a + ´h
å fi

-9
= 162.5 + ´ 25 = 162.5 – 4.5 = ` 158 Ans.
50

Section – D
21. If two positive integers x and y are expressible in terms of primes as x = p2q3 and y = p3q,
what can you say about their LCM and HCF. Is LCM a multiple of HCF ? Explain. [4]
Sol. Given, x = p2q3 = p × p × q × q × q
And y = p3q = p × p × p × q
\ HCF = product of the smallest power of each common prime factor in the numbers x and y
= p × p × q = p2q
https://www.arundeepselfhelp.info/index.php?route=product/product&path=180&product_id=395

Arundeep’s Solved Papers 8 Mathematics 2014 (Term I)


and LCM product of the greatest power of Thus, length of rectangular pond = 7 ft.
each prime factor in the numbers x and y and Breadth of rectangular pond = 4 ft.
= p × p ×p × q × q × q = p3q3 Values :
Þ LCM = pq2 × p2q = pq2 × HCF 1. Water is essential for the survival of all
Yes, LCM is a multiple of HCF. living things including street animals.
2. Water is the base of life and no one can
Explanation :
live without it.
Let a = 12 = 22 × 3
23. If a polynomial x4 + 5x3 + 4x2 – 10x – 12
b = 18 = 2 × 32 has two zeroes as – 2 and – 3, then find
\ HCF = 2 × 3 = 6 ...(i) the other zeroes. [4]
2
LCM = 2 × 3 = 362
Sol. Given, polynomial is
LCM = 6 × 6 f(x) = x4 + 5x3 + 4x2 – 10x – 12.
LCM = 6 (HCF) [From (i)] Since two zeroes of f(x) are – 2 and – 3
Here LCM is 6 times the HCF. Ans. \ (x + 2) (x + 3) = x2 + 3x + 2x + 6
22. Sita Devi wants to make a rectangular = x2 + 5x + 6
pond on the road side for the purpose of Dividing the polynomial f(x) with
providing drinking water for street x2 + 5x + 6,
animals. The area of the pond will be By division algorithm, we have
decreased by 3 square feet if its length is
decreased by 2 ft. and breadth is
increased by 1 ft. Its area will be
increased by 4 square feet if the length
is increased by 1 ft. and breadth remains
same. Find the dimensions of the pond.
What motivated Sita Devi to provide
water point for street animals ? [4]
Sol. Let length of rectangular pond = x ft.
and breadth of rectangular pond = y ft. \ x4 + 5x3 + 4x2 – 10x – 12
= (x2 + 5x + 6) (x2 – 2)
\ Area of rectangular pond = xy
According to the question, we have = (x + 2) (x + 3) ( x - 2) ( x + 2)
(x – 2) (y + 1) = (xy – 3)
Other zeroes : x - 2 = 0 or x + 2 = 0
Þ xy + x – 2y – 2 = xy – 3
Þ x – 2y = – 1 ...(i) x = 2 or x = - 2
according to second given condition, we The zeroes of the polynomial are – 2, – 3,
have
2 and - 2. Ans.
(x + 1) y = (xy + 4)
24. Find all the zeroes of the polynomial
Þ xy + y = xy + 4
8x4 + 8x3 – 18x2 – 20x – 5, if it is given
i.e. y=4 ...(ii)
Putting the value of y in eq. (i), we get 5 5
that two of its zeroes are and - .
x – 2 (4) = – 1 2 2
Þ x–8=–1 Sol. Given polynomial is given by
Þ x=–1+8=7 f(x) = 8x4 + 8x3 – 18x2 – 20x – 5
https://www.arundeepselfhelp.info/index.php?route=product/product&path=180&product_id=395

Arundeep’s Solved Papers 9 Mathematics 2014 (Term I)


5 5 5 -1 -1
Since two zeroes are and - All the zeroes are
5
,- , and .
2 2 2 2 2 2
2 Ans.
æ 5ö æ 5ö æ 5ö 5
\ çç x - ÷÷ ´ çç x + ÷÷ = ( x)2 - çç ÷÷ = x 2 - 25. In the figure, there are two points D and
è 2 ø è 2 ø è 2 ø 2
E on side AB of DABC such that
5 AD = BE. If DP || BC and EQ || AC, then
Dividing the polynomial f(x) with x 2 - prove that PQ || AB. [4]
2

Sol.

By division algorithm, we have


Q
\ f(x) = 8x4 + 8x3 – 18x2 – 20x – 5
In DABC, we have
æ 5ö DP || BC (Given)
= ç x 2 - ÷ (8 x 2 + 8 x + 2)
è 2ø AD AP
Þ = ...(i)
DB PC
æ 5ö
= ç x 2 - ÷ . 2 (4 x 2 + 4 x + 1) [Thales’ Theorem]
è 2ø
Also, EQ || AC (Given)
= (2x2 - 5) (4x2 + 2x + 2 x + 1) BE BQ
Þ = [Thales’ Theorem]
EA QC
= (2 x2 - 5) [2 x (2 x + 1) + 1(2 x + 1)]
AD BQ
Þ = ...(ii)
f(x) = (2 x 2 - 5) (2 x + 1) (2 x + 1) DB QC

all zeroes of given polynomial x given by [Q AD = BE ; \ EA = DB]


f(x) = 0 From eq. (i) and (ii) ; we have
Þ 2x2 – 5 = 0 or 2x + 1 = 0 or 2x + 1 = 0 AP BQ
=
PC PC
5 -1 -1
i.e. x = ± or x = or x = \ PQ || AB (Inverse of Thales theorem)
2 2 2
Hence proved.
https://www.arundeepselfhelp.info/index.php?route=product/product&path=180&product_id=395

Arundeep’s Solved Papers 10 Mathematics 2014 (Term I)


26. In DABC, altitudes AD and CE intersect each other at the point P. Prove that
(i) DAPE ~ DCPD (ii) AP × PD = CP × PE
(iii) DADB ~ DCEB (iv) AB × CE = BC × AD [4]
Sol. Given, In DABC, AD ^ BC and CE ^ AB
(i) In DAPE and DCPD, we have
Ð1 = Ð4 [Each 90º]
Ð2 = Ð3 [Vertically opposite angles]
By AA axiom of similarity, we have
DAPE ~ DCPD
Hence proved.
(ii) DAPE ~ DCPD [Proved above]
AP PE
\ = [CPCT]
CP PD
Þ AP × PD = CP × PE
Hence Proved
(iii) In DADB and DCEB, we have
Ð5 = Ð7 (Each 90º)
Ð6 = Ð6 (Common)
By AA axiom, of similarity
DADB ~ DCEB Hence Proved.
(iv) DADB ~ DCEB [Proved Above]
AB AD
= [cpct]
CB CE
Þ AB × CE = BC × AD Hence Proved
27. Prove that :
(cot A + sec B)2 – (tan B – cosec A)2 = 2 (cot A . sec B + tan B . cosec A). [4]
Sol. L.H.S.
= (cot A + sec B)2 – (tan B – cosec A)2
= (cot2 A + sec2 B + 2 cot A sec B) – (tan2 B + cosec2 A – 2 tan B cosec A)
= cot2 A + sec2 B + 2 cot A sec B – tan2 B – cosec2 A + 2 tan B cosec A
= (sec2 B – tan2 B) – (cosec2 A – cot2 A) + 2 (cot A sec B + tan B cosec A)
= 1 – 1 + 2 (cot A sec B + tan B cosec A) [Q sec2 B – tan2 B = 1, cosec2 A – cot2 A = 1]
= 2 (cot A sec B + tan B cosec A) = R.H.S. Hence Proved.
28. Prove that :
(sin q + cos q + 1) · (sin q – 1 + cos q) · sec q · cosec q = 2. [4]
Sol. L.H.S. = (sin q + cos q + 1) · (sin q – 1 + cos q) · sec q cosec q
= [(sin q + cos q) + 1] · [(sin q + cos q) – 1)] · sec q · cosec q
= [(sin q + cos q)2 – (1)2] sec q cosec q [Q (a + b) (a – b) = a2 – b2]
= [sin2 q + cos2 q + 2 sin q cos q – 1] · sec q cosec q
https://www.arundeepselfhelp.info/index.php?route=product/product&path=180&product_id=395

Arundeep’s Solved Papers 11 Mathematics 2014 (Term I)


= (1 + 2 sin q · cos q – 1) · sec q cosec q [Q sin2 q + cos2 q = 1]

1 1
= 2 (sin q cos q) · ·
cos q sin q
= 2 = R.H.S. Hence Proved.
29. If tan (20º – 3a) = cot (5a – 20º), then find the value of a and hence evaluate :
sin a · sec a · tan a – cosec a · cos a · cot a. [4]
Sol. Given, tan (20º – 3a) = cot (5a – 20º)
Þ tan (20º – 3a) = tan [90 – (5a – 20º)] [Q cot q = tan (90º – q)]
Þ 20º – 3a = 90º – 5a + 20º Þ – 3a + 5a = 90º + 20º – 20º
Þ 2a = 90º Þ a = 45º
Now, sin a · sec a · tan a – cosec a · cos a · cot a
= sin 45º · sec 45º · tan 45º – cosec 45º · cos 45º · cot 45º
1 1
= ´ 2 ´1 - 2 ´ ´ 1 = 1 – 1 = 0 Ans.
2 2
30. The frequency distribution of weekly pocket money received by a group of students is
given below :

Pocket More More More More More More More More More More
money than than than than than than than than than than
in ( ) or or or or or or or or or or
equal equal equal equal equal equal equal equal equal equal
to 20 to 40 to 60 to 80 to 100 to 120 to 140 to 160 to 180 to 200
Number
of
students 90 76 60 55 51 49 33 12 8 4

Draw a ‘more than type’ ogive and from it, find median. Verify median by actual
calculations. [4]
Sol.

P
Q

R
https://www.arundeepselfhelp.info/index.php?route=product/product&path=180&product_id=395

Arundeep’s Solved Papers 12 Mathematics 2014 (Term I)


Taking pocket money (in Rs.) along x-axis and number of students along y-axis, plot the points
(20, 90), (40, 76), (60, 60), (80, 55), (100, 51), (120, 49), (140, 33), (160, 12), (180, 8) and (200,
4) on graph paper and join all these point by free hand gives the required more than type ogive.
Now mark a point representing 45 on y-axis by P, from P draw a line || to x-axis meeting ogive at
Q. From Q, draw QR perpendicular to x-axis.
Then abscissa of point R gives the required median
\ Md = 125.
The table of values is given as under :

Pocket money No. of


c.i. fi c.f
(in ` ) Students
More than or equal to 20 90 20–40 14
More than or equal to 40 76 40–60 30
More than or equal to 60 60 60–80 60–55 = 5 35
More than or equal to 80 55 80–100 55–51 = 4 39
More than or equal to 100 51 100–120 51–49 = 2 41
More than or equal to 120 49 120–140 49–33=16 57
More than or equal to 140 33 140–160 33–12=21 78
More than or equal to 160 12 160–180 12 – 8 = 4 82
More than or equal to 180 8 180–200 8–4 = 4 86
More than or equal to 200 4 200–220 4 90

N = 90 åi

Here, N = 90
N 90
\ = = 45
2 2
The cumulative frequency just greater than 45 be 51 and corresponding median class is 120–140

N
- cf 45 - 41 4 ´ 20
\ Median = l + 2 ´ h = 120 + ´ 20 = 120 + = 120 + 5 = ` 125
f 16 16

Hence Verified
31. Cost of living Index for some period is given in the following frequency distribution :

Index 1500–1600 1600–1700 1700–1800 1800–1900 1900–2000 2000–2100 2100–2200


No. of weeks 3 11 12 7 9 8 2

Find the mode and median for above data. [4]


https://www.arundeepselfhelp.info/index.php?route=product/product&path=180&product_id=395

Arundeep’s Solved Papers 13 Mathematics 2014 (Term I)


The table of values is given as under :

Index Number of weeks (f i ) cf


1500–1600 3 3
1600–1700 11 f0 14
1700–1800 12 f1 26
1800–1900 7 f2 33
1900–2000 9 42
2000–2100 8 50
2100–2200 2 52
N = Sfi = 52

Here N = 52
N 52
\ = = 26 , now commulative frequency which is just greater than or equal to 26 be
2 2
1700 – 1800.
\ Median class is 1700–1800, Here l = 1700 ; c.f. = 14 ; f = 12 ; h = 100

N
- cf
\ Median = l + 2 ´h
f

26 - 14
= 1700 + ´ 100
12

12
= 1700 + ´ 100 = 1800
12
Also, Maximum frequency is 12 which lies in 1700 – 1800.
\ Modal class is 1700–1800, Here l = 1700 ; f1 = 12 ; f0 = 11 ; f2 = 7
f1 - f0
Thus, mode = l + ´h
2 f1 - f0 - f 2

12 - 11 1
= 1700 + ´ 100 = 1700 + ´ 100 = 1700 + 16.67
24 - 11 - 7 6
= 1716.67 Ans.
https://www.arundeepselfhelp.info/index.php?route=product/product&path=180&product_id=395

MATHEMATICS 2014 TERM - II (OUTSIDE DELHI)


SET I
Time allowed : 3 hours Maximum marks : 90

General Instructions :
(i) All questions are compulsory.
(ii) The question paper consists of 34 questions divided into four sections – A, B, C and D.
(iii) Section A contains 8 questions of 1 mark each, which are multiple choice type questions, Section
B contains 6 questions of 2 marks each, Section C contains 10 questions of 3 marks each and
Section D contains 10 questions of 4 marks each.
(iv) Use of calculators is not permitted.
Section – A
Question numbers 1 to 8 carry 1 mark each. For each of these questions four alternative choices have
been provided of which only one is correct. Select the correct choice.

1. The first three terms of an AP The tangent at T meets QR at P. If


respectively are 3y – 1, 3y + 5 and 5y + 1. PT = 3.8 cm, then the length of
Then y equals : QR (in cm) is:
(a) – 3 (b) 4 (a) 3.8 (b) 7.6
(c) 5 (d) 2 (c) 5.7 (d) 1.9
Sol. (c) Since 3y – 1, 3y + 5 and 5y + 1 are in A.P. Sol. (b) Given, PT= 3.8 cm
\ 2 (3y + 5) = 3y – 1 + 5y + 1 We know that the lengths of the tangents
[If a, b, c are in A.P., b – a = c – b drawn to a circle from a point outside the
Þ 2b = a + c] circle are equal.
Þ 6y + 10 = 8y \ QP = PT = 3.8 cm
Þ 10 = 2y and PR = PT = 3.8 cm
Þ y=5 \ QR = QP + PR = (3.8 + 3.8) cm = 7.6 cm
2. In Fig. 1, QR is a common tangent to 3. In Fig. given below PQ and PR are two
the given circles, touching externally at tangents to a circle with centre O. If
the point T. ÐQPR = 46º, then ÐQOR equals :

Arundeep’s Solved Papers 14 Mathematics 2014 (Outside Delhi)


https://www.arundeepselfhelp.info/index.php?route=product/product&path=180&product_id=395

Arundeep’s Solved Papers 15 Mathematics 2014 (Outside Delhi)


(a) 67º (b) 134º 1 1
(c) 44º (d) 46º (a) (b)
36 2
Sol. (b) We know that tangent to a circle is
perpendicular to the radius at the point of 1 1
(c) (d)
contact. 6 4
\ ÐOQP = Ð ORP = 90º Sol. (d) Number of possible even numbers on
Now, in quadrilateral ORPQ, we have both dice = 9, i.e. (2, 2), (2, 4), (2, 6),
(4, 2), (4, 4), (4, 6), (6, 2), (6, 4), (6, 6).
ÐQOR + 90º + 46º + 90º = 360º
Total possibilities when two dice are rolled = 36
Þ ÐQOR + 226º = 360º
{(1, 1), (1, 2), (1, 3), (1, 4), (1, 5), (1, 6),
Þ ÐQOR = 360º – 226º = 134º (2, 1), (2, 2), (2, 3), (2, 4), (2, 5), (2, 6),
4. A ladder makes an angle of 60º with the (3, 1), (3, 2), (3, 3), (3, 4), (3, 5), (3, 6),
ground when placed against a wall. If (4, 1), (4, 2), (4, 3), (4, 4), (4, 5), (4, 6),
the foot of the ladder is 2 m away from (5, 1), (5, 2), (5, 3), (5, 4), (5, 5), (5, 6),
the wall, then the length of the ladder (6, 1), (6, 2), (6, 3), (6, 4), (6, 5), (6, 6)}
(in metres) is : \ Probability of getting an even number on
4 9 1
(a) (b) 4 3 both dice = =
3 36 4
6. A number is selected at random from
(c) 2 2 (d) 4
the numbers 1 to 30. The probability that
Sol. (d) Suppose AB is the ladder of length x m. it is a prime number is :
\ OA = 2 m, ÐOAB = 60º
2 1
(a) (b)
x 3 6
In right DAOB, sec 60º =
2
1 11
(c) (d)
x 3 30
Þ 2 = Þx=4m
2 Sol. (c) Total outcomes of selecting a number
from 30 numbers = 30
Favourable numbers (prime numbers) = 10,
i.e. (2, 3, 5, 7, 11, 13, 17, 19, 23, 29)
\ Required probability of selecting a prime
10 1
number = = .
30 3
7. If the points A (x, 2), B (– 3, – 4) and
C (7, – 5) are collinear, then the value
Hence the required length of ladder be of x is :
4 metre. (a) – 63 (b) 63
5. If two different dice are rolled together, (c) 60 (d) – 60
the probability of getting an even Sol. (a) If points A (x, 2), B (– 3, – 4) and
number on both dice, is : C (7, – 5) are collinear, then area of triangle
https://www.arundeepselfhelp.info/index.php?route=product/product&path=180&product_id=395

Arundeep’s Solved Papers 16 Mathematics 2014 (Outside Delhi)


formed by these points is 0. Sol. Consider the equation
[if A(x1, y1) ; B(x2, y2) and C(x3, y3) are 2x2 + ax – a2 = 0
vertices of DABC Þ 2x2 + 2ax – ax – a2 = 0
Then area of DABC Þ 2x (x + a) – a (x + a) = 0
1 Þ (2x – a) (x + a) = 0
= |x (y – y ) + x2 (y3 – y1) + x3 (y1 – y2)| Þ 2x – a = 0 or x + a = 0
2 1 2 3
a
1 Þ x= or x = – a
Þ [x (– 4 + 5) – 3 (– 5 – 2) + 7 (2 + 4)] = 0 2
2
10. The first and the last terms of an AP
1 are 5 and 45 respectively. If the sum of
Þ [x + 21 + 42] = 0 Þ x + 63 = 0 all its terms is 400, find its common
2
difference.
Þ x = – 63
Sol. Let total no. of terms of given A.P. be n.
8. The number of solid spheres, each of
Given, a = 5, an = 45 and Sn = 400
diameter 6 cm that can be made by
melting a solid metal cylinder of height n
We know, Sn = ( a + an )
45 cm and diameter 4 cm, is : 2
(a) 3 (b) 5
n n
(c) 4 (d) 6 Þ 400 = (5 + 45) Þ 400 = ´ 50
2 2
Sol. Given diameter of sphere = 6 cm
400
6 Þ 400 = 25n Þ n =
\ radius of sphere = cm = 3 cm = r 25
2
Þ n = 16
4 3 \ a16 = 45
\ volume of each solid sphere = pr
3 Þ a + 15d = 45 Þ 5 + 15d = 45

4p 3 40 8
´ 3 = 36p Þ 15d = 40 Þ d= =
= 15 3
3
Hence the required common difference of
4
Also, diameter of solid cylinder = R = cm = 2cm 8
2 given A.P. be .
3
and height of solid cylinder = h = 45 cm
11. Prove that the line segment joining the
\Volume of solid cylinder =pR2h =p × 22 × 45=180p
points of contact of two parallel tangents
\ Required no. of solid sphere made of a circle, passes through its centre.
volume of cylinder 180p Sol. Given : PQ and RS are two parallel tangents
= = =5
volume of each sphere 36p to a circle at B and A respectively. O is the
centre of the circle.
\ Ans (b)
To prove : AB passes through O.
Section – B Construction : Join OA and OB.
Question numbers 9 to 14 carry 2 marks each. Proof : OB is perpendicular to PQ.
9. Solve the quadratic equation [Tangent is perpendicular to radius at the
2x2 + ax – a2 = 0 for x. point of contact.]
https://www.arundeepselfhelp.info/index.php?route=product/product&path=180&product_id=395

Arundeep’s Solved Papers 17 Mathematics 2014 (Outside Delhi)


\ DOQP @ DORP (SSS axiom of congruency)
\ ÐOPQ = ÐOPR = 60º (c.p.c.t)
In right-angled triangle OQP, we have
OP
PQ
= sec 60º = 2 Þ OP = 2PQ

13. Rahim tosses two different coins


Now, PQ || RS Þ BO (Produced to RS) is simultaneously. Find the probability of
perpendicular to RS. ...(i) getting at least one tail.
[A line perpendicular to one of the two Sol. When two different coins are tossed
parallel lines is perpendicular to other line simultaneously, then total possibilities = 4,
also] i.e. (H, H), (H, T), (T, H), (T, T)
Also, OA is perpendicular to RS Number of favourable outcomes for at least
[Reason as above] ...(ii) one tail = 3, i.e. (H, T), (T, H), (T, T).
From (i) and (ii), OA and OB must coincide \ Required probability of getting at least one
as only one line can be drawn perpendicular
3
from a point outside the line to the line. tail = .
4
\ A, O, B are collinear.
Þ AB Passes through O, the centre of the 14. In given below, a square OABC is
circle. inscribed in a quadrant OPBQ of a circle.
If OA = 20 cm, find the area of the
12. If from an external point P of a circle shaded region. (Use p = 3.14)
with centre O, two tangents PQ and PR
are drawn such that ÐQPR = 120º, prove
that 2PQ = PO.
Sol. Given : PQ and PR are tangents from point
P to circle with centre O.
Also, ÐQPR = 120º
To Prove : 2PQ = OP
Construction : Join OQ, OP and OR
Proof : In triangles OQP and ORP, we have Sol. OA = 20 cm.
OQ = OR (radii) \ OB = 2 × side
OP = OP (common) (Q OB is a diagonal of the square)
PQ = PR
[tangents drawn from a point outside the Þ OB = 20 2 cm
circle to the circle are equal in length]
https://www.arundeepselfhelp.info/index.php?route=product/product&path=180&product_id=395

Arundeep’s Solved Papers 18 Mathematics 2014 (Outside Delhi)


\ Shaded area = Area of sector OPBQ [_ an = a + (n – 1) d]
– Area of square OABC 1 1
Also, a9 = Þ a + 8d = ...(ii)
90° 7 7
= ´ p (20 2) 2 - (20) 2
360º Subtracting (i) from (ii), we get
ép ù é 3.14 ù 1 1
= 400 ê ´ 2 - 1ú = 400 êë 2 - 1úû a + 8d – a – 6d = -
ë4 û 7 9
= 400 [1.57 – 1] = 400 × 0.57 = 228 cm2
9-7 2 1
Section – C Þ 2d = = Þ d=
63 63 63
Question numbers 15 to 24 carry 3 marks each.
Substituting the value of d in (i), we get
4 5
15. Solve the equation -3= ; 1 1
x 2x + 3 a + 6´ =
63 9
3
x ¹ 0, - , for x. 1 6 7–6 1
2 Þ a = - = =
9 63 63 63
4 5
Sol. Given equation be, -3= , 1 1
x 2x + 3 \ a = and d =
63 63
3
x ¹ 0, -
2 1 1
Now, a63 = a + 62d = + 62 ´
63 63
4 - 3x 5
Þ =
x 2x + 3 1 + 62 63
= = =1
Þ (4 – 3x) (2x + 3) = 5x 63 63
Þ 8x + 12 – 6x2 – 9x = 5x 17. Draw a right triangle ABC in which
Þ 6x2 + 6x – 12 = 0 AB = 6 cm, BC = 8 cm and ÐB = 90º.
Þ 6 (x2 + x – 2) = 0 Draw BD perpendicular from B on AC
Þ x2 + x – 2 = 0 and draw a circle passing through the
Þ x2 + 2x – x – 2 = 0 points B, C and D. Construct tangents
Þ x (x + 2) – 1 (x + 2) = 0 from A to this circle.
Þ (x + 2) (x – 1) = 0 Sol. Steps of construction :
Þ Either x + 2 = 0 or x – 1 = 0 1. BC = 8 cm is drawn.
Þ x = – 2, 1
2. ÐABC = 90º is drawn.
1 3. AB = 6 cm is cut and AC is joined.
16. If the seventh term of an AP is and
9
4. Triangle ABC is the required triangle, right
1 angled at B.
its ninth term is , find its 63rd term.
7 5. From point B, an arc is drawn cutting AC
Sol. Let a be the first term and d be the common at X and Y.
difference of the given AP. 6. Arc XY is bisected, by drawing two arcs
1 1 of same radius cutting at N.
Given, a7 = Þ a + 6d = ...(i)
9 9
https://www.arundeepselfhelp.info/index.php?route=product/product&path=180&product_id=395

Arundeep’s Solved Papers 19 Mathematics 2014 (Outside Delhi)


19. Two ships are there in the sea on either
side of a light house in such a way that
the ships and the light house are in the
same straight line. The angles of
depression of two ships as observed from
the top of the light house are 60º and
45º. If the height of the light house is
200 m, find the distance between the two
ships. [Use 3 = 1.73]
Sol. Let AB be the light house of height 200 m.
C and D are two ships on either sides of
light house with angles of depression 60º
7. BN is joined meeting AC at D. and 45º respectively.
8. BD is perpendicular to AC. \ ÐACB = 60º and ÐADB = 45º
9. As ÐBDC = 90º, therefore, BC is [Alternate angles in both cases]
hypotenuse of DBDC. In right-angled triangle ABC,
10. Take point O as mid-point of BC.
BC
11. With O as centre, a circle is drawn passing = cot 60º
AB
through B, D and C.
12. AO is joined. 1 200
Þ BC = 200× = m ...(i)
13. With AO as diameter, a circle is drawn 3 3
cutting the circle BDC at R and B
(Q ÐABO = 90°) join AR.
14. AB and AR are the tangents from A to circle
BDC.
18. If the point A (0, 2) is equidistant from
the points B (3, p) and C (p, 5), find p.
Also find the length of AB.
Sol. Q A (0, 2) is equidistant from the points
B (3, p) and C (p, 5).
\ AB = AC Þ AB2 = AC2 In right-angled triangle ABD,
Þ (0 –3)2 + (2 – p)2 = (0 – p)2 + (2 – 5)2 BD
Þ 9 + 4 – 4p + p2 = p2 + 9 = cot 45º
AB
Þ 4p = 4 Þ p=1
Þ BD = 200 × 1 = 200 m ...(ii)
\ Coordinates of point B be (3, 1)
\ Distance between ships = CD = CB + BD
\ AB = (0 - 3)2 + (2 - 1) 2
200 200 3
= + 200 = + 200
= 9 + 1 = 10 units 3 3
https://www.arundeepselfhelp.info/index.php?route=product/product&path=180&product_id=395

Arundeep’s Solved Papers 20 Mathematics 2014 (Outside Delhi)

200 ´ 1.73 346


= + 200 = + 200 = (115.33 + 200) m
3 3
= 315.33 m.
20. If the points A (– 2, 1), B (a, b) and C (4, – 1) are collinear and a – b = 1, find the values
of a and b.
Sol. Q Points A (– 2, 1), B (a, b) and C (4, – 1) are collinear..
\ Area of triangle formed by these points is zero.
[if A(x1, y1) ; B(x2, y2) and C(x3, y3) are vertices of DABC
1
Then area of DABC = |x (y – y ) + x2 (y3 – y1) + x3 (y1 – y2)|]
2 1 2 3
1
Þ [– 2 (b + 1) + a (– 1 – 1) + 4 (1 – b)] = 0
2
Þ – 2b – 2 – 2a + 4 – 4b = 0 Þ 2a + 6b = 2
Þ a + 3b = 1 ...(i)
Also, a – b = 1 Þ a = b + 1 ...(ii)
Putting the value of a from (ii) in (i), we get
b + 1 + 3b = 1 Þ 4b = 0 Þ b = 0
\ from eqn. (ii), a = 1
Thus, a = 1, b = 0
21. In Fig. given below a circle is inscribed in an equilateral
triangle ABC of side 12 cm. Find the radius of inscribed
circle and the area of the shaded region.
[Use p = 3.14 and 3 = 1.73]

Sol. Given : An equilateral triangle of side 12 cm.


Let radius of incircle be r. Join OA, OB and OC.
Also, join OD, OE and OF.
Here, AB, BC and AC are the tangents to the circle.
\ OD ^ BC, OE ^ AC and OF ^ AB.
[Radius is perpendicular to the tangent at the point of contact]
Now, ar (ABC) = ar (AOB) + ar (BOC) + ar (COA)

3 1 1 1 3
Þ · (12) 2 = ´ 12 ´ r + ´ 12 ´ r + ´ 12 ´ r [_ area of equilateral DABC = (side)2]
4 2 2 2 4
Þ 36 3 = 18r

Þ r = 2 3 cm = 2 × 1.73 = 3.46 cm
https://www.arundeepselfhelp.info/index.php?route=product/product&path=180&product_id=395

Arundeep’s Solved Papers 21 Mathematics 2014 (Outside Delhi)


Thus, required radius of incircle = 3.46 cm 3
Area of shaded portion = Area of equilateral cm i.e. 5 cm, 3. 5 cm and 1.5 cm
2
triangle DABC – area of circle
respectively.
3 Perimeter of shaded region
= (12) 2 - p (2 3) 2
4 = length of arc PSR + length of arc PAQ +
length of arc QTR
= [p (5) + p (3.5) + p (1.5)] cm
= p × 10 cm = 3.14 × 10 cm = 31.4 cm

2
= 36 3 - 12p = 12 (3 × 1.73 – 3.14) cm
= 12 (5.19 – 3.14) cm2 = 12 × 2.05 cm2 23. A farmer connects a pipe of internal
= 24.60 cm2 diameter 20 cm from a canal into a
22. In Fig. given below PSR, RTQ and PAQ cylindrical tank which is 10 m in
are three semicircles of diameters 10 diameter and 2 m deep. If the water
cm, 3 cm and 7 cm respectively. Find flows through the pipe at the rate of 4
the perimeter of the shaded region. km per hour, in how much time will the
[Use p = 3.14] tank be filled completely ?
Sol. Internal diameter of pipe = 20 cm
1
\ Internal radius of pipe = 10 cm = m
10

Sol. Diameters of Semicircle PSR, PAQ and


QTR are 10 cm, 7 cm and 3 cm and hence
10 7
corresponding radii are cm, cm and
2 2
https://www.arundeepselfhelp.info/index.php?route=product/product&path=180&product_id=395

Arundeep’s Solved Papers 22 Mathematics 2014 (Outside Delhi)


In 1 h, 4 km length of water flows into the In right-angled triangle OAB, we have
tank.
AB r1 1
\ In 1 hour, the volume of water which flows = tan 30º Þ =
out OA 10 3

2 10
æ1ö Þ r1 = cm.
= p ç ÷ × 4 × 1000 m3 3
è 10 ø
[_ 1 km = 1000 m] In right angled triangle OCD, we have
Volume of cylindrical tank = p (5)2 × 2 m2 CD r2 1
= tan 30º Þ =
[_ diameter of tank = 10 m, height = 2 m] OC 20 3
\ Required time taken to fill the tank
Volume of tank Þ r2 = 20 cm
= 3
Volume of water flows in 1 hour
Frustum is drawn into wire of diameter
p ´ 25 ´ 2 ´ 100
= h 1
p ´ 4 ´ 1000 cm and length x cm (say)
12
=
5
h = 1 h 15 min. \ Volume of wire = volume of frustum
4
1
24. A solid metallic right circular cone 20 Þ pr2x = ph [ r12 + r22 + r1 · r2 ]
3
cm high and whose vertical angle is 60º,
is cut into two parts at the middle of its 2
æ 1 ö
height by a plane parallel to its base. If Þ p ç ÷ ´x
the frustum so obtained be drawn into è 24 ø

1
cm, find the p ´ 10 éæ 10 ö 2 æ 20 ö 2 10 20 ù
a wire of diameter êç
12 = ÷ +ç ÷ + · ú
3 êëè 3 ø è 3ø 3 3 úû
length of the wire.
Sol. Cone is cut by plane PB and PQDB is a 1 10 é100 400 200 ù
Þ ´x = + +
3 êë 3 3 úû
frustrum of cone.
576 3
OA = AC = 10 cm, AB = r1 m ; CD = r2
x 10 700
Þ = ´
576 3 3
7000
Þ x = ´ 576 = 7000 × 64 cm
9
= 448000 cm
448000
\ length of wire = km = 4.48 km
100000
[_ 1 km = 1000 m = 1000 × 100 cm
1
Vertical angle = 60º, so Semi vertical angle = 30º Þ 1 cm = km ]
100000
https://www.arundeepselfhelp.info/index.php?route=product/product&path=180&product_id=395

Arundeep’s Solved Papers 23 Mathematics 2014 (Outside Delhi)


Section – A Proof : Consider triangles OAP and OBP.
Question numbers 25 to 34 carry 4 marks each. ÐOAP = ÐOBP = 90º ...(i)
25. The difference of two natural numbers [Radius is perpendicular to the tangent at
is 5 and the difference of their the point of contact]
1
reciprocals is . Find the numbers.
10
Sol. Let numbers be x and y, such that x > y.
According to question,
x–y=5 ...(i)
1 1
If x > y, then >
y x
OA = OB (radii) ...(ii)
1 1 1
Now, y - x = ...(ii) OP is common ...(iii)
10
\ DOAP @ DOBP (RHS)
From (i) and (ii), we have [from (i), (ii), (iii)]
Þ AP = BP (cpct)
1 1 1
- = 27. The angles of elevation and depression
y y + 5 10
of the top and the bottom of a tower from
y+5- y the top of a building, 60 m high, are 30º
1
Þ = and 60º respectively. Find the difference
y ( y + 5) 10 between the heights of the building and
Þ 50 = y2 + 5y the tower and the distance between
Þ y2 + 5y – 50 = 0 them.
Þ y2 + 10y – 5y – 50 = 0 Sol. Let AB be the height of the tower and CD
Þ y (y + 10) – 5 (y + 10) = 0 be the building of height 60 m.
Þ (y + 10) (y – 5) =0 ÐACE = 30º, ÐECB = 60º
Þ Either y + 10 = 0 or y – 5 = 0
Þ y = – 10 (rejected)
Since y be any natural number.
Thus, y = 5.
\ from (i), x = 10
\ Required natural numbers are 10 and 5.
26. Prove that the length of the tangents
drawn from an external point to a circle
are equal.
Sol. Given : A circle C (O, r). P is a point
outside the circle and PA and PB are
tangents to a circle.
To Prove : PA = PB
Construction : Draw OA, OB and OP.
https://www.arundeepselfhelp.info/index.php?route=product/product&path=180&product_id=395

Arundeep’s Solved Papers 24 Mathematics 2014 (Outside Delhi)


Let difference between heights of building (iii) Favourable outcomes for a perfect square
and tower be y m and distance between the = 7, i.e., 1, 4, 9, 16, 25, 36 and 49.
tower and building by x m. \ Probability of getting a card with perfect
\ CD = EB = 60 m, CE = BD = x m. 7 1
In right-angled triangle CEB. square number = = .
49 7
CE (iv) Favourable outcomes for an even prime
= cot 60º number = 1, i.e., 2.
BE
\ Probability of getting a card numbered even
1
Þ CE = ´ 60 m = 60 3 m 1
3 3 prime = .
49
Þ BD = CE = 20 3 m 29. Find the ratio in which the point P (x, 2)
divides the line segment joining the
\ Distance between tower and building is
points A (12, 5) and B (4, – 3). Also find
20 3 m. the value of x.
Sol. Let P (x, 2) divides the join of A (12, 5) and
In right-angled triangle CEA, B (4, – 3) in the ratio k : 1.
Then by section formula, we have
AE 1
= tan 30º Þ y = 20 3 ´ = 20 m \ Point of division is P
CE 3
æ 4k + 12 -3k + 5 ö
\ Difference between the height of tower and ç , ÷ = (x, 2)
building is 20 m. è k +1 k +1 ø
28. A bag contains cards numbered from 1
to 49. A card is drawn from the bag at
random, after mixing the cards
thoroughly. Find the probability that the
number on the drawn card is : 4k + 12 -3k + 5
Þ = x and =2 ...(i)
(i) an odd number k +1 k +1
(ii) a multiple of 5 -3k + 5
(iii) a perfect square Consider, =2
k +1
(iv) an even prime number Þ –3k + 5 = 2k + 2
Sol. Total cards in the bag = 49.
3
\ Total number of outcomes = 49, when one Þ – 5k = – 3 Þ k =
card is drawn. 5
(i) No. of favourable outcomes for odd 3
number = 25, i.e. 1, 3, 5, .... 49. \ Required ratio is k : 1, i.e., :1 i.e. 3 : 5
5
25
\ Probability of getting an odd number = .
Now, substituting k =
3
in (i), we get
49 5
(ii) No. of favourable outcomes for multiple
of 5 = 9, i.e., 5, 10, 15, .... 45. 3
4´ + 12
x = 5 12 + 60 72
9 = = =9
\ Probability of getting a multiple of 5 = . 3 3+5 8
49 +1
5
https://www.arundeepselfhelp.info/index.php?route=product/product&path=180&product_id=395

Arundeep’s Solved Papers 25 Mathematics 2014 (Outside Delhi)


30. Find the values of k for which the Consider, a1 + a2 + .... + a10 = 210
quadratic equation 10
(k + 4) x2 + (k + 1) x + 1 = 0 has equal Þ [a + a ] = 210
2 1 10
roots. Also find these roots.
Sol. Given equation is n
[using Sn = (a + l ) where l is the last term]
(k + 4) x2 + (k + 1) x + 1 = 0 ...(i) 2
On comparing with ax2 + bx + c = 0, a ¹ 0 Þ 5 [a + a + 9d] = 210
Here, a = k + 4 ; b = k + 1 ; c = 1 Þ 2a + 9d = 42 ...(i)
first we note k + 4 ¹ 0 Þ k ¹ – 4 Again consider,
Here D = b2 – 4ac = (k + 1)2 – 4 · (k + 4) a36 + a37 + .... + a50= 2575
= k2 + 2k + 1 – 4k – 16
15
= k2 – 2k – 15 Þ [ a + a50 ] = 2565
2 36
Q roots are equal, therefore, D = 0
Þ k2 – 2k – 15 = 0 Þ a + 35d + a + 49d = 171 × 2
Þ k2 – 5k + 3k – 15 = 0 Þ 2a + 84d = 171 × 2
Þ k (k – 5) + 3 (k – 5) = 0 Þ a + 42d = 171 ...(ii)
Þ (k + 3) (k – 5) = 0 eqn. (i) – 2 × eqn. (ii), we have
Þ k + 3 = 0 or k – 5 = 0 Þ k = – 3, 5 2a + 9d – 2a – 84d Þ 42 – 342
when k = – 3, then from (i), we get Þ – 75d = – 300 Þ d = 4
(– 3 + 4) x2 + (– 3 + 1) x + 1 = 0 \ From (ii), a + 42 × 4 = 171
Þ x2 – 2x + 1 = 0 Þ a = 171 – 168 = 3
Þ (x – 1)2 = 0 \ a = 3 and d = 4.
Þ (x – 1) · (x – 1) = 0 Hence required AP is a + a + d, a + 2d,
Þ x = 1, 1 a + 3d, ......
when k = 5, then from (i), we get i.e. 3, 3 + 4, 3 + 8, 3 + 12, .....
(5 + 4) x2 + (5 + 1) x + 1 = 0 i.e. 3, 7, 11, 15, ....
Þ 9x2 + 6x + 1 = 0
32. Prove that a parallelogram
Þ (3x + 1)2 = 0 circumscribing a circle is a rhombus.
Þ (3x + 1) · (3x + 1) = 0
Sol. Given : A parallelogram ABCD, which
1 1 circumscribes a circle.
Þ x =- , -
3 3 To prove : ABCD is a rhombus.
1 1 Proof : We know that the lengths of the
\ Roots are 1, 1 or - , - . tangents drawn from a point outside the
3 3
circle to the circle are equal in length. Using
31. In an AP of 50 terms, the sum of first
this result, we get
10 terms is 210 and the sum of its last
15 terms in 2565. Find the A.P. AP = AS, BP = BQ, CQ = CR and DR = DS
Sol. Let a be the first term and d be the common ...(i)
difference of the given AP. Consider, AB + CD = AP + BP + CR + DR
\ According to question, we have = AS + BQ + CQ + DS [using (i)]
a1 + a2 + .... + a10 = 210 = (AS + DS) + (BQ + CQ)
and a36 + a37 + .... + a50 = 2565 = AD + BC
https://www.arundeepselfhelp.info/index.php?route=product/product&path=180&product_id=395

Arundeep’s Solved Papers 26 Mathematics 2014 (Outside Delhi)

0.5
\ Radius of spherical ball = r = cm
2
\ Volume of spherical ball
3
4 4 æ 0.5 ö
= pr 3 = p ç ÷ cm
2
3 3 è 2 ø
Let x be the number of spherical balls be
dropped.

Þ AB + AB = BC + BC
[Q AB = CD ; AD = BC, opp. sides of ||gm
are equal]
Þ 2AB = 2BC Þ AB = BC
As adjacent sides of parallelogram ABCD
are equal, hence, parallelogram ABCD is a
rhombus.
33. Sushant has a vessel, of the form of an
inverted cone, open at the top, of height
11 cm and radius of top as 2.5 cm and is
full of water. Metallic spherical balls We know volume of water displaced is
each of diameter 0.5 cm are put in the equal to volume of body immersed.
2
vessel due to which th of the water in 4 æ 0.5 ö
3
2 p
5 \ x´ pç ÷ = ´ (2.5) ´ 11
2
the vessel flows out. Find how many balls 3 è 2 ø 5 3
were put in the vessel. Sushant made
2 1 8
the arrangement so that the water that Þ x = (2.5)2 ´ 11 ´ ´
flows out irrigates the flower beds. What 5 (0.5)3 4
value has been shown by Sushant ?
4 ´ 2.5 ´ 2.5 ´ 11
Sol. Radius of base of the cone = R = 2.5 cm Þ x =
and height of cone = h = 11 cm. 5 ´ 0.5 ´ 0.5 ´ 0.5
\ Volume of water in cone
44 ´ 25 ´ 25 ´ 10
p = = 440
= R 2h 25 ´ 25
3
440 balls were put in the vessel. As the water
1 displaced was used for irrigation of flower
= p (2.5)2 ´ 11cm3
3 bed, it shows Sushant is concerned for the
conservation and best use of natural
2
th of the volume of water in cone resources.
5
34. From a solid cylinder of height 2.8 cm
2 p and diameter 4.2 cm, a conical cavity of
= ´ (2.5) 2 ´ 11cm3 ...(i)
5 3 the same height and same diameter is
Given diameter of spherical ball = 0.5 cm. hollowed out. Find the total surface area
https://www.arundeepselfhelp.info/index.php?route=product/product&path=180&product_id=395

Arundeep’s Solved Papers 27 Mathematics 2014 (Outside Delhi)


For cone :
é 22 ù
of the remaining solid. ê Take p = ú
ë 7û 4.2
Radius of base (r) = cm = 2.1 cm,
Sol. The shaded conical cavity is hollowed out. 2
For cylinder : height (h) = 2.8 cm
4.2
Radius of base (r) = cm = 2.1 cm Slant height (l) = r 2 + h2 = (2.1)2 + (2.8) 2
2
height (h) = 2.8 cm
= 4.41 + 7.84 = 12.25 = 3.5 cm
Total surface Area of remaining solid
= curved surface area of cylinder + Area
of top circular base + curved surface area
of cone
= 2prh + pr2 + prl = pr [2h + r + l]
= p × 2.1 [2 × 2.8 + 2.1 + 3.5] cm2

22
= ´ 2.1 ´ [5.6 + 5.6]
7

22
=
× 2.1 × 11.2 cm2 = 73.92 cm2
7
Set-II (Uncommon Questions to Set-I)
NOTE : Except for the following questions, all the
remaining questions have been asked in previous set.
14. The first and the last terms of an AP
3 1 2
are 7 and 49 respectively. If sum of all 22. Solve the equation - = ;
its terms is 420, find its common x + 1 2 3x - 1
1
difference. x ¹ -1, x ¹ , for x.
Sol. Let a be the first term, d be the common 3
3 1 2
difference and an be the last term of the Sol. Consider the equation - = ,
AP, with n terms and Sn be the sum to n x + 1 2 3x - 1
terms of given A.P. 1
where x ¹ – 1,
Given, a = 7, an = 49, Sn = 420 3
n n 6 - ( x + 1) 2
\ Sn = ( a + an ) Þ 420 = (7 + 49) Þ =
2 2 2 ( x + 1) 3x - 1
Þ 840 = 56n Þ n = 15
\ a15 = 49 5- x 2
Þ 2 ( x + 1) =
Þ a + 14d = 49 Þ 7 + 14d = 49 3x - 1
Þ 14d = 42 Þ d=3 Þ (3x – 1) (5 – x) = 4 (x + 1)
\ Required common difference of given A.P. Þ 15x – 3x2 – 5 + x = 4x + 4
is 3.
Þ 3x2 – 12x + 9 = 0
https://www.arundeepselfhelp.info/index.php?route=product/product&path=180&product_id=395

Arundeep’s Solved Papers 28 Mathematics 2014 (Outside Delhi)


Þ 3 (x2 – 4x + 3) = 0
\ Radius of circle = (2 + 1)2 + ( -21 - 7) 2
Þ 2
x – 4x + 3 = 0
Þ x2 – 3x – x + 3 = 0
= 9 + 784 = 793 units
Þ x (x – 3) – 1 (x – 3) = 0
Þ (x – 1) (x – 3) = 0 24. If the points P (– 3, 9), Q (a, b) and R
Þ Either x – 1 = 0 or x – 3 = 0 (4, – 5) are collinear and a + b = 1, find
the values of a and b.
Þ x = 1, 3
23. Points A (– 1, y) and B (5, 7) lie on a Sol. Since the points P (– 3, 9), Q (a, b) and R
circle with centre O (2, – 3y). Find the (4, – 5) are collinear, therefore, area of
values of y. Hence find the radius of the triangle formed by these points is zero.
circle. [if A(x1, y 1) ; B(x 2, y2) and (x3 , y 3) are
Sol. As points A (– 1, y) and B (5, 7) lie on a vertices of DABC
circle with centre O (2, – 3y). Then area of DABC
\ OA = OB (radii) Þ OA2 = OB2 1
Þ (2 + 1)2 + (– 3y – y)2 = (2 – 5)2 = |x (y – y ) + x2 (y3 – y1) + x3 (y1 – y2)|]
2 1 2 3
+ (– 3 y – 7)2
Þ 9 + 16y2 = 9 + 9y2 + 49 + 42y 1
Þ [– 3 (b + 5) + a (– 5 – 9) + 4 (9 – b)] = 0
Þ 7y2 – 42y – 49 = 0 2
Þ 7 (y2 – 6y – 7) = 0 Þ –3b – 15 – 14a + 36 – 4b = 0
Þ y2 – 7y + y – 7 = 0 Þ 14a + 7b – 21 = 0 ...(i)
Þ y (y – 7) + 1 (y – 7) = 0 Þ 2a + b = 3 ...(ii)
Þ (y + 1) (y – 7) = 0 Also, a+b=1
Þ Either y + 1 = 0 or y – 7 = 0 On subtracting, we get a = 2
Þ y =– 1 or 7 Substituting the value of a in (i), we get
when y = – 1
4+b=3 Þ b=–1
\ a=2b=–1
31. The difference of two natural numbers
is 3 and the difference of their
3
reciprocals is . Find the numbers.
28
Sol. Let the numbers be x and y, where x > y.
then centre O is (2, 3) and point A is (– 1, – 1)
Given, x – y = 3 ....(i)
\ Radius of circle = (2 + 1)2 + (3 + 1) 2 1 1
If x > y, then > .
y x
= 9 + 16 = 25 = 5 units
When y = 7, 1 1 3
then centre O is (2, – 21) and point A is \ y - x = 28
(– 1, 7)
https://www.arundeepselfhelp.info/index.php?route=product/product&path=180&product_id=395

Arundeep’s Solved Papers 29 Mathematics 2014 (Outside Delhi)

1 1 point outside the circle to the line is


3
Þ - = [using eqn. (i)] perpendicular segment.
y y+3 28

y +3- y 3
Þ =
y ( y + 3) 28

3 3
Þ =
y2 + 3y 28
l
Þ y2 + 3y = 28
Þ y2 + 3y – 28 = 0 Hence, OA is ^ to tangent l.
Þ y2 + 7y – 4y – 28 = 0 Thus, tangent at any point of a circle is
perpendicular to the radius through the point
Þ y (y + 7) – 4 (y + 7) = 0
of contact.
Þ (y – 4) (y + 7) = 0 33. All the black face cards are removed
Þ Either y – 4 = 0 or y + 7 = 0 from a pack of 52 playing cards. The
Þ y = 4 or – 7 remaining cards are well shuffled and
then a card is drawn at random. Find
Since – 7 is rejected as it is not a natural
the probability of getting a :
number, therefore, y = 4
(i) face card
Thus, from (i) ; x – 4 = 3 Þ x = 7
(ii) red card
\ Required numbers are 7 and 4.
(iii) black card
32. Prove that the tangent at any point of a
(iv) king
circle is perpendicular to the radius
through the point of contact. Sol. In total 52 cards, 6 cards are black face
cards which have been removed. [Face
Sol. Given : A circle with centre O, line l is
cards are ; 2 black Jack, 2 black Queen, 2
tangent to the circle at A.
black King]
To prove : Radius OA is perpendicular to
\ Remaining cards = (52 – 6) = 46
the tangent at A.
Total possibilities of drawing a card = 46.
Construct : Take a point P, other then A,
(i) Favourable outcomes for a face card (king,
on tangent l. Join OP, meeting the circle at R.
queen, jack) are 6 (2 red kings + 2 red
Proof : We know that tangent to the circle queens + 2 red jacks)
touches, the circle at one point and all other
\ Required probability of getting a face card
points on the tangent lie in the exterior of a
circle. 6 3
= =
\ OP > OR (radius of circle) 46 23
Þ OP > OA (ii) Favourable outcomes for a red card are 26.
(Q OR = OA, radius of circle) \ Required probability of getting a red card
26 13
Þ OA < OP = = .
46 23
Þ OA is the smallest segment, from O to a
point on the tangent. (iii) Favourable outcomes for a black card are
(26 – 6) = 20, as all black face cards have
We know that smallest line segment from a
been removed.
https://www.arundeepselfhelp.info/index.php?route=product/product&path=180&product_id=395

Arundeep’s Solved Papers 30 Mathematics 2014 (Outside Delhi)


\ Required probability of getting a black card Q roots are equal, therefore, D = 0
20 10 Þ 4 (k2 – k) = 0 Þ k2 – k = 0
= = .
46 23 Þ k (k – 1) = 0
(iv) Favourable outcomes for a king are 2(2 red Þ Either k = 0 or k – 1 = 0
kings) Þ k = 0 or k = 1
2 1 When k = 0, then from (i), we get
\ Probability of getting a king = = .
(0 + 1) x2 + 2 (0 + 1) x + 1 = 0
46 23
34. Find the values of k for which the Þ x2 + 2x + 1 = 0 Þ (x + 1)2 = 0
quadratic equation Þ (x + 1) (x + 1) = 0 Þ x = – 1, – 1
(3k + 1) x2 + 2 (k + 1) x + 1 = 0 has equal \ Required roots are – 1, – 1.
roots. Also find the roots. When k = 1, then from (i), we get
Sol. Consider the given equation
(3 + 1) x2 + 2 (1 + 1) x + 1 = 0
(3k + 1) x2 + 2 (k + 1) x + 1 = 0 ...(i)
Þ 4x2 + 4x + 1 = 0 Þ (2x + 1)2 = 0
1 Þ (2x + 1) (2x + 1) = 0
First we notice 3k + 1 ¹ 0 Þ k ¹ -
3
1 1
On comparing given eqn. with ax2 + bx + c = 0 Þ x =- , -
2 2
Here a = 3k + 1 ; b = 2(k + 1) ; c = 1
1 1
Here discriminant = b2 – 4ac \ Roots are - , -
2 2
= [2 (k + 1)]2 – 4 (3k + 1)
= 4 [k2 + 2k + 1 – 3k – 1] = 4 [k2 – k] 1 1
Hence, roots are – 1, – 1 or - , - .
2 2

Set-III (Uncommon Questions to Set-I and Set-II)


NOTE : Except for the following questions, all the
remaining questions have been asked in previous set.

73n
14. The first and the last terms of an AP Þ 730 = Þ n = 20
are 8 and 65 respectively. If the sum of 2
all its terms is 730, find its common \ a20 = 65
difference. Þ a + 19d = 65 Þ 8 + 19d = 65
Sol. Let a be the first term, d be the common
Þ 19d = 57 Þ d=3
difference, an be the last term and Sn be
the sum to first n terms of the given AP. \ Required common difference of given A.P.
be 3.
Given : a = 8, an = 65, Sn = 730
22. If the points A (– 1, – 4), B (b, c) and
n C (5, – 1) are collinear and 2b + c = 4,
Now, Sn = ( a + an )
2 find the values of b and c.
n Sol. Since the points A (– 1, – 4), B (b, c) and
Þ 730 = (8 + 65) C (5, – 1) are collinear, therefore, area of
2
triangle formed by these points is zero.
https://www.arundeepselfhelp.info/index.php?route=product/product&path=180&product_id=395

Arundeep’s Solved Papers 31 Mathematics 2014 (Outside Delhi)


[if A(x1, y 1) ; B(x 2, y2) and (x3 , y 3) are \ AP = (2 + 2)2 + (2 + 1) 2
vertices of DABC
Then area of DABC = 16 + 9 = 25 = 5 units
1 When k = – 3, then point A is (– 2, – 3)
= |x (y – y ) + x2 (y3 – y1) + x3 (y1 – y2)|]
2 1 2 3
\ AP = (2 + 2)2 + (2 + 3) 2
1
Þ [– 1(c + 1) + b (– 1 + 4) + 5 (– 4 – c)] = 0 = 16 + 25 = 41 units
2
Þ – c – 1 + 3b – 20 – 5c = 0 14 5
24. Solve the equation -1= ;
x+3 x+1
Þ 3b – 6c = 21
Þ b – 2c = 7 ...(i) x ¹ – 3, – 1, for x.
Sol. Consider the given equation,
Also, 2b + c = 4 ...(ii) [given]
14 5
Multiplying (ii) by 2 and then adding to (i), -1 = , where x ¹ – 3, – 1
we get x+3 x +1
b – 2c + 4b + 2c = 7 + 8 14 - x - 3 5
Þ =
Þ 5b = 15 x+3 x +1
Þ b=3
11 - x 5
Substituting the value of b in (i), we get Þ =
x+3 x +1
3 – 2c = 7 Þ 2c = – 4 Þ c = – 2
Þ (11 – x) (x + 1) = 5 (x + 3)
\ b = 3, c = – 2
Þ 11x + 11 – x2 – x = 5x + 15
23. If the point P (2, 2) is equidistant from Þ – x2 + 5x – 4 = 0 Þ x2 – 5x + 4 = 0
the points A (– 2, k) and B (– 2k, – 3),
Þ x2 – 4x – x + 4 = 0
find k. Also find the length of AP.
Þ x (x – 4) – 1 (x – 4) = 0
Sol. Point P (2, 2) is equidistant from the points
A (– 2, k) and B (– 2k, – 3). Þ (x – 1) (x – 4) = 0
Þ Either x – 1 = 0 or x – 4 = 0 Þ x = 1, 4
\ AP = BP
31. Find the value of p for which the
Þ AP2 = BP2 quadratic equation
Þ (2 + 2)2 + (2 – k)2 = (2 + 2k)2 + (2 + 3)2 (2p + 1) x2 – (7p + 2) x + (7p – 3) = 0 has
Þ 16 + 4 + k2 – 4k = 4 + 4k2 + 8k + 25 equal roots. Also find these roots.
Þ – 3k2 – 12k – 9 = 0 Sol. Given equation is
Þ – 3 (k2 + 4k + 3) = 0 (2p + 1) x2 – (7p + 2) x + (7p – 3) = 0 ...(i)
Þ k2 + 4k + 3 = 0
1
Þ k2 + 3k + k + 3 = 0 First we notice 2p + 1 ¹ 0 Þ p ¹ -
2
Þ k (k + 3) + 1 (k + 3) =0 2
On comparing given eqn. with ax + bx + c = 0
Þ (k + 3) (k + 1) = 0
We have a = 2p + 1 ; b = – (7p + 2) and
Þ Either k + 3 = 0 or k + 1 = 0 c = 7p – 3
Þ k = – 3 or – 1 \ Discriminant D = b2 – 4ac
When k = – 1, then point A is (– 2, – 1) \ D = [– (7p + 2)]2 – 4 (2p + 1) (7p – 3)
https://www.arundeepselfhelp.info/index.php?route=product/product&path=180&product_id=395

Arundeep’s Solved Papers 32 Mathematics 2014 (Outside Delhi)


= (7p + 2)2 – 4 (14p2 – 6p + 7p – 3) = 49p2 + 4 + 28p – 4 (14p2 + p – 3)
= 49p2 + 4 + 28p – 56p2 – 4p + 12 = – 7p2 + 24p + 16
For equal roots, D = 0 Þ – 7p2 + 24p + 16 = 0
Þ 7p2 – 24p – 16 = 0 Þ 7p2 – 28p + 4p – 16 = 0
Þ 7p (p – 4) + 4 (p – 4) = 0 Þ (7p + 4) (p – 4) = 0
4
Þ Either 7p + 4 = 0 or p – 4 = 0 Þ p = - or 4
7

4
When p = - , then from eqn. (i), we get
7

æ 8 ö 2 æ 28 ö æ 28 ö
ç - + 1÷ x - ç - + 2 ÷ x + ç - - 3÷ = 0
è 7 ø è 7 ø è 7 ø
Þ – x2 + 14x – 49 = 0 Þ x2 – 14x + 49 = 0 Þ (x – 7)2 = 0 Þ x = 7, 7
When p = 4, then from (i), we gt
(8 + 1) x2 – (28 + 2) x + (28 – 3) = 0
5 5
Þ 9x2 – 30x + 25 = 0 Þ (3x – 5)2 = 0 Þx= ,
3 3

5 5
\ Roots are 7, 7 or , .
3 3
32. Cards numbered from 11 to 60 are kept in a box. If a card is drawn at random from the
box, find the probability that the number on the drawn card is :
(i) an odd number
(ii) a perfect square number
(iii) divisible by 5
(iv) a prime number less than 20
Sol. Cards numbered 11 to 60 are kept in a box. Total cards are 50.
\ Total no. of outcomes of drawing a card = 50.
(i) Favourable outcomes for odd number = 25, (i.e., 11, 13, 15, ...., 59).
25 1
\ Probability of drawing an odd numbered card = = .
50 2
(ii) Favourable outcomes for a perfect square number = 4 (i.e. 16, 25, 36, 49).
4 2
\ Probability of drawing a perfect square numbered card = = .
50 25
(iii) Favourable outcomes for a card number divisible by 5 = 10, (i.e., 15, 20, 25, 30, .... 60).
10 1
\ Probability of drawing a card numbered divisible by 5 = = .
50 5
(iv) Favourable outcomes for a prime number less than 20 = 4 (i.e. 11, 13, 17, 19).
https://www.arundeepselfhelp.info/index.php?route=product/product&path=180&product_id=395

Arundeep’s Solved Papers 33 Mathematics 2014 (Outside Delhi)

4 2
\ Probability of drawing a prime number less than 20 = = .
50 25
5
34. The difference of two natural numbers is 5 and the difference of their reciprocals is .
14
Find the numbers.
Sol. Let two natural numbers be x and y, where x > y. Hence x – y = 5 ...(i)
1 1
If x > y, then >
y x

1 1 5
Hence, - =
y x 14

1 1 5
Þ - = [from (i)]
y y + 5 14

y+5- y 5 5 5
Þ = Þ = Þ y2 + 5y = 14
y ( y + 5) 14 y2 + 5y 14
Þ y2 + 5y – 14 = 0 Þ y2 + 7y – 2y – 14 = 0 Þ y (y + 7) – 2 (y + 7) = 0
Þ (y – 2) (y + 7) = 0
Þ y = 2, y = – 7 (rejected) as – 7 is not a natural number.
\ y=2
From (i), x – 2 = 5 Þ x = 7
\ Required numbers are 7 and 2.
https://www.arundeepselfhelp.info/index.php?route=product/product&path=180&product_id=395

MATHEMATICS 2014 TERM II ( DELHI)


SET I
Time allowed : 3 hours Maximum marks : 90

General Instructions :
(i) All questions are compulsory.
(ii) The question paper consists of 34 questions divided into four sections – A, B, C and D.
(iii) Section A contains 8 questions of 1 mark each, which are multiple choice type questions, Section
B contains 6 questions of 2 marks each, Section C contains 10 questions of 3 marks each and
Section D contains 10 questions of 4 marks each.
(iv) Use of calculators is not permitted.
Section – A
Question numbers 1 to 8 carry 1 mark each. For each of these questions four alternative choices have
been provided of which only one is correct. Select the correct choice.

1. If k, 2k – 1 and 2k + 1 are three Þ 2x + 2y = 180º


consecutive terms of an A.P., the value Þ x + y = 90º
of k is Þ ÐAPB = 90º
(a) 2 (b) 3
(c) – 3 (d) 5
Sol. (b) If k, 2k – 1, 2k + 1 are three consecutive
terms of an A.P., then
(2k – 1) – k = (2k + 1) – (2k – 1)
Þ 2k – 1 – k = 2 Þ k – 1 = 2 Þ k = 3
[if a, b, c are in A.P. then b – a = c – b]
2. Two circles touch each other externally
at P. AB is a common tangent to the 3. In a right triangle ABC, right-angled at
circles touching them at A and B. The
B, BC = 12 cm and AB = 5 cm. The
value of ÐAPB is radius of the circle inscribed in the
(a) 30º (b) 45º triangle (in cm) is
(c) 60º (d) 90º (a) 4 (b) 3
Sol. (d) We have, AT = TP and TB = TP (c) 2 (d) 1
[Lengths of the tangents from ext. point T Sol. (c) In DABC,
to the circles]
AC = AB2 + BC2 = 52 + 122
Þ ÐTAP = ÐTPA = x (say)
and ÐTBP = ÐTPB = y (say) = 25 + 144 = 169 = 13 cm
[equal sides have equal angle opposite to it] Now, ar (ABC) = ar (AOB) + ar (BOC)
Also, in triangle APB, x + x + y + y = 180º + ar (COA)

Arundeep’s Solved Papers 34 Mathematics 2014 (Delhi)


https://www.arundeepselfhelp.info/index.php?route=product/product&path=180&product_id=395

Arundeep’s Solved Papers 35 Mathematics 2014 (Delhi)

1 1 1 1 BGG, GBG, GGB, BBG, BGB, GBB, BBB,


Þ ´ 5 ´ 12 = ´ 5 ´ r + ´ 12 ´ r + ´ 13 ´ r i.e. 7.
2 2 2 2
\ Required probability of having at least one
1
[_ area of triangle = × base × altitude] 7
2 boy = .
8
Þ 60 = r (5 + 12 + 13)
Þ 30r = 60 Þ r = 2 5. The angle of depression of a car parked
on the road from the top of a 150 m high
tower is 30º. The distance of the car from
the tower (in metres) is
(a) 50 3 (b) 150 3

(c) 150 2 (d) 75


Sol. (b) Let AB be the tower of height 150 m.
C is car and angle of depression is 30º.
Let the required distance of the car from
Alternate method : the tower be x m.
BP = OR = r, BR = OP = r
[BPOR is rectangle as all angles are 90º]
\ PC = 12 – r = CQ and AR = 5 – r = AQ

Also, AC = 52 + 122

= 25 + 144 = 169 = 13
Þ AQ + QC = 13 Therefore, ÐACB = 30º (alternate angle)
Þ 5 – r + 12 – r = 13 In right-angled triangle ABC, we have
Þ 2r = 4 Þ r = 2
BC
4. In a family of 3 children, the probability = cot 30º
AB
of having at least one boy is
x
7 1 Þ = 3
(a) (b) 150
8 8
Þ x = 150 3 m.
5 3
(c) (d) Thus the required distance of the car from
8 4
the tower is 150 3 m.
Sol. (a) In a family of three children,
Total possibilities are : BBB, BGB, BBG, 6. The probability that a number selected
GBB, GGB, GBG, BGG, GGG, therefore, at random from the numbers 1, 2, 3, ...,
total no. of possible outcomes = 8 15 is a multiple of 4, is
Favourable outcomes for at least one boy 4 2
are : (a) (b)
15 15
https://www.arundeepselfhelp.info/index.php?route=product/product&path=180&product_id=395

Arundeep’s Solved Papers 36 Mathematics 2014 (Delhi)

1 1 \ AB = OA 2 + OB2 = (10) 2 + (10) 2


(c) (d)
5 3
Sol. (c) Total no. of possible outcomes = 15
= 100 + 100 = 100 ´ 2
(Q 15 numbers are given) = 10 2 cm.
Favourable outcomes for a multiple of 4
= 3 (i.e. 4, 8, 12)
\ Probability of selecting a number which is
3 1
a multiple of 4 = =
15 5
7. ABCD is a rectangle whose three Section B
vertices are B (4, 0), C (4, 3) and
Question numbers 9 to 14 carry 2 marks each.
D (0, 3). The length of one of its
diagonals is 9. Find the values of p for which the
(a) 5 (b) 4 quadratic equation 4x2 + px + 3 = 0 has
equal roots.
(c) 3 (d) 25
Sol. Given quadratic equation be 4x2 + px + 3 = 0
Sol. (a) Both the diagonals are equal
On comparing with ax2 + bx + c = 0,
\ Length of diagonal BD
We have a = 4 ; b = p ; c = 3
= (4 - 0) 2 + (0 - 3)2 Since given quadratic eqn. has equal roots,
therefore, D = 0 Þ b2 – 4ac = 0
= 16 + 9 = 25 = 5 Þ (p)2 – 4 × 4 × 3 = 0
Þ p2 = 48
Þ p = ± 48 = ± 4 3
10. Find the number of natural numbers
between 101 and 999 which are divisible
by both 2 and 5.
Sol. Numbers between 101 and 999 which are
divisible by both 2 and 5 (i.e. by L.C.M. of
2 and 5 i.e. 10) are 110, 120, 130, ... 990.
8. A chord of a circle of radius 10 cm
subtends a right angle at its centre. The This forms an A.P. with a = 110, d = 10
length of the chord (in cm) is and an = 990
Now, an = a + (n – 1) d
(a) 5 2 (b) 10 2
Þ 990 = 110 + (n – 1) 10
5 Þ 880 = (n – 1) 10
(c) (d) 10 3
2 Þ 88 = n – 1
Sol. (b) AB is a chord. ÐAOB = 90º, Þ n = 89
AO = OB = 10 cm \ There are 89 natural numbers which are
\ Triangle AOB is right angled at O. divisible both by 2 and 5.
Using pythagoras theorem, we have
https://www.arundeepselfhelp.info/index.php?route=product/product&path=180&product_id=395

Arundeep’s Solved Papers 37 Mathematics 2014 (Delhi)


11. In given below common tangents AB and Þ BD = CD
CD to the two circles with centres O1 [using (i), BF = BD and CD = CE]
and O2 intersect at E. Prove that
AB = CD.

13. Two different dice are tossed together.


Find the probability
Sol. In the given figure, AB and CD are common (i) that the number on each die is even.
tangents to the two given circles with (ii) that the sum of numbers appearing on
centres O1 and O2. the two dice is 5.
We know that the lengths of the tangents Sol. Two different dice are tossed. Therefore,
drawn from a point outside the circle to total outcomes are 36.
the circle are equal in length. Here simple space S = {(1, 1), (1, 2),
(1, 3), (1, 4), (1, 5), (1, 6), (2, 1), (2, 2),
(2, 3), (2, 4), (2, 5), (2, 6), (3, 1), (3, 2),
(3, 3), (3, 4), (3, 5), (3, 6), (4, 1), (4, 2),
(4, 3), (4, 4), (4, 5), (4, 6), (5, 1), (5, 2),
(5, 3), (5, 4), (5, 5), (5, 6), (6, 1), (6, 2),
(6, 3), (6, 4), (6, 5), (6, 6)}
(i) Favourable outcomes for even number on
both dice = 9, {(2, 2) (2, 4), (2, 6), (4, 2),
\ AE = EC and EB = ED
(4, 4), (4, 6), (6, 2), (6, 4), (6, 6)}
Þ AE + EB = CE + ED \ Probability of getting even number on both
Þ AB = CD.
9 1
12. The incircle of an isosceles triangle ABC, dice = =
36 4
in which AB = AC, touches the sides BC,
CA and AB at D, E and F respectively. (ii) Favourable outcomes that the sum of the
Prove that BD = DC. numbers appearing in two dice is 5 are
(1, 4), (2, 3), (3, 2), (4, 1), i.e. 4.
Sol. We know that the lengths of the tangents
drawn from a point outside the circle to \ Probability of getting sum of numbers
the circle are equal in length. 4 1
appearing on two dice is 5 = = .
\ AF = AE, BF = BD 36 9
and CD = CE ...(i) 14. If the total surface area of a solid
Given AB = AC hemisphere is 462 cm2, find its volume.
Þ AF + FB = AE + EC é 22 ù
êë Take p = 7 úû
Þ FB = EC [using (i), AF= AE]
https://www.arundeepselfhelp.info/index.php?route=product/product&path=180&product_id=395

Arundeep’s Solved Papers 38 Mathematics 2014 (Delhi)


Sol. Let r cm be the radius of hemisphere Þ a + 4d + a + 8d = 30
Then total surface area of a solid hemisphere Þ 2a + 12d = 30
= 2pr2 + pr2 = 3pr2
Þ a + 6d = 15 ...(i)
Now, 3pr2 = 462 (given)
Also, a25 = 3a8
462 ´ 7 Þ a + 24d = 3 (a + 7d)
Þ r2 = = 49 = 72 Þ r = 7
3 ´ 22 Þ a + 24d = 3a + 21d
Þ 3d = 2a ...(ii)
From (i) and (ii), we have
a + 4a = 15 Þ 5a = 15 Þ a = 3
\ From (ii), we get, 3d = 2 × 3 Þ d = 2
\ a = 3, d = 2
2
\ Required volume of hemisphere = pr 3 Hence required A.P. is a, a + d, a + 2d,
3 a + 3d, .... i.e. 3, 3 + 2, 3 + 4, 3 + 6, .....
2 22 2 i.e. 3, 5, 7, 9, ....
= ´ ´ (7)3 = × 22 × 49 cm3
3 7 3 17. Construct a triangle with sides 5 cm,
= 718.67 cm 3 5.5 cm and 6.5 cm. Now construct
3
Section – C another triangle, whose sides are
5
Question numbers 15 to 24 carry 3 marks each. times the corresponding sides of the
15. Solve for x : given triangle.
16 15 Sol.
-1= ; x ¹ 0, – 1
x x +1

16 15
Sol. Given equation be, -1 = , where
x x +1
x ¹ 0, – 1

16 - x 15
Þ =
x x +1
Þ (16 – x) (x + 1) = 15x
Þ 16x + 16 – x2 – x = 15x
Þ x2 = 16 = (± 4)2
Þ x=±4
16. The sum of the 5th and the 9th terms
of an AP is 30. If its 25th term is three
times its 8th term, find the AP.
Steps of construction :
Sol. Let a be the first term and d be the common
difference of a given A.P. 1. A triangle with sides BC = 6.5 cm, AB = 5
cm and AC = 5.5 cm is constructed.
Given, a5 + a9 = 30
2. ÐCBX is drawn below BC.
https://www.arundeepselfhelp.info/index.php?route=product/product&path=180&product_id=395

Arundeep’s Solved Papers 39 Mathematics 2014 (Delhi)


3. On BX, A1, A2 , ..., A5 are marked, such Þ OM = 3000 3 ´ 3 = 9000 m ...(ii)
that BA1 = A1A2 = ... = A4A5.
\ AB = LM = OM – OL
4. A5 and C are joined.
= (9000 – 3000) m = 6000 m
5. C¢A3 is drawn parallel to A5C which meets
[from (i) and (ii)]
BC at C¢.
Now in 30 s, distance covered = 6000 m
6. A¢C¢ is drawn parallel to AC meeting AB at
\ In 1 hour (3600 s), distance covered by
A¢.
7. DA¢BC¢ is the required triangle. plane =
6000 3600
´ km
18. The angle of elevation of an aeroplane 30 1000
from a point on the ground is 60º. After = 720 km
a flight of 30 seconds the angle of \ Required speed of the aeroplane = 720 km/h.
elevation becomes 30º. If the aeroplane 19. If the point P (k – 1, 2) is equidistant
is flying at a constant height of from the points A (3, k) and B (k, 5),
3000 3 m, find the speed of the find the values of k.
aeroplane. Sol. Point P (k – 1, 2) is equidistant from the
Sol. From the point of observation (O), points A (3, k) and B (k, 5).
\ AP = BP
plane is at A, AL = 3000 3 m and Þ AP2 = BP2
ÐAOL = 60º. Þ (k – 1 – 3)2 + (2 – k)2 = (k – 1 – k)2
After 30 seconds, plane is at B, therefore, + (2 – 5)2
BM = 3000 3 m and ÐBOM = 30º. Þ (k – 4) + (2 – k) = (– 1) + (– 3)2
2 2 2

Þ k2 – 8k + 16 + 4 – 4k + k2 = 1 + 9
Distance AB is covered in 30 seconds.
Þ 2k2 – 12k + 10 = 0
In right-angled triangle OLA, we have
Þ k2 – 6k + 5 = 0
OL Þ k2 – 5k – k + 5 = 0
= cot 60º Þ OL = AL cot 60° Þ k (k – 5) – 1 (k – 5) = 0
AL
Þ (k – 1) (k – 5) = 0
Þ Either k – 1 = 0 or k – 5 = 0
Þ k = 1 or 5
20. Find the ratio in which the line segment
joining the points A (3, – 3) and B (– 2, 7)
is divided by x-axis. Also find the
coordinates of the point of division.
Sol. Let point P (x, 0) on x-axis divides the join
of A (3, – 3) and B (– 2, 7) in the ratio k : 1

1
Þ OL = 3000 3 ´ = 3000 m ...(i)
3
Then by section formula, the coordinates
In right-angled triangle OMB, we have of P are
OM
= cot 30º Þ OM = BM cot 30° æ -2k + 3 7k - 3 ö
BM ç , ÷ ...(i)
è k +1 k +1 ø
https://www.arundeepselfhelp.info/index.php?route=product/product&path=180&product_id=395

Arundeep’s Solved Papers 40 Mathematics 2014 (Delhi)


If point P lies on x-axis, then y = 0 5 22
= ´ ´ (21)2 ´ 3cm 2 = 3465 cm2.
7k - 3 3 6 7
Þ = 0 Þ 7k – 3 = 0 Þ k =
k +1 7 22. The largest possible sphere is carved out
of a wooden solid cube of side 7 cm. Find
3 the volume of the wood left.
\ Required ratio is :1, i.e. 3 : 7.
7
é 22 ù
3 êë Use p = 7 úû
Substituting k = in (i), we get the
7 Sol. As largest possible sphere is carved out of
coordinates of point of division are ; a solid wooden cube of side equal to side
of cube i.e. 7 cm, so, diameter of the sphere
æ 6 ö
ç- +3 ÷ is eqnal to side of cube i.e. 7 cm.
Pç 7 , 0 ÷ , i.e. P æç 3 , 0 ö÷ .
çç 3 + 1 ÷÷ è2 ø
è 7 ø
21. In Figure given below two concentric
circles with centre O, have radii 21 cm
and 42 cm. If ÐAOB = 60º, find the area
é 22 ù
of the shaded region. ê Use p = ú
ë 7û

\ Volume of the wood left = Volume of the


cube – Volume of the sphere
é 4 æ7ö ù
3
= ê(7) - p ç ÷ ú cm3
3
ë 3 è2ø û
[_ volume of cube = a3
4p 3
Sol. Given, two concentric circles with centre O and volume of sphere = r ]
and ÐAOB = ÐCOD = 60º = q (say)
3
OC (r1) = 21 cm, OA (r2) = 42 cm é 4 22 1 ù
= (7)3 ê1 - ´ ´
\ Area of shaded region ë 3 7 8 úû
(360º - q) = 343 [1 – 0.52] = 343 × 0.48 cm3
= ´ p (r22 - r12 )
360º = 164.64 cm3 .
q 23. Water in a canal, 6 m wide and 1.5 m
[_ area of sector of circle = pr2 × ] deep, is flowing at a speed of 4 km/h.
360°
How much area will it irrigate in 10
(360º - 60º ) 22 minutes, if 8 cm of standing water is
= ´ (42 2 - 212 ) needed for irrigation ?
360º 7
Sol. Given ; width of canal = 6 m,
300° 22
= ´ ´ (21) 2 (4 - 1) and depth of canal = 1.5 m and in 1 hour,
360° 7
https://www.arundeepselfhelp.info/index.php?route=product/product&path=180&product_id=395

Arundeep’s Solved Papers 41 Mathematics 2014 (Delhi)


length of water flows out by canal = 4 km Sol. Area of trapezium = 24.5 sq. cm
\ Volume of water flows out in 1 hour AD = 10 cm, BC = 4 cm
= 6 × 1.5 × 4000 m3 = 36000 m3 Let AB = h cm
\ Volume of water flows out in 10 minutes
1
36000 \ Area of trapezium = (AD + BC) × AB
= ´ 10 = 6000 m3 ...(i)
2
60
1
Þ 24.5 = (10 + 4) ´ h
2
24.5
Þ h = = 3.5 cm
7

Suppose this water irrigates x m2 of area


and we require 8 cm of standing water.
8
\ Volume of water required = x ´ m3 Now, area of quadrant ABE
100
90º
From (i) and (ii) we get = ´ p (3.5) 2 sq. cm
360º
8
x´ = 6000 1 22
100 = ´ ´ (3.5)2 sq. cm
4 7
6000 ´ 100 = 9.625 sq. cm ...(ii)
Þ x = = 75000 m2
8 \ Area of shaded region
\ 75000 m2 of area is irrigated. = area of trap – area of quadrantABE
24. In Figure given below ABCD is a = (24.5 – 9.625) sq. cm
trapezium of area 24.5 sq. cm. In it, = 14.875 sq. cm [from (i), (ii)]
AD || BC, ÐDAB = 90º, AD = 10 cm and
Section – D
BC = 4 cm. If ABE in a quadrant of a
circle, find the area of the shaded Question numbers 25 to 34 carry 4 marks each.
é 22 ù 25. Solve for x :
region. êë Take p = 7 úû
x - 2 x - 4 10
+ = ; x ¹ 3, 5
x-3 x-5 3
Sol. Given equation be,
x - 2 x - 4 10
+ = , x ¹ 3, 5
x-3 x-5 3
( x - 3) + 1 ( x - 5) + 1 10
Þ + =
x-3 x-5 3
https://www.arundeepselfhelp.info/index.php?route=product/product&path=180&product_id=395

Arundeep’s Solved Papers 42 Mathematics 2014 (Delhi)

1 1 10 = 6 × 52 = 312
Þ 1+ +1+ =
x-3 x-5 3 Value promted :
1 1 10 Students are concerned about safe and
Þ + = -2
x-3 x-5 3 pollution free environment.

x-5+ x-3 27. The angle of elevation of the top of a


4
Þ = tower at a distance of 120 m from a point
( x - 3) ( x - 5) 3
A on the ground is 45º. If the angle of
2x - 8 4 elevation of the top of a flagstaff fixed
Þ =
x2 - 8 x + 15 3 at the top of the tower, at A is 60º, then
Þ 2
4x – 32x + 60 = 6x – 24 find the height of the flagstaff.
Þ 4x2 – 38x + 84 = 0 [Use 3 = 1.73]
Þ 2x2 – 19x + 42 = 0
Þ 2x2 – 12x – 7x + 42 = 0 Sol. Let BC be the tower and BD is flagstaff of
Þ 2x (x – 6) – 7 (x – 6) = 0 height h m.
Þ (2x – 7) (x – 6) = 0
Þ Either 2x – 7 = 0 or x – 6 = 0
7
Þ x = , 6.
2
26. In a school, students decided to plant
trees in and around the school to reduce
air pollution. It was decided that the
number of trees, that each section of
each class will plant, will be double of the
class in which they are studying. If there
are 1 to 12 classes in the school and each
class has two sections, find how many
trees were planted by the students.
Which value is shown in this question ? Let BC = x m.
Sol. According to question, each section of : AC = 120 m, ÐBAC = 45º
Class 1 will plant 2 trees, class II will plant
and ÐDAC = 60º
4 trees, class III will plant 6 trees and so
on.. class 12 will plant 24 trees and each In right-angled triangle ACB, we have
class has 2 sections.
AC 120
\ Number of trees planted = cot 45º Þ =1
BC x
= 4 + 8 + 12 + ... + 48
This forms an A.P. with a = 4, d = 8 – 4 = 4 Þ x = 120 ...(i)
and n = 12 In right angled triangle ACD, we have
\ Required number of trees planted
CD h+ x
é n ù = tan 60º Þ = 3
êQ Sn = 2 ( a + l ) ú
12 AC 120
S12 = (4 + 48)
2 ë û
https://www.arundeepselfhelp.info/index.php?route=product/product&path=180&product_id=395

Arundeep’s Solved Papers 43 Mathematics 2014 (Delhi)


Þ h + x = 120 3 Þ h = 120 3 - 120 [using (i), x = 120]

Þ h = 120[ 3 - 1] = 120 [1.73 – 1] m = 120 × 0.73 = 87.6 m


\ Required height of the flagstaff is 87.6 m.
28. Red queens and black jacks are removed from a pack of 52 playing cards. A card is drawn
at random from the remaining cards, after reshuffling them. Find the probability that
the drawn card is
(i) a king (ii) of red colour (iii) a face card (iv) a queen
Sol. Red queens and black jacks, i.e. 2 + 2 = 4 cards are removed from a pack of 52 playing cards.
Remaining cards are 52 – 4 = 48.
\ Possible number of outcomes of drawing one card from 48 cards is 48.
(i) Favourable outcomes for drawing a king are 4.
4 1
\ Probability of drawing a king = =
48 12
(ii) Favourable outcomes for a card of red colour are 24 as 2 red queens have already been removed
and in total there are 24 red cards.
24 1
\ Probability of drawing a red card = = .
48 2
(iii) Favourable outcomes for a face card (4 kings, 2 queens, 2 jacks) = 8
8 1
\ Probability of drawing a face card = =
48 6
(iv) Favourable outcomes for drawing a queen are 2, as 2 red queens have been removed.
2 1
\ Probability of drawing a queen = = .
48 24
29. If A (– 3, 5), B (– 2, – 7), C (1, – 8) and D (6, 3) are the vertices of a quadrilateral ABCD,
find its area.
Sol. Area of quadrilateral ABCD
= Area of triangle ABC + Area of triangle ACD ...(i)
Since we know that if A(x1, y1), B(x2, y2) and C(x3, y3) are vertices of DABC
1
Then area of DABC = |x (y – y ) + x2 (y3 – y1) + x3 (y1 – y2)|
2 1 2 3
1
Now, ar (ABC) = [– 3 (– 7 + 8) – 2 (– 8 – 5) + 1 (5 + 7)]
2
1
= [– 3 + 26 + 12]
2
35
= sq. units ...(ii)
2
https://www.arundeepselfhelp.info/index.php?route=product/product&path=180&product_id=395

Arundeep’s Solved Papers 44 Mathematics 2014 (Delhi)


Also, ar (ACD)
1
= [– 3 (– 8 – 3) + 1 (3 – 5) + 6 (5 + 8)
2
1
= [33 – 2 + 78]
2
109
= sq. units ...(iii)
2
from (i), (ii), (iii), we get

35 109 144
ar (ABCD) = + = = 72 sq. units
2 2 2
30. A motorboat whose speed in still water is 18 km/h, takes 1 hour more to go 24 km
upstream than to return downstream to the same spot. Find the speed of the stream.
Sol. Speed of motor boat in still water = 18 km/h
Let speed of the stream = x km/h
\ Speed upstream = (18 – x) km/h
Speed downstream = (18 + x) km/h
24
Thus, time taken by motor boat to covered a distance of 24 km stream = hr
18 - x
24
and Time taken by motor boat to covered a distance of 24 km downstream = km
18 + x
According to question, we have
24 24 24 (18 + x) - 24 (18 - x )
Þ - =1 Þ =1
18 - x 18 + x (18 - x) (18 + x)
432 + 24 x - 432 + 24 x
Þ =1 Þ 48x = 324 – x2
324 - x 2
Þ x2 + 48x – 324 = 0 Þ x2 + 54x – 6x – 324 = 0
Þ x (x + 54) – 6 (x + 54) = 0 Þ (x – 6) (x + 54) = 0
Þ Either x – 6 = 0 or x + 54 = 0 Þ x = 6 or x = – 54 (rejected)
\ Required speed of the stream is 6 km/h.
31. In Figure given below PQ is a chord of length 16 cm, of
a circle of radius 10 cm. The tangents at P and Q intersect
at a point T. Find the length of TP.
Sol. Given : PQ is chord of length 16 cm, TP and TQ are the
tangents to a circle with centre O, radius 10 cm.
To find : TP.
Solution : Join OP and OQ.
In triangles OTP and OTQ, we have
https://www.arundeepselfhelp.info/index.php?route=product/product&path=180&product_id=395

Arundeep’s Solved Papers 45 Mathematics 2014 (Delhi)


2
æ 32 ö
ç ÷ = TP2 – 64
è 3ø

1024
Þ TP2 = + 64
9

2
1024 + 576 1600 æ 40 ö
OT is common = = = ç ÷
9 9 è 3 ø
OP = OQ (radii)
TP = TQ [length of the tangents drawn 40
from a point outside the circle to the circle \ TP = cm
3
are equal]
32. Prove that the tangent at any point of a
\ DOPT @ DOQT circle is perpendicular to the radius
(SSS axiom of congruency) through the point of contact.
\ ÐPOT = ÐQOT (by cpct) ...(i) Sol. Given : A circle with centre O, line l is
Consider, triangles OPR and OQR, we have tangent to the circle at A.
OP = OQ (radii) To Prove : Radius OA is perpendicular to
the tangent at A.
OR is common
Construct : Take a point P, other then A, on
ÐPOR = ÐQOR [from (i)]
tangent l. Join OP, meeting the circle at R.
\ DOPR @ DOQR Proof : We know that tangent to the circle
(SAS axiom of congruency) touches, the circle at one point and all other
points on the tangent lie in the exterior of a
1
\ PR = RQ = ´ 16 = 8 cm ...(ii) circle.
2
ÐORP = ÐORQ = 90º ...(iii)
In right angled triangle TRP, we have
TR2 = TP2 – (8)2 = TP2 – 64
Also OT2 = TP2 + (10)2
Þ (TR + 6)2 = TP2 + 100

[Q OR = 100 - 64 = 6]
Þ TR2+ 12TR + 36 = TP2 + 100 \ OP > OR (radius of circle)
Þ TP2 – 64 + 12TR + 36 = TP2 + 100 Þ OP > OA (Q OR = OA, radius of circle)
Þ 12TR = 128
Þ OA < OP
32 Þ OA is the smallest segment, from O to a
Þ TR = cm
point on the tangent.
3
From (iv) ; we have We know that smallest line segment from a
point outside the circle to the line is
perpendicular segment.
https://www.arundeepselfhelp.info/index.php?route=product/product&path=180&product_id=395

Arundeep’s Solved Papers 46 Mathematics 2014 (Delhi)


34. A container open at the top, is in the
Hence, OA is ^ to tangent l.
form of a frustum of a cone of height
Þ tangent at any point of a circle is 24 cm with radii of its lower and upper
perpendicular to the radius through the point circular ends as 8 cm and 20 cm
of contact. respectively. Find the cost of milk which
33. 150 spherical marbles, each of diameter can completely fill the container at the
1.4 cm, are dropped in a cylindrical
é 22 ù
vessel of diameter 7 cm containing some rate of Rs. 21 per litre. ê Use p = ú
water, which are completely immersed ë 7û
in water. Find the rise in the level of Sol. Given radius of lower end (r1) = 8 cm
water in the vessel.
and Radius of upper end (r2) = 20 cm
Sol. Let rise in water in cylinder be h cm when
150 spherical marbles, each of diameter Height of frustum = 24 cm
1.4 cm are dropped and fully immersed.
Then volume of 150 spherical marbles
= volume of water raised in cylinder

Volume of the container

ph 2
= [r + r22 + r1r2 ]
3 1

22 24
4 æ7ö
2 = ´ [(8) 2 + (20) 2 + 8 ´ 20]
Þ 150 ´ p (0.7)3 = p ç ÷ h 7 3
3 è2ø
22
4 7´7´7 7´7 = ´ 8 [64 + 400 + 160]
Þ 150 ´ ´ = ´h 7
3 1000 4
22
4 ´ 4 ´ 7 28 = ´ 8 ´ 624 cm3 = 15689.14 cm3
Þ h = = cm = 5.6 cm 7
20 5
\ Required rise in water level by 5.6 cm. = 15.68914 L

1
[Q 1L = 1000 cm3 Þ 1 cm3 = L]
1000
\ Cost of milk which can completely fill the
container = Rs. 21 × 15.68914
= Rs. 329.47.
https://www.arundeepselfhelp.info/index.php?route=product/product&path=180&product_id=395

Arundeep’s Solved Papers 47 Mathematics 2014 (Delhi)


Set-II (Uncommon Questions to Set-I)
NOTE : Except for the following questions, all the
remaining questions have been asked in previous set.
14. Find the values of k for which the of radius 4 cm is drawn and with centre B,
quadratic equation 9x2 – 3kx + k = 0 has a circle of radius 3 cm is drawn.
equal roots. 2. With AB as diameter, a circle is drawn
Sol. Given equation be, 9x2 – 3kx + k = 0 meeting circle with centre A at S and T and
on comparing with ax 2 + bx + c = 0 circle with centre B at P and Q.
where a ¹ 0 3. Then AP and AQ are tangents from A to
We have a = 9 ; b = –3k ; c = k circle with centre B and BS and BT are
For equal roots, D = 0 tangents from B to circle with centre A.
\ b2 – 4ac = 0
Þ (– 3k)2 – 4 × 9 × k = 0
Þ 9k2 – 36k = 0 Þ 9k (k – 4) = 0
Þ Either k = 0 or k – 4 = 0 i.e. k = 0 or 4
22. The sum of the 2nd and the 7th terms
of an AP is 30. If its 15th term is 1 less
than twice its 8th term, find the AP.
Sol. Let a be the first term and d be the common
difference of a given A.P.
Given, a2 + a7 = 30
24. Prove that the diagonals of a rectangle
Þ a + d + a + 6d = 30 [_ an = 2a + (n – 1)d] ABCD, with vertices A (2, – 1), B (5, – 1),
Þ 2a + 7d = 30 ...(i) C (5, 6) and D (2, 6), are equal and bisect
Also, a15 = 2a8 – 1 each other.
Þ a + 14d = 2 (a + 7d) – 1 Sol. Given : A (2, – 1), B (5, – 1), C (5, 6) and
Þ a + 14d = 2a + 14d – 1 Þ a = 1 D (2, 6) are the vertices of a rectangle
Substituting the value of a in (i), we get ABCD.
2 + 7d = 30 Þ 7d = 28 Þ d = 4 \ AC = (5 - 2) 2 + (6 + 1) 2
\ a = 1, d = 4
= 9 + 49 = 58
Hence, required A.P. is a, a + d, a + 2d,
a + 3d, ..... BD = (5 - 2) 2 + ( -1 - 6)2
i.e. 1, 1 + 4, 1 + 2, 1 + 12, .....
i.e. 1, 5, 9, 13, ... = 9 + 49 = 58
23. Draw a line segment AB of length 8 cm. \ AC = BD, i.e. diagonals are equal
Taking A as centre, draw a circle of
radius 4 cm and taking B as centre, draw
another circle of radius 3 cm. Construct
tangents to each circle from the centre
of the other circle.
Sol. Steps of construction :
1. AB = 8 cm is taken. With centre A, a circle
https://www.arundeepselfhelp.info/index.php?route=product/product&path=180&product_id=395

Arundeep’s Solved Papers 48 Mathematics 2014 (Delhi)


Now, the coordinates of mid-point of AC 32. All the red face cards are removed from
a pack of 52 playing cards. A card is
æ 2 + 5 6 -1ö æ7 5ö
are ç , ÷ , i.e. ç , ÷ drawn at random from the remaining
è 2 2 ø è2 2ø cards, after reshuffling them. Find the
The coordinates of mid-point of BD are probability that the drawn card is
(i) of red colour
æ 5 + 2 -1 + 6 ö æ7 5ö
ç , ÷ , i.e. ç , ÷ (ii) a queen
è 2 2 ø è2 2ø
(iii) an ace
As the coordinates of the mid-points of AC (iv) a face card
and BD are same, hence diagonals bisect
Sol. Face cards are taken as (4 jacks, 4 queens
each other.
and 4 kings) :
31. Solve for x :
As all the red face cards are removed
x - 3 x - 5 10 \ 6 cards (2 jacks, 2 queens and 2 kings) are
+ = ; x ¹ 4, 6
x-4 x-6 3 removed from 52 cards.
\ remaining no. of cards = 52 – 6 = 46 cards.
x - 3 x - 5 10 \ Total no. of outcomes of drawing a card
Sol. Consider the equation + = ,
x-4 x-6 3 from 46 cards = 46.
where x ¹ 4, 6 (i) Favourable outcomes of drawing a red card
( x - 4) + 1 ( x - 6) + 1 10 = 20 (as 6 red cards from 26 have been
Þ + = removed).
x-4 x-6 3
\ Probability of drawing a red card
1 1 10
Þ 1+ +1+ = =
20 10
=
x-4 x-6 3 46 23
1 1 10 (ii) Favourable outcomes of drawing a queen
Þ + = -2 = 2 (as 2 red queens have already been
x-4 x-6 3
removed out of 4).
x-6+ x-4 4
Þ ( x - 4) ( x - 6) = \ Probability of drawing a queen =
2
=
1
3 46 23
2 x - 10 4 (iii) Favourable outcomes of drawing an ace = 4
Þ =
x2 - 10 x + 24 3 4 2
\ Probability of drawing an ace = =
x-5 2 46 23
Þ = (iv) Favourable outcomes of drawing a face
x2 - 10 x + 24 3
card = 6 (as 2 jacks, 2 queens, 2 kings
Þ 2
2x – 20x + 48 = 3x – 15
have already been removed).
Þ 2x2 – 23x + 63 = 0 \ Required probability of drawing a face card
Þ 2x2 – 14x – 9x + 63 = 0
6 3
Þ 2x (x – 7) – 9 (x – 7) = 0 = = .
Þ (2x – 9) (x – 7) = 0 46 23
Þ Either 2x – 9 = 0 or x – 7 = 0 33. A (4, – 6), B (3, – 2) and C (5, 2) are the
vertices of a DABC and AD is its median.
9 Prove that the median AD divides DABC
i.e. x = or 7.
2 into two triangles of equal areas.
https://www.arundeepselfhelp.info/index.php?route=product/product&path=180&product_id=395

Arundeep’s Solved Papers 49 Mathematics 2014 (Delhi)


Sol. A (4, – 6), B (3, – 2) and C (5, 2) are the vertices of DABC.
Q AD is the median, so, D is the mid-point of BC.
\ Coordinates of D are by using mid point formula we have
æ 3 + 5 -2 + 2 ö
=ç , ÷ = (4, 0)
è 2 2 ø
Now, ar (ABD)
1
= [3(0 + 6) + 4 (- 6 + 2) + 4 ( - 2 - 0)]
2

1 1
= [18 - 15 - 8] = | - 6 | = 3 sq. units
2 2
[if A(x1 y1) ; B (x2, y2) and C(x3, y3) are the vertices of DABC.
1
Then area of DABC = |x (y – y3) + x2 (y3 – y1) + x3 (y1 – y2)|]
2 1 2

1
Thus, ar (ADC) = [4 (2 + 6) + 5 ( -6 - 0) + 4 (0 - 2)]
2

1 1
= [32 - 30 - 8] = | - 6 | = 3 sq. units
2 2
\ ar (ABD) = ar (ADC)
34. Prove that opposite sides of a quadrilateral circumscribing a circle subtend supplementary
angles at the centre of the circle.
Sol. Given : ABCD is a quadrilateral, circumscribing a circle with centre O and touches the quadrilateral
at P, Q, R and S.
To prove :
(i) ÐAOB + ÐCOD = 180º
(ii) ÐBOC + ÐAOD = 180º
Construction : Join OP, OQ, OR and OS.
Proof : Consider, triangles APO and ASO,
AP = AS [Lengths of the tangents drawn from a point outside the circle to the circle are equal]
OS = OP (radii)
OA is common
\ DAPO @ DASO [SSS axiom of congrency rule]
\ ÐOAP = ÐOAS = x (say) [c.p.c.t.]
Similarly, ÐOBP = ÐOBQ = y (say)
ÐOCQ = ÐOCR = z (say)
and ÐODR = ÐODS = w (say)
We have,
https://www.arundeepselfhelp.info/index.php?route=product/product&path=180&product_id=395

Arundeep’s Solved Papers 50 Mathematics 2014 (Delhi)


ÐDAB + ÐABC + ÐBCD + ÐCDA = 360º \ ÐAOB + ÐCOD = 180º
Þ 2x + 2y + 2z + 2w = 360º Again consider, ÐBOC + ÐAOD
Þ x + y + z + w = 180º ...(i) = [180º – y – z] + [180º – x – w]
Consider, ÐAOB + ÐCOD [Sum of angles of a triangle is 180º]
= [180º – x – y] + [180º – w – z]
= 360º – (x + y + z + w)
[Sum of angles of a triangle is 180º]
= 360º – 180º = 180º [using (i)]
= 360º – (x + y + z + w) = 360º – 180º
Hence proved.
[using (i)]
Set-III (Uncommon Questions to Set-I and Set-II)
NOTE : Except for the following questions, all the
remaining questions have been asked in previous set.

14. Find the value of p so that the quadratic


equation px (x – 3) + 9 = 0 has equal roots. Þ 5a + 15d = a + 16d Þ 4a = d ...(ii)
Sol. Consider the equation px (x –3) + 9 = 0 From (i) and (ii), we get
\ px2 – 3px + 9 = 0 ...(i) a + 12a = 26 Þ 13a = 26 Þ a = 2
2
on comparing eqn. (i) with ax + bx + c = 0 From (ii), we get d = 8
We have a = p ; b = –3p ; c = 9 \ a = 2 and d = 8
For equal roots, discriminant D = 0 \ Required A.P. is
\ b2 – 4ac = 0 a, a + d, a + 2d, .....
Þ (– 3p)2 – 4 × p × 9 = 0
2, 2 + 8, 2 + 16, .....
Þ 9p2 – 36p = 0 Þ 9p (p – 4) = 0
2, 10, 18, ....
Þ Either p = 0 or p – 4 = 0
23. From the top of a 60 m high building,
Þ p = 0 or p = 4
the angles of depression of the top and
But from (i), we notice p ¹ 0 \ p = 4 the bottom of a tower are 45º and 60º
22. The sum of the first seven terms of an respectively. Find the height of the
AP is 182. If its 4th and the 17th terms
tower. [Take 3 = 1.73]
are in the ratio 1 : 5, find the AP.
Sol. Let a be the first term and d be the common Sol. Let AB be 60 m high building and CD be
difference of a given A.P. the tower of height h. Angles of depression
from top of building, to the top and bottom
According to question, S7 = 182
of tower are 45º and 60º.
7
Þ [2a + (7 - 1) d ] = 182 \ ÐACE = 45º and
2
n ÐADB = 60º [using alternate angles]
[_ Sn = {2a + (n – 1) d}] Let BD = CE = x m and BE = CD = h m
2
Þ a + 3d = 26 ...(i) \ AE = (60 – h) m
a4 1 a + 3d 1 In right-angled triangle ABD, we have
Also, a = Þ =
17 5 a + 16d 5
BD
[_ an = a + (n – 1) d] = cot 60º
AB
https://www.arundeepselfhelp.info/index.php?route=product/product&path=180&product_id=395

Arundeep’s Solved Papers 51 Mathematics 2014 (Delhi)

1 60 31. Solve for x :


x
Þ = Þx = = 20 3 ...(i)
60 3 3 x - 4 x - 6 10
+ = ; x ¹ 5, 7
x-5 x-7 3
Sol. Consider the equation
x-4 x-6 10
+ = , where x ¹ 5, 7
x-5 x-7 3
( x - 5) + 1 ( x - 7) + 1 10
Þ + =
x-5 x-7 3
1 1 10
Þ 1+ +1+ =
x-5 x-7 3
1 1 10
Þ + = -2
x-5 x-7 3
x-7+ x-5 4
In right angled triangle AEC, we have Þ =
( x - 5) ( x - 7) 3
AE
= tan 45º
CE 2 x - 12 4
Þ =
x2 - 12 x + 35 3
60 - h
Þ =1 Þ 60 – h = x
x x-6 2
Þ =
Þ h = 60 – x x2 - 12 x + 35 3
Þ 2x2 – 24x + 70 = 3x – 18
Þ h = 60 - 20 3 = 20 [3 - 3]
Þ 2x2 – 27x + 88 = 0
= 20 [3 – 1.73] = 20 × 1.27 = 25.4 m Þ 2x2 – 11x – 16x + 88 = 0
[using (i)]
Þ x (2x – 11) – 8 (2x – 11) = 0
\ required height of the tower is 25.4 m.
Þ (x – 8) (2x – 11) = 0
24. Find a point P on the y-axis which is
Þ Either x – 8 = 0 or 2x – 11 = 0
equidistant from the points A (4, 8) and
B (– 6, 6). Also find the distance AP. 11
Þ x = 8,
Sol. Let point P (0, y) on y-axis is equidistant 2
from the points A (4, 8) and B (– 6, 6) 32. Five cards – the ten, jack, queen, king
\ AP = BP and ace of diamonds, are well shuffled
Þ AP2 = BP2 with their faces downwards. One card is
Þ (4 – 0)2 + (8 – y)2 = (– 6 – 0)2 + (6 – y)2 then picked up at random.
Þ 16 + 64 – 16y + y2 = 36 + 36 – 12y + y2 (a) What is the probability that the drawn
Þ – 4y = – 8 Þ y = 2 card is the queen ?
\ Coordinates of point P are (0, 2) (b) If the queen is drawn and put aside, and
a second card is drawn, find the
\ Required distance AP = (4 - 0)2 + (8 - 2)2 probability that the second card is (i) an
ace (ii) a queen.
= 16 + 36 = 52 = 2 13 units Sol. Five cards : a ten, jack, queen, king and
https://www.arundeepselfhelp.info/index.php?route=product/product&path=180&product_id=395

Arundeep’s Solved Papers 52 Mathematics 2014 (Delhi)


ace of diamonds are shuffled with their faces [if A(x, y) ; B (x2, y2) and C(x3 , y3) are
downwards. vertices of DABC
\ Total no. of possibilities of selecting one 1
card = 5 Then area of DABC = |x (y – y3) + x2
2 1 2
(a) Favourable outcomes for drawing a queen = 1.
(y3 – y1) + x3 (y1 – y2)]
\ Required probability of drawing a queen
Now, ar (DABD)
1
= 1
5 = [4 (6 - 5) + 7 (5 - 2) + 4 (2 - 6)]
2
(b) When queen is kept aside, then remaining
cards = 4. 1 9
So, total outcomes of drawing a card = 4 = [4 + 21 - 16] = sq. units
2 2
(i) Favourable outcomes of drawing an ace is 1.
ar (DADC)
1
\ Probability of drawing an ace = . 1
4 = [4 (5 - 4) + 4 (4 - 2) + 1(2 - 5)]
(ii) Favourable outcomes of drawing a queen = 0 2
(as there is no card of queen).
1 9
\ Required probability of drawing a queen = [4 + 8 - 3] = sq. units
2 2
0
= , i.e. 0. \ ar (ABD) = ar (ADC)
4
34. In Figure given below, a triangle ABC
33. If A (4, 2), B (7, 6) and C (1, 4) are the is drawn to circumscribe a circle of
vertices of a DABC and AD is its median, radius 4 cm, such that the segments BD
prove that the median AD divides DABC and DC are of lengths 8 cm and 6 cm
into two triangles of equal areas. respectively. Find the sides AB and AC.
Sol. Given ; A (4, 2), B (7, 6) and C (1, 4) are
the vertices of a triangle ABC and AD is the
median.
\ D is the mid-point of BC.
\ The coordinates of D are given by using
mid point formula
æ 7 +1 6 + 4ö
ç , ÷ , i.e. (4, 5) 4c
è 2 2 ø m
4 cm

Sol. We know that the lengths of the tangents


drawn from a point outside the circle to
the circle are equal in length.
\ AF = AE = x, (say)
BF = BD = 8 cm
CE = CD = 6 cm
https://www.arundeepselfhelp.info/index.php?route=product/product&path=180&product_id=395

Arundeep’s Solved Papers 53 Mathematics 2014 (Delhi)


\ AB = (x + 8) cm
AC = (x + 6) cm
and BC = 14 cm.
Now, 2S = x + 8 + x + 6 + 14 = 28 + 2x
Þ S = 14 + x
Thus by Heron’s formula, we have
\ Area of DABC
= (14 + x) (14 + x - x - 8) (14 + x - x - 6)(14 + x - 14)

= (14 + x) ´ 6 ´ 8 ´ x ...(i)
Also, the area of DABC = ar (BOC) + ar (BOA) + ar (AOC)
1 1 1
= ´ 14 ´ 4 + ´ (8 + x) ´ 4 + (6 + x) ´ 4
2 2 2
= 28 + 16 + 2x + 12 + 2x = 56 + 4x ...(ii)
From (i) and (ii), we get
48 x (14 + x ) = 56 + 4x
On squaring both sides, we get
48x (14 + x) = (56 + 4x)2
Þ 48x (14 + x) = 16 (14 + x)2
Þ 3x (14 + x) – (14 + x)2 = 0
Þ (14 + x) (3x – 14 – x) = 0
Þ (14 + x) (2x – 14) = 0
Þ 14 + x = 0 or 2x – 14 = 0
Þ x = – 14 (rejected) or x = 7
\ AB = (7 + 8) cm =15 cm.
and AC = (7 + 6) cm = 13 cm.
https://www.arundeepselfhelp.info/index.php?route=product/product&path=180&product_id=395

MATHEMATICS 2015 TERM I


SET I
Time allowed : 3 hours Maximum marks : 90

SECTION – A

1. In DDEW, AB || EW. If AD = 4 cm, DE = 12 cm and DW = 24 cm, then find the value of DB.
[1]
Sol. Let BD = x cm, D

m
DW = 24 cm,

4c
A B

24
cm
Then, BW = (24 – x) cm,

cm
12
AE = 12 – 4 = 8 cm
In DDEW, AB || EW
E W
AD BD
\ = [using Thales’ Theorem]
AE BW

4 x
Þ = Þ 8x = 96 – 4x Þ 12x = 96
8 24 - x

96
Þ x = =8
12
\ DB = 8 cm Ans.
2. In DABC is right angled at B, what is the value of sin (A + C). [1]
Sol. ÐB = 90º [Given]
We know that in DABC,
ÐA + ÐB + ÐC = 180º [_ Angle sum property of a D]
Þ ÐA + ÐC + 90º = 180º
Þ ÐA + ÐC = 180º – 90º = 90º
\ sin (A + C) = sin 90º = 1 Ans. A

C B

Arundeep’s Solved Papers 54 Mathematics 2015 (Term I)


https://www.arundeepselfhelp.info/index.php?route=product/product&path=180&product_id=395

Arundeep’s Solved Papers 55 Mathematics 2015 (Term I)

3 cos 2 q + 2 cos q
3. If 3 sin q = cos q, find the value of . [1]
3 cos q + 2

Sol. Given, 3 sin q = cos q

sin q 1 1
Þ = or tan q =
cos q 3 3
Þ tan q = tan 30º Þ q = 30º

3cos 2 q + 2 cos q = cos q (3 cos q + 2)


Now,
3 cos q + 2 (3 cos q + 2)

3
= cos q = cos 30° = [_ q = 30°]
2
4. From the following frequency distribution, find the median class :

Cost of
living 1400-1550 1550-1700 1700-1850 1850-2000
index
Number [1]
of 8 15 21 8
weeks

Sol. The table of values is given as under :

Cost of living No. of weeks


c.f.
index (f)
1400-1550 8 8
1550-1700 15 23
1700-1850 21 44
1850-2000 8 52
N = Sf = 52

Here, N = 52

N 52
Þ = = 26,
2 2
Since C.F. which is just greater than 26 be 44 and corresponding class interval 1700-1850.
\ Median class is 1700-1850. Ans.
https://www.arundeepselfhelp.info/index.php?route=product/product&path=180&product_id=395

Arundeep’s Solved Papers 56 Mathematics 2015 (Term I)


SECTION – B on comparing (1) and eqn. (2) with a1 x + b1
y + c1 = 0 and a2x + b2 y + c2 = 0, we have
5. Show that 3 7 is an irrational number.. a1 = 3 ; b1 = 2 ; c1 = –8
[2] a2 = 6 ; b2 = –4 ; c2 = –9
Sol. Let us assume, to the contrary, that 3 7 a1 3 1 b1 2 -1
be a rational number Here, a = 6 = 2 , b = - 4 = 2
2 2
That is, we can find co-prime a and
a1 b1
a \ ¹ , which will give a unique
b (b ¹ 0) such that 3 7 = a2 b2
b
solution.
a
Rearranging, we get 7= Hence, given pair of linear equations is
3b
consistent. Ans.
a 8. X and Y are points on the sides AB and
Since 3, a and b are integers, can be
3b AC respectively of a triangle ABC such
p AX 1
, so
a that = , AY = 2 cm and YC = 6 cm.
written in the form of is a AB 4
q 3b
Find whether XY || BC or not. [2]
rational number and so 7 is rational.
AX 1
But this contradicts the fact that Sol. Given, =
7 is AB 4
irrational number. i.e., AX = 1K, AB = 4K
Hence our supposition is wrong. (where K be constant)
So, we conclude that 3 7 is irrational. \ BX = AB – AX = 4K – 1K = 3K
Hence Proved. A
6. Explain why (17 × 5 × 11 × 3 × 2 + 2 × 11) 2 cm
is a composite number ? [2] X Y

Sol. 17 × 5 × 11 × 3 × 2 + 2 × 11 6 cm
= 17 × 5 × 3 × 22 + 22
= 22 (17 × 5 + 3 × 1)
B C
= 22 (255 + 1) = 2 × 11 × 256
\ Given expression is divisible by 2, 11 and
AX 1K 1
256, which means it has more than 2 prime Now, = =
factors. XB 3K 3
\ (17 × 5 × 11 × 3 × 2 + 2 × 11) is a composite AY 2 1 AX AY
number. Ans. And, = = Þ =
YC 6 3 XB YC
7. Find whether the following pair of linear
\ XY || BC Ans.
equations is consistent or inconsistent :
(By converse of Thales’ theorem)
3x + 2y = 8
9. Prove the following identity :
6x – 4y = 9 [2]
Sol. Given lines are 3x + 2y – 8 = 0 ...(1) sin3 q + cos3 q
= 1 – sin q · cos q [2]
and 6x – 4y – 9 = 0 ...(2) sin q + cos q
https://www.arundeepselfhelp.info/index.php?route=product/product&path=180&product_id=395

Arundeep’s Solved Papers 57 Mathematics 2015 (Term I)


sin 3 q + cos3 q
Sol. L.H.S. =
sin q + cos q

(sin q + cos q) (sin 2 q + cos 2 q - sin q ·cos q)


= [_ a3 + b3 = (a + b) (a2 + b2 – ab)]
(sin q + cos q)

= 1 – sin q · cos q = R.H.S. [Q sin2 q + cos2 q = 1] Hence Proved.


10. Show that the mode of the series obtained by combining the two series S1 and S2 given
below is different from that of S1 and S2 taken separately :
S1 : 3, 5, 8, 8, 9, 12, 13, 9, 9
S2 : 7, 4, 7, 8, 7, 8, 13 [2]
Sol. Mode of S1 series = 9 [_ observation 9 repeated max. no. of times i.e. 3 times]
Mode of S2 series = 7 [_ observation 7 repeated max no. of times i.e. 3 times]
After combining S1 and S2, the new series will be ; 3, 5, 8, 8, 9, 12, 13, 9, 9, 7, 4, 7, 8, 7, 8, 13.
Mode of combined series = 8 (maximum times i.e. 4 times)
Mode of (S1, S2) is different from mode of S1 and mode of S2 separately. Hence Proved.
SECTION – C
11. The length, breadth and height of a room are 8 m 50 cm, 6 m 25 cm and 4 m 75 cm
respectively. Find the length of the longest rod that can measure the dimensions of the
room exactly. [3]
Sol. To find the length of the longest rod that can measure the dimensions of the room exactly, we
have to find HCF of length, breadth and height
Given length of room L = 8 m 50 cm = 850 cm
= 21 × 52 × 17
Breadth of room B = 6 m 25 cm = 625 cm = 54
Height of room H = 4 m 75 cm = 475 cm = 52 × 19
\ HCF of L, B and H is 52 = 25 cm
[HCF of given numbers be the product of smallest power of each common prime factors in given
numbers]
\ Length of the longest rod = 25 cm Ans.
12. Solve by elimination :
3x – y = 7
2x + 5y + 1 = 0 [3]
Sol. Given equations are : 3x – y = 7 ...(i)
2x + 5y = – 1 ...(ii)
Multiplying equation (i) by 5 and solving it with equation (ii), we get
2x + 5y = – 1
15x – 5y = 35
17x = 34 (on Adding)
34
Þ x = =2
17
https://www.arundeepselfhelp.info/index.php?route=product/product&path=180&product_id=395

Arundeep’s Solved Papers 58 Mathematics 2015 (Term I)


Putting the value of x in (i), we have 14. The sum of the digits of a two digit
3 (2) – y = 7 number is 8 and the difference between
Þ 6–y=7Þ–y=7–6 Þ y=–1 the number and that formed by reversing
\ x = 2, y = – 1 Ans. the digits is 18. Find the number. [3]
13. Find a quadratic polynomial, the sum Sol. Let unit digit = x
and product of whose zeroes are 0 and and Tens digit = y
3 So, original number = unit digit +10× tens digit
- respectively. Hence find the zeroes.
5 = x + 10y
[3] According to question, we have
Sol. Given sum of zeroes = 0 Sum of digits of required number = 8
-3 so, x + y = 8 ...(i)
and product of zeroes = On reversing the digits, unit digit = y
5
Required quadratic polynomial = f(x) Tens digit = x
= x2 – (Sum of zeroes) x + Product of zeroes so, New number = 10x + y
According to question, we have
æ -3 ö 3
= x 2 - (0) x + ç ÷ = x 2 - Þ x + 10y – (10x + y) = 18
è 5 ø 5
Þ x + 10y – 10x – y = 18
æ 3ö
2
Þ 9y – 9x = 18
= ( x) 2 - ç ÷
è 5ø Þ y–x=2 ...(ii)
By adding eq. (i) and (ii)
æ 3ö æ 3ö
=çx- ÷ çx + ÷ 10
è 5ø è 5ø 2y = 10 Þ y =
2
é By applying ù Þ y=5
ê(a 2 - b2 ) = ( a + b) (a - b) ú Put the value of y in eq. (i) ; we have
ë û
Þ x+5=8 Þ x=8–5
Zeroes are given by putting f(x) = 0
Þ x=3
æ 3ö æ 3ö \ Required original number = 10y + x
çç x - ÷÷ çç x + ÷=0
è 5ø è 5 ÷ø = 10 × 5 + 3 = 50 + 3 = 53 Ans.
15. In given figure, EB ^ AC, BG ^ AE and
3 3 CF ^ AE. Prove that :
Þ x = or x = -
5 5
E

3 5 3 5
Þ x = ´ or x = - ´ F
5 5 5 5
G
D
15 - 15
Þ x= or x = Ans.
5 5 A C
B
https://www.arundeepselfhelp.info/index.php?route=product/product&path=180&product_id=395

Arundeep’s Solved Papers 59 Mathematics 2015 (Term I)


(i) DABG ~ DDCB Sol. AC2 = BC2 – AB2 [Given]
BC BE AC2 + AB2 = BC2
(ii) = [3]
BD BA \ ÐBAC = 90º

C
Sol.

E P

F
G A B
7 D
2 3 [By converse of Pythagoras theorem]
A
1 6 5 4
C
DAPB ~ DCPA
B
[If a perpendicular is drawn from the vertex
Given : EB ^ AC, BG ^ AE and CF ^ AE of the right angle of a triangle to the
To prove : hypotenuse then triangles on both sides of
the perpendicular are similar to the whole
(i) DABG ~ DDCB triangle and to each other]
BC BE
(ii) = AP PB
BD BA Þ =
CP PA
Proof : (i) In DABG and DDCB, BG || CF
as corresponding angles are equal. [In similar triangle, corresponding sides are
proportional]
Ð2 = Ð5 [Each 90º]
Þ PA2 = PB · CP Hence Proved.
Ð6 = Ð4 [Corresponding angles]
\ DABG ~ DDCB Hence Proved. 12
17. If sin q = , 0º < q < 90º, find the value
[By AA axiom of similarity] 13
Ð1 = Ð3 [CPCT] of :
(ii) In DABE and DDBC
sin 2 q - cos 2 q 1
Ð1 = Ð3 [Proved above] ´ [3]
2 sin q ·cos q tan 2 q
ÐABE = Ð5
[Each is 90º, EB ^ AC (Given] 12
Sol. Given, sin q =
DABE ~ DDBC[By AA axiom of similarity] 13
In similar triangles, corresponding sides are
P 12
proportional. Þ sin q = =
H 13
BC BE
\ = Hence Proved. Let, P = 12K, H = 13K
BD BA
P2 + B2 = H2 [Pythagoras theorem]
16. In triangle ABC, if AP ^ BC and
AC 2 = BC2 – AB 2 , then prove that Þ (12K)2 + B2 = (13K)2
PA2 = PB × CP. [3] 144K2 + B2 = 169K2
https://www.arundeepselfhelp.info/index.php?route=product/product&path=180&product_id=395

Arundeep’s Solved Papers 60 Mathematics 2015 (Term I)


B2 = 169K2 – 144K2 = 25K2 Þ B = 5K

B 5K 5
\ cos q = = =
H 13K 13
H
P
P 12K 12
\ tan q = = =
B 5K 5
B
sin 2 q - cos 2 q 1
Now, ´
2 sin q ·cos q tan 2 q

2 2
æ 12 ö æ5ö 144 - 25
ç ÷ -ç ÷
= è 13 ø è 13 ø ´
1 25
= 169 ´
æ 12 ö æ 5 ö 2
2 ç ÷ ç ÷ æç 12 ö÷
120 144
è 13 ø è 13 ø è 5 ø 169

119 25 595
= ´ =
120 144 3456 Ans.

2
12 æ 12 ö 144 25 5
Aliter : Given sin q = \ cos q = 1 - sin 2 q = 1 - ç ÷ = 1 - = =
13 è ø
13 169 169 13

sin q 12 / 13 12
\ tan q = = =
cos q 5 / 13 5

2 2
æ 12 ö æ 5 ö 144 - 25
ç ÷ -ç ÷
è 13 ø è 13 ø ´ 1
= 169 ´
25 119 25
´ =
595
Thus, given expression = 2 =
12 5 æ 12 ö 120 144 120 144 3456
2´ ´
13 13 ç ÷ 169
è 5ø

p2 - 1
18. If sec q + tan q = p, prove that sin q = [3]
p2 + 1

p2 - 1
Sol. R.H.S. =
p2 + 1

(sec q + tan q)2 - 1 sec 2 q + tan 2 q + 2 sec q tan q - 1


= = [By (a + b)2 = a2 + b2 + 2ab]
(sec q + tan q) + 1
2
sec 2 q + tan 2 + 2 sec q tan q + 1

(sec 2 q - 1) + tan 2 q + 2 sec q tan q


=
sec2 q + (1 + tan 2 q) + 2 sec q tan q
https://www.arundeepselfhelp.info/index.php?route=product/product&path=180&product_id=395

Arundeep’s Solved Papers 61 Mathematics 2015 (Term I)

tan 2 q + tan 2 q + 2 sec q tan q éQ sec 2 q - 1 = tan 2 q ù


= ê 2 ú
ëand sec q = 1 + tan qû
2
sec 2 q + sec2 q + 2 sec q tan q

sin q
2 tan 2 q + 2 sec q tan q 2 tan q (tan q + sec q) tan q
= = = = cos q
2 sec 2 q + 2 sec q tan q 2 sec q (sec q + tan q) sec q 1
cos q
= sin q = L.H.S.
Aliter : Given sec q + tan q = p ...(1)
\ sec2q – tan2q = 1 Þ (sec q – tan q) (sec q + tan q) = 1
1
Þ sec q – tan q = ....(2)
p
on adding (1) and (2) ; we have
1 p2 + 1 2p
2 sec q = p + p = Þ cos q = 2
p p +1

2
æ 2p ö
\ sin q = 1 - cos2 q = 1 - ç 2 ÷
è p +1ø

( p 2 + 1) 2 - 4 p 2 ( p 2 - 1) 2 p2 - 1
= = =
( p 2 + 1) 2
( p 2 + 1) p2 + 1
2

Hence Proved.
19. Find the mean of the following distribution by Assumed Mean Method : [3]

Class interval Frequency


10-20 8
20-30 7
30-40 12
40-50 23
50-60 11
60-70 13
70-80 8
80-90 6
90-100 12
https://www.arundeepselfhelp.info/index.php?route=product/product&path=180&product_id=395

Arundeep’s Solved Papers 62 Mathematics 2015 (Term I)


Sol. The table of values is given as under :

Class Frequency di = xi –55


interval xi fi di
(fi ) where A = 55
10-20 8 15 -40 -320
20-30 7 25 -30 -210
30-40 12 35 -20 -240
40-50 23 45 -10 -230
50-60 11 55 0 0
60-70 13 65 10 130
70-80 8 75 20 160
80-90 6 85 30 180
90-100 12 95 40 480
Sfi Sfidi = –50

S f i di æ - 50 ö = 55 - 50
Mean = A + = 55 + ç ÷ = 55 – 0.5 = 54.5 Ans.
S fi è 100 ø 100
20. The average score of boys in the examination of a school is 71 and that of the girls is 73.
The average score of the school in the examination is 71.8. Find the ratio of number of
boys in the number of girls who appeared in the examination. [3]
Sol. Let the number of boys = n1
and number of girls = n2
Average boys’ score = 71 = X1 (Let)

Average girls’ score = 73 = X 2 (Let)

n1X1 + n2 X 2
Combined mean =
n1 + n2

n1 (71) + n2 (73)
Þ 71.8 = Þ 71n1 + 73n2 = 71.8n1 + 71.8n2
n1 + n2
Þ 71n1 – 71.8n1 = 71.8n2 – 73n2 Þ – 0.8n1 = – 1.2n2
n1 1.2 n1 3
Þ n = Þ n = Þ n1 : n2 = 3 : 2
2 0.8 2 2
\ No. of boys : No. of girls = 3 : 2. Ans.
https://www.arundeepselfhelp.info/index.php?route=product/product&path=180&product_id=395

Arundeep’s Solved Papers 63 Mathematics 2015 (Term I)


SECTION – D
x 6 3 0 x 6 3 0
21. Find HCF of numbers 134791, 6341 and y 0 1 2 y 0 -2 -4
6339 by Euclid’s division algorithm. [4]
Sol. First we find HCF of 6339 and 6341 by draw the points (6, 0), (3, 1), (0, 2) on graph
Euclid’s division method paper and join them by straight line to get
the graph of x + 3y = 6 and draw the points
6339 6341 1 (6, 0), (3, –2), (0, –4) on graph paper and
6339
2 6339 3169 join them by line to give the graph of
6 2x – 3y = 2.
3
2
13 Y
12
19
18 4
x+3
y=6
1 2 2 3
2 2
B(0, 2)
0 (3, 1)
1 (6, 0)
A
6341 > 6339 Þ 6341 = 6339 × 1 + 2 X' O X
–5 –4 –3 –2 –1 1 2 3 4 5 6
Also, 6339 = 2 × 3169 + 1 –1
–2 (3, –2)
2=1×2+0
–3
\ HCF of 6341 and 6339 is 1.
–4 C (0, –4)
Now, we find the HCF of 134791 and 1 12 –5
y=
134791 = 1 × 134791 + 0 x –3
2
\ HCF of 134791 and 1 is 1. Y'

Hence, HCF of given three numbers is 1.


Ans. Since, area of triangle
22. Draw the graph of the following pair of
1
linear equations : = × base × corresponding altitude
2
x + 3y = 6 and 2x – 3y = 2
Find the ratio of the areas of the two 1
triangles formed by first line, x = 0, y = 0 ´ OA ´ OB
Area of DAOB 2
and second line, x = 0, y = 0. [4] \ =
Area of DAOC 1 ´ OA ´ OC
Sol. First Line Second Line 2
x + 3y = 6 2x – 3y = 12
Þ x = 6 – 3y Þ 2x = 12 + 3y OB 2 1
= = =
OC 4 2
12 + 3 y
Þx = \ Required ratio = 1 : 2 Ans.
2
https://www.arundeepselfhelp.info/index.php?route=product/product&path=180&product_id=395

Arundeep’s Solved Papers 64 Mathematics 2015 (Term I)


23. If the polynomial (x4
+ 2x3
+ 8x2
+ 12x + 18) is divided by another polynomial
(x2 + 5), the remainder comes out to be (px + q), find the values of p and q. [4]
Sol. By long division method, we have 2
+2 +3
\ Remainder = 2x + 3 2
x + 4
x + 3
+8 2
+ 1 2 x +1 8
Also given remainder = px + q
i.e., px + q = 2x + 3 + x4 + 5x2
\ p = 2, q = 3 Ans. – –
2x + 3x2 + 12 x + 18
3

+ 2x 3 + 10x
– –
3x2 + 2x + 18
+ 3x 2 + 15
– –
2x + 3

24. What must be subtracted from p (x) = 8x4 + 14x3 – 2x2 + 8x – 12 so that 4x2 + 3x – 2 is
factor of p (x) ? This question was given to group of students for working together. Do you
think teacher should promote group work ? [4]
Sol. By long division method, we have 2 x 2 +2x - 1
For 4x2 + 3x – 2 be a factor of p(x) 4 x +3x – 2 8 x + 14x 3 – 2x 2 + 8x – 12
\ remainder must be equal to 0.
8x4 + 6x3 – 4 x 2
Thus, polynomial to be subtracted is (15x – 14). – – +
8x3 + 2x2 + 8x – 12
+ 8x3 + 6x2 – 4 x
– – +
– 4x2 + 12 x –12
– 4x2 –3x + 2
+ + –
15x – 14
Value : Yes, as it increases confidence and team spirit among students. Ans.
25. Prove “If a line is drawn parallel to one side of a triangle to intersect the other two sides in
distinct points, the other two sides are divided in the same ratio.” [4]
Sol. Given, In DABC, DE || BC.
AD AE
To prove : =
DB BC
Construction : Draw EM ^ AB and DN ^ AC. Join B to E and C to D.
A
Proof : In DADE and DBDE, we have
M N
1
´ AD ´ EM D E
ar ( DADE) = 2 =
AD
1 ...(i)
ar ( DBDE) ´ DB ´ EM
DB
2 B C
https://www.arundeepselfhelp.info/index.php?route=product/product&path=180&product_id=395

Arundeep’s Solved Papers 65 Mathematics 2015 (Term I)

1 EC CF
[Area of D = × base × corresponding =
2 EA CB
altitude] ÐECF = ÐACB [Common]
In DADE and DCDE, we have
DCFE ~ DCBA [SAS similarity]
1
´ AE ´ DN EF CE
ar ( DADE) AE Þ =
= 2 = ...(ii) AB CA
ar ( DCDE) 1 EC
´ EC ´ DN [In similar D’s, corresponding sides are
2
proportional]
Since, DE || BC [Given]
\ ar (DBDE) = ar (DCDE) ...(iii) EF 4
Þ = [Q AB = 3 + 4 = 7 cm]
[Ds on the same base and between the same 7 9
parallel sides are equal in area]
28
From eq. (i), (ii) and (iii) ; we have \ EF = cm and AB = 7 cm Ans.
9
AD AE
= Hence Proved. 1
DB EC 27. If tan (A + B) = 3 and tan (A – B) = ,
26. In the given figure, AD = 3 cm, AE = 5 cm, 3
BD = 4 cm, CE = 4 cm, CF = 2 cm, BF = where 0 < A + B < 90º, A > B, find A
2.5 cm, then find the pair of parallel line and B. Also calculate
and hence their lengths. [4] tan A . sin (A + B) + cos A . tan (A – B).
[4]
A Sol. Given,
3 cm 5 cm 1
tan (A + B) = 3, and tan (A – B) =
D E 3
4 cm 4 cm Þ tan (A + B) = tan 60º
Þ (A + B) = 60º ...(i)
B C
2.5 cm F 2 cm and, tan (A – B) = tan 30º
\ (A – B) = 30º ...(ii)
EC 4 CF 2 4
Sol. = and = = On adding eq. (i) and (ii) ; we have
EA 5 FB 2.5 5 2A = 90º
EC CF 90º
Þ = Þ A= = 45º
EA FB 2
In DABC, EF || AB From eq. (i), A + B = 60º
[Converse of Thales’ Theorem] Þ 45º + B = 60º
Þ B = 15º
CE 4 4
Also, = = ...(i) \ A = 45º, B = 15º
CA 4+5 9
Now,
CF 2 2 4 tan A . sin (A + B) + cos A . tan (A – B)
and = = =
CB 2 + 2.5 4.5 9 = tan 45º . sin (60º) + cos 45º . tan (30º)
https://www.arundeepselfhelp.info/index.php?route=product/product&path=180&product_id=395

Arundeep’s Solved Papers 66 Mathematics 2015 (Term I)

3 1 1 3 1 6 3 6 3 3+ 6
= 1´ + ´ = + ´ = + = Ans.
2 2 3 2 6 6 2 6 6
28. Prove that :

sec 3 A - cosec 3 A
(1 + cot A + tan A) · (sin A – cos A) = [4]
sec 2 A ·cosec 2 A
Sol. L.H.S. = (1 + cot A + tan A) (sin A – cos A)

æ cos A sin A ö æ sin A cos A + cos 2 A + sin 2 A ö


= ç1 + + ÷ (sin A – cos A) = ç ÷ (sin A – cos A)
è sin A cos A ø è sin A .cos A ø

sin 3 A - cos3 A
= [Using a3 – b3 = (a – b) (a2 + ab + b2)]
sin A .cos A

sin3 A cos3 A
-
sin 3 A .cos3 A sin 3 A .cos3 A
=
sin A cos A [Dividing Num. and Deno. by sin3 A . cos3 A]
sin 3 A .cos3 A

sec3 A - cosec3 A
= = R.H.S. Hence Proved.
sec 2 A .cosec2 A
29. Prove the identity :
sin A + cos A sin A - cos A 2
+ = [4]
sin A - cos A sin A + cos A 1 - 2 cos2 A

sin A + cos A sin A - cos A


Sol. L.H.S. = +
sin A - cos A sin A + cos A

sin 2 A + cos2 A + 2 sin A cos A


(sin A + cos A)2 + (sin A - cos A) 2 = + sin 2 A + cos 2 A - 2 sin A cos A
=
(sin A - cos A) (sin A + cos A) sin 2 A - cos 2 A

1+1
= [Q sin2 A + cos2 A = 1, sin2 A = 1 – cos2 A]
1 - cos 2A - cos 2 A

2
= = R.H.S. Hence Proved.
1 - 2 cos2 A
30. The following table gives the daily income of 50 workers of a factory. Draw both types
(“less than type” and “greater than type”) ogives.
https://www.arundeepselfhelp.info/index.php?route=product/product&path=180&product_id=395

Arundeep’s Solved Papers 67 Mathematics 2015 (Term I)

Daily income (in `) No. of workers


100 – 120 12
120 – 140 14
140 – 160 8
160 – 180 6
180 – 200 10

[3]
Sol.

Less than ogive More than ogive

No. of No.
Daily In- work- Daily Income
of
come (in `) ers (in `)
work-
(c.f.) ers
Less than 120 12 (c.f.)
Less than 140 26 More than 100 50
Less than 160 34 More than 120 38
Less than 180 40 More than 140 24
Less than 200 50 More than 160 16
More than 180 10
For less than ogive : Plot the points (120, 12), (140, 26), (160, 34), (180, 40) and (200, 50) on
graph paper and join them by free hand to gives the required less than ogive.
For more than ogive : plot the points (100, 50), (120, 38), (140, 24), (160, 16), (180, 10) on
graph paper and join them by free hand to give the required more than ogive.

31. In a class test, marks obtained by 120 students are given in the following frequency
distribution. If it is given that mean is 59, find the missing frequencies x and y. [4]
https://www.arundeepselfhelp.info/index.php?route=product/product&path=180&product_id=395

Arundeep’s Solved Papers 68 Mathematics 2015 (Term I)

Marks No. of students


0 – 10 1
10 – 20 3
20 – 30 7
30 – 40 10
40 – 50 15
50 – 60 x
60 – 70 9
70 – 80 27
80 – 90 18
90 – 100 y

Sol. The table of values is given as under :

No. of
Students x i - 55
Marks xi di = fi d i
fi 10

0-10 1 5 –5 –5
10-20 3 15 –4 –12
–73

20-30 7 25 –3 –21
30-40 10 35 –2 –20
40-50 15 45 –1 –15
50-60 x A= 55 0 0
60-70 9 65 1 9
117 + 4

70-80 27 75 2 54
80-90 18 85 3 54
90-100 y 95 4 4y
Sfi = 90 fi di = 44 + 4y
+x+y

Since, Sfi = 90 + x + y
https://www.arundeepselfhelp.info/index.php?route=product/product&path=180&product_id=395

Arundeep’s Solved Papers 69 Mathematics 2015 (Term I)

But Sfi = 120 [Given]


\ 90 + x + y = 120
x = 120 – 90 – y = 30 – y ...(i)

S fi di
Also, mean = A + ´ h [using step deviation method]
S fi

æ 44 + 4 y ö
Þ 59 = 55 + ç ´ 10 ÷ [A = 55, h = 10, Sfi = 120]
è 120 ø

4 (11 + y )
Þ 59 – 55 = Þ 4 × 3 = 11 + y Þ y = 12 – 11 = 1
12
From eq. (i), x = 30 – 1 = 29
\ x = 29, y = 1 Ans.
https://www.arundeepselfhelp.info/index.php?route=product/product&path=180&product_id=395

MATHEMATICS 2015 TERM II (OUTSIDE DELHI)


SET I
Time allowed : 3 hours Maximum marks : 90
General Instructions – As usual.

3. Two different dice are tossed together. Find


1. If the quadratic equation the probability that the product of the two
px 2 - 2 5 px + 15 = 0 has two equal numbers on the top of the dice is 6.
roots, then find the value of p. Sol. Total number of outcomes = 36
Sol. Given quadratic eqn. be Here sample space = {(1, 1), (1, 2)....(1, 6)
(2, 1), (2, 2) ......(2, 6)
px2 – 2 5 px + 15 = 0
(3, 1), (3, 2) .... (3, 6)
On comparing with ax2 + bx + c = 0
(4, 1), (4, 2) .... (4, 6)
we have a = p : b = – 2 5 p ; c = 15 (5, 1), (5, 2) ..... (5, 6)
For equal roots, D = 0 (6, 1), (6, 2) ...... (6, 6)
Þ b2 – 4ac = 0 Let A be the event such that
A = Product of the numbers on the top of
Þ ( -2 5) 2 - 4 ´ p ´ 15 = 0
the dice is 6.
Þ 20 – 60p = 0 \ Favourable outcomes (1, 6), (2, 3), (3, 2),
1 (6, 1)
Þ p= \ No. of favourable out comes = 4
3
2. In Figure given below a tower AB is 20 4 1
m high and BC, its shadow on the \ Required probability = P (A) = =
36 9
ground, is 20 3 m long. Find the Sun’ss 4. In Figure given below PQ is a chord of a
altitude. circle with centre O and PT is a tangent.
If ÐQPT = 60º, find ÐPRQ.

Sol. Let Sun’s altitude = q = ÐACB Sol. ÐXPQ + ÐQPT = 180º (linear pair)
In right angled DABC, we have Þ ÐXPQ + 60º = 180º
AB Þ ÐXPQ = 120º
\ tan q =
BC Now ÐXPQ = ÐPRQ [Angle made by
a chord with a tangent is equal to angle
20 1 subtended by the chord in alternate segment]
Þ tan q = = = tan 30° Þ q = 30º
20 3 3 \ ÐPRQ = 120º
Arundeep’s Solved Papers 70 Mathematics 2015 (Outside Delhi)
https://www.arundeepselfhelp.info/index.php?route=product/product&path=180&product_id=395

Arundeep’s Solved Papers 71 Mathematics 2015 (Outside Delhi)


5. In Figure given below two tangents RQ
and RP are drawn from an external point
R to the circle with centre O. If ÐPRQ =
120º, then prove that OR = PR + RQ.

Sol. Let AF = x cm
Q AF = AE [tangents from A]
[Q tangents drawn from a point outside the
circle are of equal length.]
Sol. OR bisects ÐPRQ
\ AE = x
\ ÐPRO = ÐQRO = 60º
Also BD = DF = 6 cm
In right DOPR (Q OP ^ PR) and CD = CE = 9 cm
PR 1 \ AB = (6 + x) cm and AC = (9 + x) cm
= cos 60º =
OR 2
Þ OR = 2PR ...(i)
QR 1
Similarly =
OR 2

Area DABC = Area DBOC + Area DCOA


+ Area DAOB
Þ OR = 2QR ...(ii)
(i) + (ii), we get 1 1 1
Þ 54 = BC × OD + AC × OE + AB ×OF
2OR = 2PR + 2QR 2 2 2
Thus, OR = PR + RQ Þ 54 × 2 = 15 × 3 + (9 + x) × 3 + (6 + x) 3
6. In Figure given below a triangle ABC is Þ 108 = 45 + 27 + 3x + 18 + 3x
drawn to circumscribe a circle of radius
Þ 6x = 18 Þ x = 3
3 cm, such that the segments BD and DC
are respectively of lengths 6 cm and 9 cm. \ AB = 6 + x = 6 + 3 = 9 cm
If the area of DABC is 54 cm2, then find and AC = 9 + x = 9 + 3 = 12 cm
the lengths of sides AB and AC.
https://www.arundeepselfhelp.info/index.php?route=product/product&path=180&product_id=395

Arundeep’s Solved Papers 72 Mathematics 2015 (Outside Delhi)


7. Solve the following quadratic equation When d =5, eqn. (i) becomes
for x : 12a + 155 = 167 Þ a = 1
4x2 + 4bx – (a2 – b2) = 0 Thus required AP is 1, 6, 11, ....
Sol. Given quadratic eqn. be, 9. The points A (4, 7), B (p, 3) and C (7, 3)
4x2 + 4bx – (a2 – b2) = 0 are the vertices of a right triangle, right-
angled at B. Find the value of p.
On comparing with ax2 + bx + c = 0
Sol. Using distance formula, we have
We have a = 4 ; b = 4b ; c = –(a2 – b2)
\ Discriminant D = b2 – 4ac Now AC = 32 + (- 4) 2 = 5
D = (4b)2 – 4 × 4 × [– (a2 – b2)]
= 16b2 + 16a2 – 16b2 = 16a2 AB = ( p - 4) 2 + 16

-b ± D A(4, 7)
\ x=
2a

- 4b ± 16a 2 - 4b ± 4a
\ x = =
2´4 8

4 ( - b ± a) - b ± a
= = C(7, 3) B(p, 3)
8 2
8. In an AP, if S5 + S7 = 167 and S10 = 235,
BC = ( p - 7)2 + 0
then find the AP, where Sn denotes the
sum of its first n terms. Now AC2 = AB2 + BC2
Sol. Let Ist term of the AP = a (using pythagoras theorem)
and common difference = d Þ 25 = (p – 4) + 16 + (p – 7)2
2

and required A.P. be a, a + d, a + 2d, .... Þ 25 = p2 – 8p + 16 + 16 + p2 – 14p + 49


Now S5 + S7 = 167 Þ 2p2 – 22p + 56 = 0 Þ p2 – 11p + 28 = 0
Þ p2 – 4p – 7p + 28 = 0
é n ù Þ
êëQ Sn = 2 {2a + (n - 1)d }úû
p(p – 4) – 7 (p – 4) = 0
Þ (p – 4) (p – 7) = 0 Þ p = 4 or p = 7
If p = 7 then B = (7, 3)
5 7
Þ (2a + 4d ) + (2a + 6d ) = 167 It coincide with C \ p ¹ 7
2 2
Hence, p = 4
Þ 5a + 10d + 7a + 21d = 167 10. Find the relation between x and y if the
Þ 12a + 31d = 167 ...(i) points A (x, y), B (– 5, 7) and C (– 4, 5)
Also S10 = 235 are collinear.
10
Sol. Q A, B and C are collinear
Þ (2a + 9d) = 235 Þ 2a + 9d = 47 Thus, A, B and C lies on same line
2
\ area of DABC = 0
Þ 6 (2a + 9d) = 6 × 47 \ x1 (y2 – y3) + x2 (y3 – y1) + x3 (y1 – y2) = 0
Þ 12a + 54d = 282 ...(ii) Þ x (7 – 5) + (– 5) (5 – y) + (– 4) (y – 7) = 0
eqn. (i) – eqn. (ii) gives ; Þ 2x – 25 + 5y – 4y + 28 = 0
– 23d = – 115 Þ d = 5
https://www.arundeepselfhelp.info/index.php?route=product/product&path=180&product_id=395

Arundeep’s Solved Papers 73 Mathematics 2015 (Outside Delhi)


Þ 2x + y + 3 = 0 at Q
gives the required relation. \ Distance covered in 15 seconds = PQ
11. The 14th term of an AP is twice its 8th PB
term. If its 6th term is – 8, then find the In right DPBA, = tan 60º
AB
sum of its first 20 terms.
Sol. Let 1st term of AP = a and 1500 3
common difference = d = 3 Þ AB = x = 1500 m
AB
According to given condition, we have
In right DQCA, we have
a14 = 2a8
Þ a + 13d = 2 (a + 7d) Þ a = – d QC 1500 3 1
= tan 30º Þ =
[_ an = a + (n – 1) d] AC x+ y 3
Also a6 = 8 Þ a + 5d = – 8 Þ x + y = 4500 m
Þ – d + 5d = – 8 Þ 4d = – 8 Þ d = – 2 \ BC = AC – AB = 3000 m
\ a=2 Also PQ = BC
n \ PQ = 3000 m
We know that Sn = {2a + (n – 1)d}
2 Distance covered
Thus required speed =
20 Time taken
S20 = (2 ´ 2 + 19 ´ ( - 2)
2 3000
= m/s = 200 m/s
= 10 × (– 34) = – 340 15
12. Solve for x :
3600
= 200 ´ km/hr = 720 km/hr
3x 2 - 2 2x - 2 3 = 0 1000
Sol. Given eqn. be 3x 2 - 2 2 x - 2 3 = 0 [_ 1 km = 1000 m and 1 hr = 3600s]

Þ 3x 2 - 3 2 x + 2 x - 2 3 = 0
P
Q
Þ 3x ( x - 6) + 2 ( x - 6) = 0

Þ ( x - 6) ( 3x + 2) = 0

either x – 6 = 0 or 3x + 2 =0

- 2 C
x = 6,
3 14. If the coordinates of points A and B are
13. The angle of elevation of an aeroplane (– 2, – 2) and (2, – 4) respectively, find
from a point A on the ground is 60º. After the coordinates of P such that
a flight of 15 seconds, the angle of 3
elevation changes to 30º. If the aeroplane AP = AB, where P lies on the line
7
is flying at a constant height of
segment AB.
1500 3 m, find the speed of the plane in
3
km/hr. Sol. Given AP = AB
7
Sol. Let plane is at P. After 15 seconds it reaches
https://www.arundeepselfhelp.info/index.php?route=product/product&path=180&product_id=395

Arundeep’s Solved Papers 74 Mathematics 2015 (Outside Delhi)


3 Þ 3x – y = 10 ...(i)
Þ AP = (AP + PB) Prob. of drawing a blue ball from jar
7
Þ 7AP = 3AP + 3PB y
Þ 4AP = 3PB Þ AP : PB = 3 : 4 =
x + y + 10
Also given prob. of drawing a blue ball
3:4
1
A(–2, –2) P B(2, –4) from jar =
3
\ P divides AB in the ratio 3 : 4
1 y
Then by section formula, we have
Also =
\ x coordinates of P 3 x + y + 10

3 ´ 2 + 4 ´ ( - 2) - 2 Þ x + y + 10 = 3y Þ x = 2y – 10 ...(ii)
= =
3+4 7 Putting x = 2y – 10 in equation (i) ; we have
and y coordinate of P 3 (2y – 10) – y = 10
Þ 6y – 30 – y = 10 Þ 5y Þ y = 8
3 ´ (- 4) + 4 ´ (- 2) -20
= = When y = 8, equation (ii) becomes
3+ 4 7
x = 2 × 8 – 10 Þ x = 6
æ - 2 - 20 ö \ Total number of balls in the jar = 6+ 8+ 10 = 24
\ Required coordinates of P are ç , ÷
è 7 7 ø 16. Find the area of the minor segment of a
15. The probability of selecting a red ball at circle of radius 14 cm, when its central
random from a jar that contains only angle is 60º. Also find the area of the
1 corresponding major segment.
red, blue and orange balls is . The
4 é 22 ù
probability of selecting a blue ball at êë Use π = 7 úû
1 Sol. In DAOB, ÐAOB = 60º
random from the same jar is . If the
3 Also AO = BO = radii of circle
jar contains 10 orange balls, find the total Since equal sides have equal angle opposite
number of balls in the jar. to it.
\ ÐOAB = ÐOBA
Sol. Let number of red balls in the jar = x
Also ÐOAB + ÐOBA + 60º = 180°
and number of blue balls in the jar = y
\ 2ÐOAB = 120° Þ ÐOBA = ÐOAB = 60°
\ Total number of balls in the jar = x + y = 10 \ DAOB is an equilateral D.
[We have given no. of orange balls in Jar = 10]
3
1 \ Area of equilateral DAOB = ´ 14 ´ 14
Given probability of selecting red ball = 4
4
Also prob. of selecting red ball = 49 3 cm 2
x and Area of sector AOB
= x + y + 10 q 60°
= pr2× = ´ p ´ 14 ´ 14
360° 360°
x 1
Þ = Þ 4x = x + y + 10 1 22 308
x + y + 10 4 = ´ ´ 14 ´ 14 = cm 2
6 7 3
https://www.arundeepselfhelp.info/index.php?route=product/product&path=180&product_id=395

Arundeep’s Solved Papers 75 Mathematics 2015 (Outside Delhi)


Radius of cone = R = 2.1 m,
height of cone h = 2.8 m.
Major segment

O
14
cm 60° cm
14 2.8 m

A Minor segment
B
4.2 m

Area of minor segment


4m
æ 308 ö
=ç - 49 3 ÷ cm 2
è 3 ø

22
Also, Area of circle = pr2 = ´ 14 ´ 14
7 Let slant height of conical part = l
= 616 cm2
Area of major segment \ l = (2.1)2 + (2.8) 2 = 3.5 m
\ Curved surface area of cone = prl
é æ 308 öù
= ê616 - ç - 49 3 ÷ú cm2
ë è 3 øû 22
= ´ 2.1 ´ 3.5 = 23.1 m2
7
æ 1540 ö
=ç + 49 3 ÷ cm 2 Area of canvas required for one tent
è 3 ø
= 52.8 + 23.1 = 75.9 m2
17. Due to sudden floods, some welfare
Canvas required for 100 tents = 100 × 75.9
associations jointly requested the
government to get 100 tents fixed = 7590 m2
immediately and offered to contribute \ Total cost of canvas = ` (7590 × 100)
50% of the cost. If the lower part of each = ` 759000
tent is of the form of a cylinder of
diameter 4.2 m and height 4 m with the Thus, amount to be paid by association
conical upper part of same diameter but 50
of height 2.8 m, and the canvas to be used = ´ 759000 = ` 379500
100
costs ` 100 per sq. m, find the amount,
the associations will have to pay. What So, values promoted are care for the society.
values are shown by these associations ? 18. A hemispherical bowl of internal
diameter 36 cm contains liquid. This
é 22 ù
êë Use π = 7 úû liquid is filled into 72 cylindrical bottles
of diameter 6 cm. Find the height of the
Sol. Given Height of cylinder = H = 4 m each bottle, if 10% liquid is wasted in
Radius of cylinder = R = 2.1 m this transfer.
\ Curved surface area of cylinder = 2pRH Sol. Radius of hemispherical bowl = R = 18 cm
22 2
= 2´ ´ 2.1 ´ 4 = 52.8 m2 Volume of liquid in bowl = pR 3
7 3
https://www.arundeepselfhelp.info/index.php?route=product/product&path=180&product_id=395

Arundeep’s Solved Papers 76 Mathematics 2015 (Outside Delhi)

2 = 6 × 102 + 2 × 3.14 × 5 × 5 – 3.14 × 5 × 5


= × p × 18 × 18 × 18 = 3888 p cm3 = 600 + 78.5 = 678.5 cm2
3
\ Total cost of painting the solid @ the rate
\ Quantity of Liquid wasted = 10% of total
10 678.5 ´ 5
volume of liquid = ´ 3888 p cm3 of Rs. per 100 eqn. cm = `
100 100
= ` 33.92
\ Volume of Liquid transferred into bottles
20. 504 cones, each of diameter 3.5 cm and
3888 height 3 cm, are melted and recast into a
= 3888p - p
metallic sphere. Find the diameter of the
10
sphere and hence find its surface area.
34992
= p cm3 é 22 ù
10 êë Use π = 7 úû
Given Radius of bottle = r = 3 cm
Let height of each bottle = x cm 3.5
Sol. Given radius of cone r = cm
2
\ Volume of one bottle = pr2h
and height of cone = h = 3 cm
=p×3×3×x
= 9px cm3 \ Volume of one cone =
1
´ p r2 h
Thus, Volume of 72 bottles = 72 × 9px cm3 3
= 648px cm3 1 3.5 3.5 12.25
= p´ ´ ´ 3 cm3 = p cm3
according to given condition, we have 3 2 2 4
34992 \ Volume of 504 cones
648px = p
10 12.25
= 504 ´ p cm3
34992 4
x = = 5.4 cm = 1543.5 p cm3
10 ´ 648
Since it is given that 504 cones are melted
\ required height of each bottle be 5.4 cm.
and recast into a metallic sphere let R be
19. A cubical block of side 10 cm is the radius of sphere
surmounted by a hemisphere. What is
the largest diameter that the hemisphere 4p 3
Then volume of sphere = R
can have ? Find the cost of painting the 3
total surface area of the solid so formed,
at the rate of ` 5 per 100 sq. cm. \ Volume of sphere = 1543.5 p cm3
[Use p = 3.14]
4
Sol. Largest diameter of hemisphere = side of pR 3 = 1543.5 p
3
cubical block = 10 cm
\ Radius of hemisphere = 5 cm 1543.5 ´ 3
Þ R3 = = 1157.625
Total surface area of the solid = Surface 4
area of cube + Curved surface area of R = 3 1157.625 = 10.5 cm
hemisphere – Area of base of hemisphere
\ Surface area of sphere = 4pR2
= 6r2 + 2pr2 – pr2
https://www.arundeepselfhelp.info/index.php?route=product/product&path=180&product_id=395

Arundeep’s Solved Papers 77 Mathematics 2015 (Outside Delhi)


22 n
= 4´ ´ 10.5 ´ 10.5 cm2 = 1386 cm2 [we know that Sn = (a + an)]
7 2
21. The diagonal of a rectangular field is 60
16 metres more than the shorter side. If \ S60 = ( a + a60 ) = 30 (8 + 126)
2
the longer side is 14 metres more than
the shorter side, then find the lengths of = 30 × 134 = 4020
the sides of the field. 50
Sol. Let shorter side of rectangular field = x cm and S50 = (2a + 49d ) = 25 (16 + 49 × 2)
2
\ Diagonal of rectangular field = (x + 16) m
= 25 (114) = 2850
and longer side of rectangular field \ Sum of last 10 terms = S60 – S50
= (x + 14) m
= 4020 – 2850
Then by pythagoras theorem, we have
= 1170
Now (x + 16)2 = (x + 14)2 + x2 23. A train travels at a certain average speed
Þ x2 + 32x + 256 = x2 + 28x + 196 + x2 for a distance of 54 km and then travels
Þ x2 – 4x – 60 = 0 a distance of 63 km at an average speed
Þ x2 – 10x + 6x – 60 = 0 of 6 km/h more than the first speed. If it
Þ x(x – 10) + 6 (x – 10) = 0 takes 3 hours to complete the total
journey, what is its first speed ?
Þ (x – 10) (x + 6) = 0
Sol. Let average speed of the train = x km
Þ x = 10 or x = – 6
Distance covered by train = 54 km
Rejecting x = – 6,
\ Time taken by train to covered a distance
Since length of side of rectangular field
54
can’t be negative of 54 km = hr
Thus x = 10 x
\ Shorter side of rectangular field = 10 m, Also distance to be covered by train = 63 km
length of diagonal = 26 m and speed = (x + 6) km/hr
and longer side of rectanglar field = 24 m \ Time taken by train to covered a distance
63
of 63 km = hr
x+6
According to given condition, we have
54 63
+ =3
x x+6
54 ( x + 6) + 63x
Þ =3
22. Find the 60th term of the AP 8, 10, 12, ( x + 6) x
..., if it has a total of 60 terms and hence Þ 54x + 324 + 63x = 3 (x2 + 6x)
find the sum of its last 10 terms. Þ 117x + 324 = 3x2 + 18x
Sol. Given, AP is 8, 10, 12, ... Þ 3x2 – 99x – 324 = 0
Here, a = 8, d = 2 Þ x2 – 33x – 108 = 0
a60 = a + 59d [_ an = a + (n – 1) d] Þ x2 – 36x + 3x – 108 = 0
= 8 + 59 × 2 Þ (x – 36) (x + 3) = 0
= 8 + 118 = 126 Þ x(x – 36) +3 (x – 36) = 0
https://www.arundeepselfhelp.info/index.php?route=product/product&path=180&product_id=395

Arundeep’s Solved Papers 78 Mathematics 2015 (Outside Delhi)


Þ x = 36 or x = – 3
\ First speed of train = 36 km/hr
24. Prove that the lengths of the tangents
drawn from an external point to a circle
are equal.
Sol. Given : A circle C (O, r), P is a point outside
the circle and PA and PB are tangents to a Þ AC = BC (CPCT)
circle. Þ OC ^ AB (line joining mid-point of chord
To Prove : PA = PB with centre of the circle is perpendicular to
Construction : Draw OA, OB and OP. the chord)
Proof : Consider triangles OAP and OBP. Also ÐOPY = 90º
ÐOAP = ÐOBP = 90º ...(i) Þ ÐOCB=ÐOPYthese are corresponding angles
A \ AB || XY
26. Construct a DABC in which AB = 6 cm,
ÐA = 30º and ÐB = 60º. Construct
P O
another DAB¢C¢ similar to DABC with
base AB¢ = 8 cm.
B Sol.

[Radius is perpendicular to the tangent at


the point of contact]
OA = OB (radii) ...(ii)
OP is common ...(iii)
\ DOAP @ DOBP (RHS axiom of congruency)
[from (i), (ii), (iii)]
Þ AP = BP (cpct)
25. Prove that the tangent drawn at the mid-
point of an arc of a circle is parallel to
the chord joining the end points of the
arc. Steps of construction :
Sol. Given : AB is arc of the circle C (O, r), P is
(i) draw in line segment AB = 6 cm
mid-point of arc AB and XY is tangent to
the circle at P. (ii) at A, construct ÐBAQ = 30°
To Prove : AB || XY (iii) at B, construct ÐABP = 60°
Join OA and OB (iv) Let ÐABP and ÐBAQ intersects at C
Here Join CA and CB, to get the DABC.
ÐAOP = ÐBOP (angle subtended by
(v) Produce AB to B¢ s.t. AB¢ = 8 cm
equal arc)
OA = OB, OC = OC (vi) at B¢, construct ÐAB¢Z = 60°
\ DACO @ DBCO making ÐBAQ at C¢ s.t. B¢C¢ || BC
(SAS axiom of congurency) Then DAB¢C¢ is similar to DABC.
https://www.arundeepselfhelp.info/index.php?route=product/product&path=180&product_id=395

Arundeep’s Solved Papers 79 Mathematics 2015 (Outside Delhi)


27. At a point A, 20 metres above the level Now In right DADC, we have
of water in a lake, the angle of elevation
of a cloud is 30º. The angle of depression AC
= cosec 30º
of the reflection of the cloud in the lake, CD
at A is 60º. Find the distance of the cloud
from A. AC
Þ =2
20
Þ AC = 40 m
\ Required distance of the cloud from A= 40m
x
28. A card is drawn at random from a well-
D shuffled deck of playing cards. Find the
E probability that the card drawn is
(i) a card of spade or an ace.
x (ii) a black king.
R (iii) neither a jack nor a king
(iv) either a king or a queen
Sol. Let C is cloud and R is its reflection.
Sol. Total no. of outcomes = 52
ÐDAC = 30º, ÐDAR = 60º, let CD = x m
A = Card is spade or an ace
\ Height of the cloud above the lake
Cards favourable to A = 13 + 3 = 16
= (x + 20) m
[Since there are 13 spade cards and 3 ace
\ ER = (20 + x) m. cards, 1 ace of spade is already counted in
Now In right DADC, we have spade cards.]

CD 16 4
= tan 30º \ Required prob. P (A) = =
AD 52 13

x 1 (ii) B = Card is black king


Þ = \ No. of black kings = 2 = total no. of
AD 3
favourable cases
Þ AD = 3x ...(i) 2 1
In right DADR, we have \ P (B) = =
52 26
DR (iii) C = Card is neither a jack nor a king
= tan 60º
AD No. of favourable cards to C= 52 – 4– 4 = 44

Þ DE + ER = 3 \ Required prob. P (C) =


44 11
=
AD 52 13
(iv) D = Card is either a king or a queen
20 + 20 + x
Þ = 3 (using (i)) No. of cards favourable to event D = 4 + 4 = 8
3x
Þ 40 + x = 3x 8 2
\ Required prob. P (D) = =
52 13
Þ x = 20 m
https://www.arundeepselfhelp.info/index.php?route=product/product&path=180&product_id=395

Arundeep’s Solved Papers 80 Mathematics 2015 (Outside Delhi)


29. Find the values of k so that the area of the triangle with vertices (1, – 1), (–4, 2k)
and (– k, – 5) is 24 sq. units.
Sol. Area of D = 24
1
Þ | x (y – y ) + x2 (y3 – y1) + x3 (y1 – y2) | = 24 | 1 (2k + 5) – 4 (– 5 + 1) – k (– 1 – 2k) | = 48
2 1 2 3
Þ | 2k + 5 + 16 + k + 2k2 | = 48 Þ | 2k2 + 3k + 21 | = 48
Þ 2k2 + 3k + 21 = ± 48 Þ 2k2 + 3k + 21 = 48 or 2k2 + 3k + 21 = – 48
Þ 2k2 + 3k – 27 = 0 or 2k2 + 3k + 69 = 0
Case - I : 2k2 + 3k – 27 = 0
Þ 2k2 + 9k – 6k – 27 = 0
Þ k (2k + 9) – 3 (2k + 9) = 0 Þ (2k + 9) (k – 3) = 0
-9 -9
Þ k = or k = 3 Þ k = or k = 3
2 2
Case - II : 2k2 + 3k + 21 + 48 = 0
Þ 2k2 + 3k + 69 = 0
on comparing with ax2 + bx + c = 0
we have a = 2 ; b = 3 ; c = 69
Here, D = b2 – 4ac = 32 – 4 × 2 × 69 = 9 – 552 = – 543 < 0
\ given quadratic eqn. has no. soln. in this case.
-9
Thus, k = or 3
2
30. In Figure given below PQRS is a square lawn with side
PQ = 42 metres. Two circular flower beds are there on the
sides PS and QR with centre at O, the intersection of its
diagonals. Find the total area of the two flower beds
(shaded parts).
Sol. Here PR2 = PQ2 + QR2 [using pythagoras theorem]
PR2 = (42)2 + (42)2
Þ PR = 42 2 m

42 2
Þ PO = = 21 2 m [Since diagonals of square bisect each other at right angles]
2
90º
Area of sector POS = ´ p (21 2)2
360º
1 22
= ´ ´ 21 ´ 21 ´ 2 = 693 m2
4 7
1
\ Area of DPOS = PO × OS (Q PO ^ OS)
2
https://www.arundeepselfhelp.info/index.php?route=product/product&path=180&product_id=395

Arundeep’s Solved Papers 81 Mathematics 2015 (Outside Delhi)

1 \ Volume of 2 hemispheres
= ´ 21 2 ´ 21 2 = 441 m2
2 = 2 × 49.392 p cm3
\ Area of one flower bed = 693 – 441 = 252 m2 = 98.784 p cm3
Thus, Area of two flower bed = 2 × 252 = Given Height of cylinder = 10 cm = h
504 m2 and Radius of cylinder = 4.2 cm = r
31. From each end of a solid metal cylinder, \ Volume of cylinder = pr2h
metal was scoped out in hemispherical
= p × (4.2)2 × 10 = 176.4 p
form of same diameter. The height of the
cylinder is 10 cm and its base is of radius \ Volume of metal left
4.2 cm. The rest of the cylinder is melted = 176.4 p – 98.784 p
and converted into a cylindrical wire of
1.4 cm thickness. Find the length of the = 77.616 p cm2
Given Radius of wire = 0.7 cm
é 22 ù
wire. ê Use π = ú let length of wire = x
ë 7û
\ Volume of wire = p × 0.7 × 0.7 × x
Sol. Given Radius of hemisphere = 4.2 cm = r
= 0.49px cm3
2 3
\ Volume of hemisphere = pr Since the rest of the cylinder in melted and
3 converted into cylinderical wire.

2 Þ 0.49px = 77.616 p
= p ´ (4.2)3 cm3 Þ x = 158.4 cm
3
\ Required length of wire = 158.4 cm
= 49.392 p cm3
SET-II [UNCOMMON QUESTIONS TO SET-I]
10. If A (4, 3), B (– 1, y) and C (3, 4) are the 18. All the vertices of a rhombus lie on a
vertices of right triangle ABC, right- circle. Find the area of the rhombus, If
angled at A, then find the value of y. the area of the circle is 1256 cm2.
Sol. \ BC2 = (3 + 1)2 + (4 – y)2 = 16 + (4 – y)2 [Use p = 3.14]
AB2 = (– 1 – 4)2 + (y – 3)2 = 25 + (y – 3)2 Sol. Diagonal of a rhombus are perpendicular
bisector of each other.
and AC2 = (4 – 3)2 + (3 – 4)2 = 2
\ Each diagonal of rhombus is diameter of
Also BC2 = AB2 + AC2
the circle.
[using pythagoras theorem]
Þ 16 + (4 – y)2 = 25 + (y – 3)2 + 2
Þ 16 + 16 + y2 – 8y = 25 + y2 – 6y + 9 + 2
Þ – 2y = 4 C(3, 4)
Þ y=–2

A(4, 3) B(–1, y) Now area of circle = 1256 cm2


https://www.arundeepselfhelp.info/index.php?route=product/product&path=180&product_id=395

Arundeep’s Solved Papers 82 Mathematics 2015 (Outside Delhi)


Þ pr2 = 1256 Þ 41d = 164 Þ d = 4
1256 When d = 4, eq. (i) becomes
Þ r2 =
p 5
a = ´4 Þa= 5
1256 4
Þ r2 = = 400 = 202
3.14 n
Þ r = 20 cm [we know that Sn = {2a + (n – 1) d}]
2
\ Diameter of the circle = 40 cm
\ Each diagonal of the rhombus = 40 cm 15
\ S15 = (2a + 14d )
Thus Area of rhombus 2
1 1
= d ´ d = ´ 40 ´ 40 = 800 cm2 =
15
(2 ´ 5 + 14 ´ 4)
2 1 2 2 2
19. Solve for x : 2 x 2 + 6 3 x - 60 = 0
15
= ´ 66 = 15 × 33 = 495
Sol. Given eqn. be, 2 x2 + 6 3x - 60 = 0 2
Þ 2 x 2 + 10 3x - 4 3x - 60 = 0 28. A bus travels at a certain average speed
for a distance of 75 km and then travels
Þ 2 x ( x + 5 3) - 4 3 ( x + 5 3) = 0 a distance of 90 km at an average speed
of 10 km/h more than the first speed. If
Þ ( x + 5 3) (2 x - 4 3) = 0 it takes 3 hours to complete the total
journey, find its first speed.
x + 5 3 = 0 or 2 x - 4 3 = 0 Sol. Let average speed of the bus = x km/h
Distance to be covered by bus = 75 km
x = - 5 3 or x =
4 3
= 2 3. _ Time taken by bus to covered a distance of
2
75
20. The 16th term of an AP is five times its 75 km by bus = hr
x
third term. If its 10th term is 41, then
find the sum of its first fifteen terms. Speed of bus for a distance of 90 km
= (x + 10) km/h
Sol. Let the 1st term of the AP = a
\ Time taken by bus to covered a distance of
Common difference = d
90
A.T.Q. ; we have 90 km by bus = hr
x + 10
a16 = 5 × a3
Þ a + 15d = 5 (a + 2d) [_ an = a + (n – 1) d] 75 90
A.T.Q., + =3
Þ a + 15d = 5a + 10d x x + 10

5 75 ( x + 10) + 90 x
Þ 5d = 4a Þ a = d ...(i) Þ x ( x + 10)
=3
4
Also a10 = 41 Þ a + 9d = 41 Þ 75x + 750 + 90x = 3x2 + 30x
5 Þ 3x2 – 135x – 750 = 0
Þ d + 9d = 41 (Using eqn. (i))
4 Þ x2 – 45x – 250 = 0
https://www.arundeepselfhelp.info/index.php?route=product/product&path=180&product_id=395

Arundeep’s Solved Papers 83 Mathematics 2015 (Outside Delhi)


Þ x2 – 50x + 5x – 250 = 0 Sol. Steps of Construction :
Þ x(x – 50) + 5(x – 50) = 0 Draw a line segment BC = 8 cm
Þ (x – 50) (x + 5) = 0 From B draw an angle of 90o

Þ x = 50, x = – 5 Draw an arc BA = 6cm cutting the angle


at A.
\ Required speed of bus = 50 km/h
Join AC.
29. Prove that the tangent at any point of a DABC is the required D.
circle is perpendicular to the radius Draw ^ bisector of BC cutting BC at M.
through the point of contact.
Take M as centre and BM as radius, draw
Sol. Proof : We are given a circle with centre O a circle.
and a tangent XY to the circle at a point P. Take A as centre and AB as radius draw
We need to prove that OP is perpendicular an arc
to XY. cutting the circle at E. Join AE.
Take a point Q on XY other than P and join AB and AE are the required tangents.
OQ.
The point Q must lie outside the circle. X
(Note that if Q lies inside the circle, XY
A
will become a secant and not a tangent to
the circle). Therefore, OQ is longer than
6 cm

the radius OP of the circle. That is, D E


90º
90º

OQ > OP.
90
º

90º
B C
8 cm M O

Since this happens for every point on the


line XY except the point P. OP is the
shortest of all the distances of the point O Justification :
to the points of XY. So OP is perpendicular
ÐABC = 90° (Given)
to XY.
Since, OB is a radius of the circle.
30. Construct a right triangle ABC with
\ AB is a tangent to the circle.
AB = 6 cm, BC = 8 cm and ÐB = 90º.
Also AE is a tangent to the circle.
Draw BD, the perpendicular from B on
AC. Draw the circle through B, C and D 31. Find the values of k so that the area of
and construct the tangents from A to this the triangle with vertices (k + 1, 1),
(4, – 3) and (7, – k) is 6 sq. units.
circle.
https://www.arundeepselfhelp.info/index.php?route=product/product&path=180&product_id=395

Arundeep’s Solved Papers 84 Mathematics 2015 (Outside Delhi)

1
Sol. \ Area of D = | x (y – y ) + x2 (y3 – y1) + x3(y1 – y2) |
2 1 2 3
Here x1 = k + 1 ; y1 = 1 ; x2 = 4 ; y2 = –3 ; x3 = 7 and y3 = –k
1
Þ | (k + 1) (– 3 + k) + 4 (– k – 1) + 7 (1 + 3) | = 6
2
Þ | – 3k – 3 + k2 + k – 4k – 4 + 28 | = 12 Þ | k2 – 6k + 21 | = 12
Þ k2 – 6k + 21 = ± 12
Þ k2 – 6k + 21 = 12 or k2 – 6k + 21 = – 12
k2 – 6k + 9 = 0 or k2 – 6k + 33 = 0
Þ (k – 3)2 = 0 Here, D = (– 6)2 – 4 × 1 × 33
k =3 = 36 – 132 = – 96 < 0
\ k=3 \ Given quadratic eqn. has no solution.

SET-III [UNCOMMON QUESTIONS TO SET-I and Set-II]

10. Solve the following quadratic equation the sum of its first ten terms.
for x : Sol. Let 1st term of the AP = a
x2 – 2ax – (4b2 – a2) = 0 Common difference = d
Sol. Given quadratic eqn. be A.T.Q. we have
x2 – 2ax – (4b2 – a2) = 0 a13 = 4 × a3
on comparing with ax2 + bc + c = 0, we Þ a + 12d = 4 (a + 2d)
have [_ an = a + (n – 1)d]
a = 1, b = –2a ; c = –(4b2 – a2) Þ a + 12d = 4a + 8d Þ 3a = 4d
\ Distriminant D = b2 – 4ac
4
i.e. D = (– 2a)2 – 4 × [– (4b2 – a2)] Þ a = d ...(i)
3
= 4a2 + 16b2 – 4a2
Also a5 = 16 Þ a + 4d = 16
= 16b2
4
By quadratic formula, we have Þ d + 4d = 16 [Using (i)]
3
-b ± D Þ 16d = 48 Þ d = 3
\ x=
2a When d = 3, (i) becomes ;
4
- (-2a) ± 16b 2 a = ´3= 4 Þ a = 4
\ x = 3
2 ´1
10
2a ± 4b Now, S10 = (2a + 9d )
= = a ± 2b 2
2
é n ù
êQ Sn = 2 {2a + (n - 1)d }ú
18. The 13th term of an AP is four times its
3rd term. If its fifth term is 16, then find ë û
https://www.arundeepselfhelp.info/index.php?route=product/product&path=180&product_id=395

Arundeep’s Solved Papers 85 Mathematics 2015 (Outside Delhi)


= 5 (2 × 4 + 9 × 3) Now, P (R) + P (B) + P (W) = 1
= 5 × 35 = 175.
2 3
19. Find the coordinates of a point P on the Þ P (R) + + =1
5 10
line segment joining A (1, 2) and B (6, 7)
2 2 3
such that AP = AB. Þ P (R) = 1 - -
5 5 10
Sol. 10 - 4 - 3 3
= =
10 10
2:3
(1, 2) (x, y) (6,7) Let total number of balls = x
A 2 P 3 B 20
\ P (B) =
2 x
Given, AP = AB
5 A.T.Q. ; we have
2 20 2
Þ AP == (AP + PB) = Þ x = 50
5 x 5
Þ 5AP = 2AP + 2PB \ Total no. of balls = 50
Þ 3AP = 2PB 28. A truck covers a distance of 150 km at a
AP 2 certain average speed and then covers
Þ = another 200 km at an average speed
PB 3
which is 20 km per hour more than the
\ AP : PB = 2 : 3
first speed. If the truck covers the total
Thus, P divides AB in the ratio 2 : 3 distance in 5 hours, find the first speed
2 ´ 6 + 3 ´1 of the truck.
\ x coordinate of P = =3
2+3 Sol. Let average speed of truck = x km/hr
Distance covered by truck = 150 km
2 ´ 7 + 3´ 2
and y coordinate of P = =4 \ Time taken by truck to covered a distance
2+3
\ Coordinates of P are (3, 4). of 150 km =
150
hrs
20. A bag contains, white, black and red balls x
only. A ball is drawn at random from the A.T.Q., Speed to cover 200 km = (x + 20) km/hr
bag. If the probability of getting a white
\ Time taken by truck to covered a distance
3 2
ball is and that of a black ball is , 200
10 5 of 200 km = hr
x + 20
then find the probability of getting a red
ball. If the bag contains 20 black balls, Given Total time taken = 5 hrs
then find the total number of balls in the
150 200
bag. Þ + =5
x x + 20
Sol. Let R = getting a red ball
B = getting a black ball 150 ( x + 20) + 200 x
Þ x ( x + 20)
=5
W = getting a white ball
https://www.arundeepselfhelp.info/index.php?route=product/product&path=180&product_id=395

Arundeep’s Solved Papers 86 Mathematics 2015 (Outside Delhi)


Þ 150x + 3000 + 200x = 5 (x2 + 20x) Þ 350x + 3000 = 5x2 + 100x
Þ 5x2 – 250x – 3000 = 0 Þ x2 – 50x – 600 = 0
Þ x2 – 60x + 10x – 600 = 0 Þ x (x – 60) + 10 (x – 60) = 0
Þ (x – 60) (x + 10) = 0 Þ x = 60 or x = – 10
\ Average speed of truck = 60 km/hr.
29. An arithmetic progression 5, 12, 19, ... has 50 terms. Find its last term. Hence find the sum
of its last 15 terms.
Sol. Let a be the first term and d be the common difference of given A.P.
\ a = 5, d = 7, n = 50
\ a50 = a + 49d = 5 + 49 × 7 = 348 [_ an = a + (n – 1)d]

é n ù
êQ Sn = 2 ( a + an )ú
50
and S50 = ( a + a50 ) = 25 (5 + 348) = 8825
2 ë û
\ Sum of last 15 terms = S50 – S35

35
Also, S35 = (2 ´ 5 + 34 ´ 7) = 4340
2
\ Sum of last fifteen terms = 8825 – 4340 = 4485.
30. Construct a triangle ABC in which AB = 5 cm, BC = 6 cm and ÐABC = 60º. Now construct
5
another triangle whose sides are times the corresponding sides of DABC.
7
Sol. Steps of construction :
(i) Draw a line segment BC = 6 cm.
(ii) Draw a ray BX making an angle of 60º and cut off BA = 5 cm.
(iii) Join AC. Then ABC is the required triangle. X

(iv) Draw a ray BY making an acute angle with BC and cut off A
7 equal parts making BB1 = B1B2 = B2B3 = B3B4 = B4B5 =
B5B6 = B6B7 A¢

(v) Join B7 and C


5 cm
(vi) Draw B5C¢ parallel to B7C and C¢A¢ parallel to CA.
Then DA¢BC¢ is the required triangle.
60º C
B
6 cm C¢
B1
B2
B3
B4
B5
B6
B7
Y
https://www.arundeepselfhelp.info/index.php?route=product/product&path=180&product_id=395

Arundeep’s Solved Papers 87 Mathematics 2015 (Outside Delhi)

31. Find the values of k for which the points A (k + 1, 2k), B (3k, 2k + 3) and C (5k – 1, 5k) are
collinear.
Sol. If points are collinear then area of triangle having given points are vertices be 0.
\ x1 (y2 – y3) + x2 (y3 – y1) + x3 (y1 – y2) = 0
Þ (k + 1) (2k + 3 – 5k) + 3k (5k – 2k) + (5k – 1) (2k – 2k – 3) = 0
Þ (k + 1) (– 3k + 3) + 3k × 3k + (5k – 1) (– 3) = 0
Þ – 3k2 + 3k – 3k + 3 + 9k2 – 15k + 3 = 0
Þ 6k2 – 15k + 6 = 0 Þ 3 (2k2 – 5k + 2) = 0
Þ 2k2 – 5k + 2 = 0
Þ 2k2 – 4k – k + 2 = 0
Þ 2k (k – 2) – 1 (k – 2) = 0 Þ (k – 2) (2k – 1) = 0
1
Þ k = 2 or k = .
2

1
Þ k = 2, .
2
https://www.arundeepselfhelp.info/index.php?route=product/product&path=180&product_id=395

MATHEMATICS 2015 TERM II (DELHI)


SET I
Time allowed : 3 hours Maximum marks : 90

General Instructions :
(i) All questions are compulsory.
(ii) The question paper consists of 31 questions divided into four sections – A, B, C and D.
(iii) Section A contains 4 questions of 1 mark each, Section-B contains 6 questions of 2 marks each,
Section-C contains 10 questions of 3 marks each and Section-D contains 11 questions of 4
marks each.
(iv) Use of calculators is not permitted.

1 12 - 3
1. If x = - , is a solution of the quadratic Þ –k =
2 4
equation 3x2 + 2kx – 3 = 0, find the value
of k. 9
Þ –k =
Sol. p (x) = 3x2 + 2kx – 3 4
1 9
Q x =- is a solution of p (x) Þ k =-
2 4
2. The tops of two towers of height x and y,
æ 1ö
\ p ç- ÷ = 0 standing on level ground, subtend angles
è 2ø of 30º and 60º respectively at the centre
2 of the line joining their feet, then find
æ 1ö
p ç - ÷ = 3 æç - ö÷ + 2k æç - ö÷ - 3
1 1
x : y.
è 2ø è 2ø è 2ø
Sol. Let AB and DC are towers of height x and
y respectively
æ1ö æ -1 ö
= 3 ç ÷ + 2k ç ÷ - 3 In right angled DABE, we have
è4ø è 2ø
AB
3 = tan 30°
= -k -3 BE
4
x
æ 1ö Þ = tan 30º
Q p çè - 2 ÷ø = 0 a

x 1
3 Þ =
\ -k-3 =0 a 3
4
a
3 3 Þ x =
–k = - 3
1 4

Arundeep’s Solved Papers 88 Mathematics 2015 (Delhi)


https://www.arundeepselfhelp.info/index.php?route=product/product&path=180&product_id=395

Arundeep’s Solved Papers 89 Mathematics 2015 (Delhi)


In right angled DCDE, we have Sol. Since we know that tangents drawn from a
point exterior to circle are of equal length
DC y
Þ = tan 60° Þ = tan 60º \ PA = PB
EC a Since equal sides have equal angle opposite
y to it
Þ = 3 Þy =a 3
a \ ÐPAB = ÐABP
Since the sum of all three angles of DPAB
D
is 180°.
A
\ 2ÐPAB = 180° – 50° = 130°
y Þ ÐPAB = 65°
Since radius of circle is ^ to the tangent
30° 60° to circle at their point of contact.
B C
E
\ OA ^ PA Þ ÐOAP = 90°
x a Thus ÐOAB = ÐOAP – ÐPAB
a 3 = 90° – 65°= 25°
Now, =
y a 3 5. In Fig. given below AB is the diameter
a of a circle with centre O and AT is a
tangent. If ÐAOQ = 58º, find ÐATQ.
x a a 1
Þ a´ y = ´
3 a 3
x 1
Þ y = Thus x : y = 1 : 3
3
3. A letter of English alphabet is chosen at
random. Determine the probability that
the chosen letter is a consonant.
Sol. Total numbers (letters) of English alphabet Sol. ÐAOQ = 2ÐABQ
= 26 [ _ angle made by arc at centre of circle is
No. of consonant letters = 21 double the angle made by same arc at any
21 point on circumference of circle]
\ P (getting consonant letter) =
26 1
Þ ÐABQ = ´ 58º = 29º
4. In Fig. given below PA and PB are 2
tangents to the circle with centre O such In DABT, we have
that ÐAPB = 50º. Write the measure of ÐBAT + ÐABT + ÐATB = 180º
ÐOAB.
Þ 90º + 29º + ÐATB = 180º
Þ ÐATB = 61º
as ÐATB = ÐATQ
Þ ÐATQ = 61º
6. Solve the following quadratic equation
for x :
4x2 – 4a2x + (a4 – b4) = 0
Sol. Given, quadratic eqn. be
https://www.arundeepselfhelp.info/index.php?route=product/product&path=180&product_id=395

Arundeep’s Solved Papers 90 Mathematics 2015 (Delhi)


4x2
– 4a2x + (a4
– b4)
=0 Þ OR ^ PR [line joining mid point of a chord
2
on comparing with ax + bc + c = 0, we to the centre of a circle is perpendicular to
have the chord]
a = 4, b = – 4a2, c = (a4 – b4) \ OT is the perpendicular bisector of PQ.
\ D = b2 – 4ac 8. Find the middle term of the A.P.
= (– 4a2)2 – 4(4) (a4 – b4) 6, 13, 20, ..., 216
= 16a4 – 16 (a4 – b4) Sol. Given A.P. be, 6, 13, 20, ... 216
= 16a4 – 16a4 + 16b4 = 16b4 Here a = 6, d = 7
by quadratic formula, we have Let the given A.P. contains n terms
then Tn = 216
- b ± D - (- 4a 2 ) ± 16b4
\ x = = We know that
2a (2) (4)
Tn = a + (n – 1) d
4a 2 ± 4b 2 4 ( a 2 ± b2 ) Þ 216 = 6 + (n – 1) 7
= = Þ 216 = 6 + 7n – 7
8 8
Þ 216 = 7n – 1
a 2 ± b2 Þ
= 216 + 1 = 7n
2
Þ 217 = 7n
7. From a point T outside a circle of centre
O, tangents TP and TQ are drawn to the 217
Þ =n
circle. Prove that OT is the right bisector 7
of line segment PQ. i.e. 31 = n
Sol. In DTPO and DTQO, we have i.e. n = 31
TP = TQ Thus, the given A.P. contains 31 terms.
OP = OQ (radii)
1
OT = OT \ Its middle term = (31 + 1)
2
\ DTPO @ DTQO
[by SSS axiom of congruency] 1
= (32) = 16th term
\ Ð1 = Ð2 2
P \ T16 = 6 + (16 – 1)7
= 6 + (15) (7) = 6 + 105 = 111
1
O 2 R T Hence, middle term = 111
9. If A (5, 2), B (2, – 2) and C (– 2, t) are the
Q vertices of a right angled triangle with
ÐB = 90º, then find the value of t.
In DOPR and DOQR,
OP = OQ (radii) Sol. AB = (5 - 2) 2 + (2 - ( -2))2
OR = OR
Ð1 = Ð2 [Proved] = (3) 2 + (4) 2 = 9 + 16 = 25
\ DOPR @ DOQR
BC = (2 - ( -2)) 2 + (-2 - t )2
Þ PR = QR
https://www.arundeepselfhelp.info/index.php?route=product/product&path=180&product_id=395

Arundeep’s Solved Papers 91 Mathematics 2015 (Delhi)

= (2 + 2)2 + (t + 2) 2 æ1 3ö
line segment joining the point A ç , ÷
è2 2ø
= (4) 2 + t 2 + 4 + 4t and B(2, –5) in the ratio k : 1 internally.
Then by section formula,
= 16 + t 2 + 4 + 4t
æ1ö
3 k (2) + 1 ç ÷
= t2 + 4t + 20 è2ø
4 = k +1

k 1
æ1 3ö æ3 5 ö B(2, –5)
Aç , ÷ Pç , ÷
è2 2ø è 4 12 ø

1
2k +
2 = 4k + 1
3
and AC = (5 + 2)2 + (2 - t ) 2 =
4 k +1 2 ( k + 1)
= (7) 2 + 4 + t 2 - 4t Þ 3 (2) (k + 1) = 4 (4k + 1)
Þ 6 (k + 1) = 4 (4k + 1)
= 49 + 4 + t 2 - 4t Þ 6k + 6 = 16k + 4
= t 2 - 4t + 53 Þ 6k – 16k = 4 – 6 Þ – 10k = – 2
by using Pythagoras Theorem, we have 2 1
i.e. k = =
(AC)2 = (AB)2 + (BC)2 10 5
t2 – 4t + 53 = 25 + t2 + 4t + 20 Thus, required ratio be k : 1
Þ – 4t – 4t = 45 – 53 1
i.e. :1 i.e. 1 : 5.
Þ – 8t = – 8 5
8 11. Find the area of the triangle ABC with
i.e. t = =1 A (1, – 4) and mid-points of sides through
8
A being (2, – 1) and (0, – 1).
i.e. t = 1 Sol. Let P(2, –1) be the mid point of AB and
10. Find the ratio in which the point Q(0, –1) be the mid point of AC. Let
æ3 5 ö (x 1 , y 1 ), (x 2 , y 2 ) be the coordinates of
P ç , ÷ divides the line segment vertices B and C of DABC.
è 4 12 ø
Since P(2, –1) be the mid point of AB.
æ1 3ö x1 + 1
joining the points A ç , ÷ and
è2 2ø \ =2
2
B (2, – 5).
Þ x1 + 1 = 4 Þ x1 = 4 – 1 = 3
æ3 5 ö y1 + ( - 4)
Sol. Let the given point P ç , ÷ divides the
è 4 12 ø and
2
=–1
https://www.arundeepselfhelp.info/index.php?route=product/product&path=180&product_id=395

Arundeep’s Solved Papers 92 Mathematics 2015 (Delhi)


Þ y1 – 4 = – 2 Sol. Given quadratic eqn. be,
Þ y1 = – 2 + 4 Þ kx2 + 1 – 2 (k – 1) x + x2 = 0
i.e. y1 = 2 Þ kx2 + x2 – 2 (k – 1) x + 1 = 0
Also Q(0, –1) be the mid point of AC. (k + 1) x2 – 2 (k – 1) x + 1 = 0
x2 + 1 on comparing with ax2 + bx + c = 0, we
\ =0 have
2
x2 + 1 = 0 a = k + 1, b = – 2 (k – 1), c = 1
Þ x2 = – 1 Here, D = b2 – 4ac
= [– 2 (k – 1)]2 – 4 (k + 1) (1)
y2 + ( - 4)
and =–1 = 4 (k – 1)2 – 4 (k + 1)
2
Q Roots are equal
y2 - 4 \ D=0
Þ =–1
2 \ 4 (k – 1)2 – 4 (k + 1) = 0
Þ y2 – 4 = – 2 4 [(k – 1)2 – (k + 1)] = 0
i.e. y2 = – 2 + 4 = 2 Þ (k – 1)2 – k – 1 = 0
i.e. y2 = 2 Þ k2 + 1 – 2k – k – 1 = 0
Þ k2 – 3k = 0
Þ k (k – 3) = 0
i.e. k = 0 or k – 3 = 0
i.e. k = 0 or k = 3
\ Non zero value of k = 3
After putting the value of k = 3 in given
equation, we get
\ Vertices of triangle ABC are 3x2 + 1 – 2 (3 – 1) x + x2 = 0
A (1, – 4), B (3, 2) and C (– 1, 2) Þ 3x2 + 1 – 2 (2) x + x2 = 0
1 Þ 3x2 + 1 – 4x + x2 = 0
\ Area of DABC = | x (y – y3) + Þ 4x2 – 4x + 1 = 0
2 1 2
x2 (y3 – y1) + x3 (y1 – y2) | Þ 4x2 – 2x – 2x + 1 = 0
Þ 2x (2x – 1) – 1 (2x – 1) = 0
1
= | 1 (2 – 2) + 3 (2 + 4) + (– 1) (– 4 – 2) | Þ (2x – 1) (2x – 1) = 0
2
Þ either 2x – 1 = 0 2x – 1 = 0
1
= | 1 (0) + 3 (6) + (– 1) (– 6) | i.e. 2x = 1 or 2x = 1
2
1 1
1 1 i.e. x = or x =
= | 18 + 6 | = | 24 | = 12 sq. unit 2 2
2 2
1 1
12. Find that non-zero value of k, for which \ Required roots are , .
the quadratic equation 2 2
kx2 + 1 – 2 (k – 1) x + x2 = 0 has equal 13. The angle of elevation of the top of a
roots. Hence find the roots of the equation. building from the foot of the tower is 30º
https://www.arundeepselfhelp.info/index.php?route=product/product&path=180&product_id=395

Arundeep’s Solved Papers 93 Mathematics 2015 (Delhi)


and the angle of elevation of the top of (2, 1), (2, 2) ......(2, 6)
the tower from the foot of the building is (3, 1), (3, 2) .... (3, 6)
45º. If the tower is 30 m high, find the (4, 1), (4, 2) .... (4, 6)
height of the building. (5, 1), (5, 2) ..... (5, 6)
Sol. Let DC be the building is of height be h m (6, 1), (6, 2) ...... (6, 6)}
AB (i) Let E1 be the event of getting two numbers
In right DABC, = tan 45º whose sum is 5. Then favourable outcomes
BC
are (1, 4), (2, 3), (3, 2), (4, 1)
30
Þ =1 4 1
BC \ P (E1) = =
36 9
Þ BC = 30
(ii) Let E2 be the event of getting even number
on both dice, then favourable outcomes are
(2, 2), (2, 4), (2, 6), (4, 2), (4, 4), (4, 6),
(6, 2), (6, 4), (6, 6)
Number of fevourable outcomes = 9
9 1
\ P (E2) = =
36 4
15. If Sn, denotes the sum of first n terms of
In right DACD, we have an A.P., prove that S12 = 3 (S8 – S4).
CD Sol. Let first term of an A.P. = a
= tan 30º Let common difference = d
BC
Then sum of nth term is
h 1
Þ = n
30 3 Sn = [2a + (n - 1) d ]
2
30
Þ h = 12
3 S12 = [2a + (12 - 1) d ]
2
30 3 30 3 = 6 [2a + 11d] = 12a + 66d
Þ h = ´ = = 10 3
3 3 3 8
S8 = [2a + (8 – 1) d] = 4 [2a + 7d]
2
\ h = 10 3 m
= 8a + 28d
Thus required height of building DC
4
= 10 3 m S4 = [2a + (4 – 1) d] = 2 (2a + 3d)
2
14. Two different dice are rolled together. = 4a + 6d
Find the probability of getting : \ RHS = 3 (S8 – S4) = 3 [(8a + 28d) – (4a + 6d)]
(i) the sum of numbers on two dice to be 5. = 3 [8a + 28d – 4a – 6d] = 3 [4a + 22d]
(ii) even numbers on both dice. = 12a + 66d = LHS
Sol. When two dice are thrown simultaneously, Thus, LHS = RHS
all possible outcomes = 36
16. In Fig. given below APB and AQO are
Since, sample space = {(1, 1), (1, 2) .... (1, 6) semicircle, and AO = OB. If the
https://www.arundeepselfhelp.info/index.php?route=product/product&path=180&product_id=395

Arundeep’s Solved Papers 94 Mathematics 2015 (Delhi)


perimeter of the figure is 40 cm, find the 80 ´ 7
i.e. r = = 7 cm
é 22 ù 80
area of the shaded region. ê Use π = ú
ë 7û
pr 2
Now, Area of APB =
2
22 ´ 7 ´ 7
= = 77 cm 2
7´2

pr 2
Area of AQO =
2
P
22 7 7 1 77
Sol. Let AO = OB = r = ´ ´ ´ = cm 2
7 2 2 2 4
Then perimeter of semicircle APB
77
2pr \ Area of shaded region = 77 +
= = pr 4
2
308 + 77 385
r = =
2p 4 4
Perimeter of semicircle AQO = 2 = pr 2
= 96.25 cm
2 2
17. In Fig. given below From the top of a
æ rö solid cone of a height 12 cm and base
çQ radius of semicircle AQO = ÷ radius 6 cm, a cone of height 4 cm is
è 2ø
removed by a plane parallel to the base.
\ Perimeter of shaded region Find the total surface area of the
pr remaining solid.
= pr + +r
2 æ 22 ö
ç Use π = and 5 = 2.236 ÷
2pr + pr + 2r è 7 ø
=
2
But Perimeter is 40 cm. (given)
2pr + pr + 2r
\ = 40
2
Þ r (2p + p + 2) = 80
Þ r (3p + 2) = 80

æ 22 ö
Þ r ç3 ´ + 2 ÷ = 80 Sol. Let radius of the upper face = BE = x
è 7 ø
DABE ~ DACD
æ 66 ö
Þ rç + 2 ÷ = 80 AB BE
è 7 ø Þ =
AC CD
æ 80 ö 4 x
Þ r ç ÷ = 80 Þ = Þ x = 2 cm
è 7ø 12 6
https://www.arundeepselfhelp.info/index.php?route=product/product&path=180&product_id=395

Arundeep’s Solved Papers 95 Mathematics 2015 (Delhi)


Remaining solid is a frustum of cone with painting the hemispherical part of the
height = 12 – 4 = 8 cm = h toy at the rate of `10 per cm2.
r1 = 2 cm, r2 = 6 cm
é 22 ù
Let Slant height = l êë Use π = 7 úû

\ l = h2 + ( r2 - r1 ) 2 Sol.

l = 64 + 16
Þ l = 80

Þ l = 4 5 cm

Let height of the cone = h cm


B
E Radius of the cone = 3.5 cm
Radius of the hemisphere = 3.5 cm

C 5 3
D
Given volume of toy = 166 cm
6
1 2 2
\ Volume of the toy = pr h + pr 3
Now, total surface area of frustum of cone 3 3
(remaining solid)
5 1 22 2 22
= pl (r1 + r2 ) + pr12 + pr22 Þ 166 = ´ (3.5) 2 ´ h + ´ ´ (3.5)3
6 3 7 3 7
= p [l (r1 + r2 ) + r12 + r22 ] 1001 269.5 1886.5
Þ = h+
22 6 21 21
= [4 5 (2 + 6) + 4 + 36]
7 1001 1886.5 269.5
Þ - = h
22 6 21 21
= ´ [32 5 + 40]
7 7007 - 3773 269.5
Þ = h
22 42 21
= [32 ´ 2.236 + 40] cm2
7
3234 ´ 21
= 350.592 cm2 Þ = h Þ h = 6 cm
42 ´ 269.5
18. A solid wooden toy is in the form of a
hemisphere surmounted by a cone of \ Height of toy = 6 + 3.5 = 9.5 cm
same radius. The radius of hemisphere Thus, curved surface area of hemispherical
is 3.5 cm and the total wood used in the part = 2pr2
5 22
making of toy is 166 cm3 . Find the = 2´ ´ 3.5 ´ 3.5
6 7
height of the toy. Also, find the cost of = 77 cm2
https://www.arundeepselfhelp.info/index.php?route=product/product&path=180&product_id=395

Arundeep’s Solved Papers 96 Mathematics 2015 (Delhi)


Thus, cost of painting at the rate of ` 10
per cm2 = 77 × 10 = ` 770
19. In Fig. given below from a cuboidal solid
metallic block, of dimensions 15 cm × 10
cm × 5 cm, a cylindrical hole of diameter
7 cm is drilled out. Find the surface area
é 22 ù
of the remaining block. ê Use π = ú
ë 7û

Sol. Here PQ = (14 – 6) cm = 8 cm


8
\ OP = OQ = cm = 4 cm
2
and XY = (14 – 6) cm = 8 cm
each side of square EFGH = 4 cm
\ Area of unshaded region = Area of 4
semicircles of radius 2cm + Area of square
of side 4 cm

pr 2
= 4´ + side × side
2
7
Sol. Given radius of cylindrical hole = r = cm
2
D C
and height of cylindrical hole = height of
cuboid = h = 5 cm
\ Surface area of remaining block = total H G
surface area of cuboid X Y
O
+ Curved surface area of cylinder E F

– Surface area of two circular end of Q


cylindrical hole
A 14 cm B
= 2 [lb + bh + lh] + 2prh – 2pr2
4 ´ 3.14 ´ 2 ´ 2
= 2 [15 × 10 + 10 × 5 + 15 × 5] = + 42
2
22 7 22 7 7 = 25.12 + 16 = 41.12 cm2
+2× ´ ´5- 2´ ´ ´
7 2 7 2 2 Area of square of side 14 cm = 14 × 14
= 2 [(150 + 50 + 75) + 110 – 77] cm2 = 196 cm2
\ Area of shaded region = (196 – 41.12) cm2
= 616 cm2
= 154.88 cm2
20. In Fig. given below find the area of the 21. The numerator of a fraction is 3 less than
shaded region. [Use p = 3.14]
its denominator. If 2 is added to both the
https://www.arundeepselfhelp.info/index.php?route=product/product&path=180&product_id=395

Arundeep’s Solved Papers 97 Mathematics 2015 (Delhi)


numerator and the denominator, then daughter to school after 12 weeks.
the sum of the new fraction and original What value is generated in the above
29 situation ?
fraction is . Find the original Sol. Ramkali saving per week forms a following
20
sequence 100, 120, 140, 160...
fraction.
which clearly forms A.P.
Sol. Let denominator of fraction = x
Whose first term (a) = 100 and
Then, numerator = x – 3
common difference (d) = 20
x-3 Then, we know that,
Then, fraction be
x n
Sn = [2a + (n - 1) d ]
x -1 x - 3 29 2
ATQ, + =
x+2 x 20 12
\ S12 = [2 (100) + (12 - 1) 20]
x ( x - 1) + ( x + 2) ( x - 3) 29 2
Þ =
( x + 2) ( x ) 20 = 6 [200 + 220] = 6 [420] = 2520
Yes, she will be concern for education of
x 2 - x + x 2 - 3x + 2 x - 6 29 girl child.
Þ =
x2 + 2 x 20 23. Solve for x :
2 3 23
2 x2 - 2 x - 6 29 + = , x ¹ 0, – 1, 2
Þ = x + 1 2 ( x - 2) 5 x
x + 2x
2 20
Þ 20 (2x – 2x – 6) = 29 (x2 + 2x)
2 2 3 23
Sol. Given eqn. be, + =
Þ 40x2 – 40x – 120 = 29x2 + 58x x + 1 2 ( x - 2) 5x
Þ 40x2 – 40x – 120 – 29x2 – 58x = 0
4 ( x - 2) + 3 ( x + 1) 23
Þ 11x2 – 98x – 120 = 0 Þ =
2 ( x + 1) ( x - 2) 5x
Þ 11x2 – 110x + 12x – 120 = 0
Þ 11x (x – 10) + 12 (x – 10) = 0 4 x - 8 + 3x + 3 23
either (x – 10) (11x + 12) = 0 Þ 2 ( x 2 - 2 x + x - 2) =
5x
x – 10 = 0 or 11x + 12 = 0
7x - 5 23
-12 Þ 2 ( x 2 - x - 2) =
x = 10 or x = 5x
11
\ x = 10 Þ 5x (7x – 5) = 46 (x2 – x – 2)
[_ x be a natural number] Þ 35x2 – 25x = 46x2 – 46x – 92
Hence, required fraction is given by Þ 35x2 – 25x – 46x2 + 46x + 92 = 0
x - 3 10 - 3 7 Þ – 11x2 + 21x + 92 = 0
= =
x 10 10 Þ 11x2 – 21x – 92 = 0
22. Ramkali required ` 2500 after 12 weeks Þ 11x2 – 44x + 23x – 92 = 0
to send her daughter to school. She saved Þ 11x (x – 4) + 23 (x – 4) = 0
` 100 in the first week and increased her
Þ (x – 4) (11x + 23) = 0
weekly saving by ` 20 every week. Find
whether she will be able to send her either x – 4 = 0 or 11x + 23 = 0
https://www.arundeepselfhelp.info/index.php?route=product/product&path=180&product_id=395

Arundeep’s Solved Papers 98 Mathematics 2015 (Delhi)


\ x = 4 or 11x = – 23 Sol. In DRQP, QP = RP

-23
Thus, x = ,4
11
24. Prove that the tangent at any point of a
circle is perpendicular to the radius
through the point of contact.
Sol. Proof : We are given a circle with centre O
and a tangent XY to the circle at a point P.
We need to prove that OP is perpendicular
to XY. \ Ð3 = Ð4
Take a point Q on XY other than P and join Now, Ð3 + Ð4 + 30º = 180º
OQ. Þ 2Ð3 = 150º Þ Ð3 = 75º
The point Q must lie outside the circle. Now, ÐQOR + 90° + 90° + 30° = 360°
(Note that if Q lies inside the circle, XY Þ ÐQOR = 150°
will become a secant and not a tangent to
the circle). Therefore, OQ is longer than 1
\ ÐQSR = Ð1 = ÐQOR = 75°
the radius OP of the circle. That is, 2
OQ > OP. Also SR || QP
Since this happens for every point on the \ Ð1 = Ð2 [Alternate interior angles]
line XY except the point P. OP is the Þ Ð2 = 75º
shortest of all the distances of the point O Now, Ð2 + ÐRQS + Ð3 = 180º
to the points of XY. So OP is perpendicular
Þ ÐRQS = 180º – 150º = 30º
to XY.
26. Construct a triangle ABC with BC = 7 cm,
ÐB = 60º and AB = 6 cm. Construct
3
O another triangle whose sides are times
4
the corresponding sides of DABC.
R Sol. Steps of Construction
A P Q B (i) Draw a line BC = 7 cm
(ii) Draw angle 60º at B. Cut AB = 6 cm
25. In Fig. given below tangents PQ and PR
are drawn from an external point P to a (iii) Join AC.
circle with centre O, such that ÐRPQ = Thus, DABC is obtained.
30º. A chord RS is drawn parallel to the (iv) Below BC, make an acute ÐCBX.
tangent PQ. Find ÐRQS. (v) Along BX, mark off four points B1, B2, B3,
B4 such that BB1 = B1B2 = B2B3 = B3B4.
(vi) Join B4C.
(vii) From B3, draw B3D || B4C, meeting BC at
D.
(viii) From D, draw DE || CA, meeting AB at E.
Then, DEBD is the required triangle, each
https://www.arundeepselfhelp.info/index.php?route=product/product&path=180&product_id=395

Arundeep’s Solved Papers 99 Mathematics 2015 (Delhi)

3 \ PB = h 3 ...(i)
of whose sides is of the corresponding side of
4 In right angled DABP, we have
DABC.
AB
= tan 60º
PB

Þ h+5 = 3
PB

h+5
Þ = PB
3

h+5
or PB = ...(ii)
3
From (i) and (ii) ; we have
h+5
h 3 = 3
Þ 3h = h + 5
27. From a point P on the ground the angle Þ 3h – h = 5
of elevation of the top of a tower is 30º Þ 2h = 5
and that of the top of a flag staff fixed
on the top of the tower, is 60º. If the 5
i.e. h = = 2.5 m
length of the flag staff is 5 m, find the 2
height of the tower. Thus required height of tower be 2.5 m.
Sol. Let height of the tower BC be h 28. A box contains 20 cards numbered from
and AC be the flag staff s.t AC = 5 m 1 to 20. A card is drawn at random from
In DPBC, we have the box. Find the probability that the
BC number on the drawn card is
= tan 30º (i) divisible by 2 or 3
PB
(ii) a prime number.
h 1
Þ = Sol. Total number of all possible outcomes = 20
PB 3 (i) Let E1 be the event of getting card divisible
by 2 or 3. Then number of favourable
outcomes = 13 {2, 3, 4, 6, 8, 9, 10, 12, 14,
15, 16, 18, 20}
13
\ P (E1) =
20
h (ii) Let E2 be the event of getting card a prime
number.
Then number of favourable outcomes = 8
{2, 3, 5, 7, 11, 13, 17, 19}
Þ h 3 = PB
https://www.arundeepselfhelp.info/index.php?route=product/product&path=180&product_id=395

Arundeep’s Solved Papers 100 Mathematics 2015 (Delhi)

8 2 35 + 109 144
\ P (E2) = = = = = 72 sq. units
20 5 2 2
29. If A (– 4, 8), B (– 3, – 4), C (0, – 5) and D 30. A well of diameter 4 m is dug 14 m deep.
(5, 6) are the vertices of a quadrilateral The earth taken out is spread evenly all
ABCD, find its area. around the well to form a 40 cm high
Sol. Area of quadrilateral ABCD = Area of embankment. Find the width of the
DABC + Area of DACD embankment.
1 4
We know, Area of D = | x (y – y ) + x2 Sol. Given radius of the well = R = =2m
2 1 2 3 2
(y3 – y1) + x3 (y1 – y2) | and Depth of the well = h = 14 m
1 \ Volume of the earth dug out = pR2h
Now, Area of DABC = | – 4 (– 4 + 5) + = p (2)2 × 14 = p × 4 × 14 = 56 p m2
2
(– 3) (– 5 – 8) + 0 (8 + 4) | Let the width of the embankment be r m.
Clearly, embankment forms a cylindrical
1 1 shell whose inner and outer radii are 2 m
= | – 4 (1) + (– 3) (– 13) | = | – 4 + 39 |
2 2 and (2 + r) m respectively and height
35 40
= sq. unit m
2 100
\ Volume of embankment
1
Now, Area of DACD = | – 4 (– 5 – 6) + 0
2 40
= p [(2 + r)2 – (2)2] ×
(6 – 8) + 5 (8 + 5) | 100

40
= p [(4 + r2 + 4r – 4] ×
100

2pr
= ( r + 4)
5

1
= | – 4 (– 11) + 0 + 5 (13) | rm
2 2
1 1
= | 44 + 65 | = (109)
2 2
109
= sq. unit
2
quarding to given condition, we have
35 109
\ Area of quadrilateral ABCD = + Volume of embankment = Volume of the
2 2
https://www.arundeepselfhelp.info/index.php?route=product/product&path=180&product_id=395

Arundeep’s Solved Papers 101 Mathematics 2015 (Delhi)


earth dug out
2pr (r + 4) 56p ´ 5
Þ = 56p Þ r (r + 4) = = 140
5 2p
Þ r2 + 4r = 140 Þ r2 + 4r – 140 = 0
Þ r2 + 14r – 10r – 140 = 0 Þ r (r + 14) – 10 (r + 14) = 0
Þ (r + 14) (r – 10) = 0
either r + 14 = 0 or r – 10 = 0
i.e. r = – 14 or r = 10
But r ¹ – 14, Since width can’t be negative
\ r = 10 m
Thus required width of embankment be 10 metre.
31. Water is flowing at the rate of 2.52 km/h through a cylindrical pipe into a cylindrical tank,
the radius of whose base is 40 cm, If the increase in the level of water in the tank, in half an
hour is 3.15 m, find the internal diameter of the pipe.
Sol. Let internal radius of the pipe = x cm
Speed of water = 2.52 km/h = 2520 m/h

1 2
\ Volume of water that flows in half an hour = pr h
2

1 x x
= p´ ´ ´ 2520
2 100 100

126px 2 3
= m
1000
Given radius of cylindrical tank = R = 40 cm
40 40
\ Volume of water in cylindrical tank = p ´ ´ ´ 3.15 m3
100 100
according to given condition, we have volume of water that flows in half an hour = volume of
water in cylindrical tank

126px 2 40 40
Þ = p´ ´ ´ 3.15
1000 100 100

40 40 1000
Þ x2 = ´ ´ 3.15 ´
100 100 126
Þ x2 = 4 Þ x = 2 cm
\ Internal diameter of the pipe = 2x = 4 cm
https://www.arundeepselfhelp.info/index.php?route=product/product&path=180&product_id=395

Arundeep’s Solved Papers 102 Mathematics 2015 (Delhi)

SET-II [UNCOMMON QUESTIONS TO SET-I]

10. Find the middle term of the A.P. 213, 205, \ First term (a) = 5
197, ..., 37.
1
Sol. Given A.P be, 213, 205, 197, ...., 37. Where n = 2 ; S2 = [3 (2)2 + 7 (2)]
2
Let a be the first term and d be the common
difference of given A.P 1
= [3 (4) + 14]
213, 205, 197, ..., 37 2
a = 213 ; d = – 8
1 1
and an = 37 = (12 + 14) = (26) = 13
2 2
We know that an = a + (n – 1) d \ 2nd term = S2 – S1 = 13 – 5 = 8
Þ 37 = 213 + (n – 1) (– 8)
1
Þ 37 = 213 – 8n + 8 Where n = 3 ; S3 = [3 (3)2 + 7 (3)]
2
Þ 37 = 221 – 8n
Þ 37 – 221 = – 8n =
1
[3 (9) + 21]
i.e. – 184 = – 8n 2
184 1 1
i.e. =n = (27 + 21) = (48) = 24
8 2 2
Thus, 23 = n \ 3rd term S3 – S2 = 24 – 13 = 11
\ n = 23 Hence required A.P. is 5, 8, 11...
1 \ a = 5, d = 3
Now, Middle term = (23 + 1)
2 and nth term
1 = an = a + (n – 1) d
= (24) = 12th
2 = 5 + (n – 1) 3 = 5 + 3n – 3
Now, a12 = 213 + (12 – 1) (– 8) = 3n + 2
= 213 + (11) (– 8) Thus, a20 = 3 (20) + 2 = 60 + 2 = 62
= 213 – 88 = 125 19. Three distinct coins are tossed together.
18. If the sum of the first n terms of an A.P. Find the probability of getting.
1 (i) at least 2 heads
is (3n 2 + 7n), then find its nth term.
2 (ii) at most 2 heads
Hence write its 20th term. Sol. When 3 coins are tossed simultaneously, all
possible outcomes are
1
Sol. Given, Sn = (3n2 + 7n) {HHH, HHT, HTH, THH, HTT, THT,
2
TTH, TTT}
1 Total number of possible outcomes = 8
Where n = 1 ; S1 = [3 (1)2 + 7 (1)]
2 (i) Let E1 be the event of getting at least 2 head.
Then, favourable outcomes are HHT, HTH,
1 1
= (3 + 7) = (10) = 5 THH, HHH
2 2
https://www.arundeepselfhelp.info/index.php?route=product/product&path=180&product_id=395

Arundeep’s Solved Papers 103 Mathematics 2015 (Delhi)


Number of favourable outcomes = 4 p = 3 or p=–1
4 \ p¹–1 [Given]
\ P (Getting at least 2 heads) = P (E1) = [_ a ¹ 0 Þ p + 1 ¹ 0 Þ p ¹ –1]
8
\ p=3
1
= \ equation (i) becomes ;
2
(ii) Let E2 be the event of getting at most 2 4x2 – 24x + 36 = 0
heads. Þ 4(x2 – 6x + 9) = 0
Then, E2 = event of getting 0 or 1 or 2 heads Þ x2 – 6x + 9 = 0
So, the favourable outcomes are Þ (x – 3)2 = 0 Þ x = 3.
TTT, HTT, THT, TTH, HHT, HTH, THH 28. To fill a swimming pool two pipes are to
Number of favourable outcomes = 7 be used. If the pipe of larger diameter is
\ P (Getting at most 2 heads) = P (E2) used for 4 hours and the pipe of smaller
diameter for 9 hours, only half the pool
7
= can be filled. Find, how long it would take
8 for each pipe to fill the pool separately,
20. Find that value of p for which the if the pipe of smaller diameter takes 10
quadratic equation hours more than the pipe of the larger
(p + 1) x2 – 6 (p + 1) x + 3 (p + 9) = 0, diameter to fill the pool.
p ¹ – 1 has equal roots. Hence find the Sol. Let time taken by the pipe of larger diameter
roots of the equation. = x hr
Sol. Given quadratic eqn. be \ Time taken by the pipe of smaller diameter
(p + 1) x2 – 6 (p + 1) x + 3 (p + 9) = 0 ...(i) = (x + 10) hr
On comparing with ax2 + bx + c = 0, a ¹ 0
4 ´1 9 1
we have a = p + 1 ; b = –6 (p + 1) and ATQ + =
C = 3 (p + 9) x x + 10 2
\ Roots are equal \ D = 0
4 x + 40 + 9 x 1 13 x + 40 1
Now, D = b2 – 4ac Þ = Þ 2 =
x ( x + 10) 2 x + 10 x 2
D = [– 6 (p + 1)]2 – 4 (p + 1) 3 (p + 9)
= [36 (p2 + 1 + 2p) – 12 (p + 1) (p + 9)] Þ 26x + 80 = x2 + 10x
= 36p2 + 72p + 36 – 12 (p2 + 10p + 9) Þ x2 – 16x – 80 = 0
= 36p2 + 72p + 36 – 12p2 – 120p – 108 Þ x2 – 20x + 4x – 80 = 0
= 24p2 – 48p – 72 Þ x (x – 20) + 4(x – 20) = 0
Q D=0 Þ (x – 20) (x + 4) = 0
\ 24p2 – 48p – 72 = 0 Þ x = 20 or x = – 4 (rejected)
Þ 24 (p2 – 2p – 3) = 0 \ Time taken by larger pipe = 20 hr
Þ p2 – 2p – 3 = 0 and Time taken by smaller pipe = 20 + 10 = 30 hr
Þ p2 – 3p + p – 3 = 0 30. Construct an isosceles triangle whose
Þ p (p – 3) + 1 (p – 3) = 0 base is 6 cm and altitude 4 cm. Then
construct another triangle whose sides
Þ (p – 3) (p + 1) = 0
3
either p – 3 = 0 or p + 1 = 0 are times the corresponding sides of
4
https://www.arundeepselfhelp.info/index.php?route=product/product&path=180&product_id=395

Arundeep’s Solved Papers 104 Mathematics 2015 (Delhi)


the isosceles triangle. X
Sol. Steps of construction :
(i) Draw a line segment AB = 6 cm and draw its ^ bisector
DX and cut off DC = 4 cm
(ii) Join AC and BC to get the required DABC
(iii) draw a ray AB making on acute angle with AB and cut
off 4 equal parts making AA1 = A1A2 = A2A3 = A3A4
(iv) Join A4 B
(v) From A3, draw A3 B¢ || to A4B meeting AB at B¢
(vi) Draw B¢C¢ || to BC meeting AC at C¢
Then AB¢C¢ is the required triangle.

31. If P (– 5, – 3), Q (– 4, – 6), R (2, – 3) and S (1, 2) are the vertices of a quadrilateral PQRS,
find its area.
1
Sol. \ Area of DPQR = | x (y – y ) + x2 (y3 – y1) + x3 (y1 – y2) |
2 1 2 3
1
= | – 5 (– 6 + 3) + (– 4) (– 3 + 3) + 2 (– 3 + 6) |
2 S(1, 2) R(2, –3)
1 21
= | 15 + 6 | = sq. unit
2 2
1
Area of DPRS = | – 5 (– 3 – 2) + 2 (2 + 3) + 1 (– 3 + 3) |
2
1 35
= | 25 + 10 | = sq. units P(–5, –3) Q(–4, –6)
2 2

Area of quadrilateral PQRS = area of DPQR + area of DPRS


21 35 56
= + = = 28 sq. units.
2 2 2

SET-III [UNCOMMON QUESTIONS TO SET-I and Set-II]


10. Solve the following quadratic equation for x :
9x2 – 6b2x – (a4 – b4) = 0
Sol. Given quadratic eqn. be,
9x2 – 6b2x – (a4 – b4) = 0
On comparing with ax2 + bx + c = 0, where a ¹ 0
https://www.arundeepselfhelp.info/index.php?route=product/product&path=180&product_id=395

Arundeep’s Solved Papers 105 Mathematics 2015 (Delhi)


a = 9, b = – 6b2, c = – (a4 – b4) 13
\ Discriminant D = b2 – 4ac (iii) Probability (club card) =
46
\ D = (– 6b2)2 – 4 × 9 [– (a4 – b4)] 19. Find the area of the triangle PQR with
= 36b4 + 36a4 – 36b4 Q (3, 2) and the mid-points of the sides
= 36a4 through Q being (2, – 1) and (1, 2).
By quadratic formula, we have Sol. Q M is the mid point of PQ.

-b ± D x1 + 3
Thus x = \ =2
2a 2
Þ x1 + 3 = 4
- (-6b 2 ) ± 36a 4 Þ x1 = 4 – 3 = 1
\ x =
2´9
y1 + 2
6b 2 ± 6a2 Also, =–1
= 2
18 y1 + 2 = – 2
b2 ± a 2 y1 = – 2 – 2 = – 4
=
3 Q N is the mid point of QR.
18. All red face cards are removed from a x3 + 3
pack of playing cards. The remaining \ =1
2
cards were well shuffled and then a card
is drawn at random from them. Find the Þ x3 + 3 = 2
probability that the drawn card is Þ x3 = 2 – 3
(i) a red card Þ x3 = – 1
(ii) a face card
y3 + 2
(iii) a card of clubs Also, =2
2
Sol. Total number of cards = 52
Number of red face cards = 6 Þ y3 + 2 = 4
[2 red kings, 2 red queens and 2 red jacks] Þ y3 = 4 – 2 Þ y3 = 2
\ Number of remaining cards = 52 – 6 = 46

20 10
(i) Probability (a red card) = =
46 23
(Since 6 red cards have already been
removed)

6 3
(ii) Probability (face card) = =
46 23
[as it is given that 6 red face cards are Hence, vertices of DPQR are ;
removed] P (1, – 4), Q (3, 2) and R (–1, 2)
https://www.arundeepselfhelp.info/index.php?route=product/product&path=180&product_id=395

Arundeep’s Solved Papers 106 Mathematics 2015 (Delhi)


We know that
1
Area of DPQR = | x (y – y ) + x2 (y3 – y1) + x3 (y1 – y2) |
2 1 2 3

1
= | 1 (2 – 2) + 3 (2 + 4) – 1 (– 4 – 2) |
2

1 1 1
= | 1 (0) + 3 (6) + 6 | = | 18 + 6 | = | 24 | = 12 sq. units
2 2 2
20. If Sn denotes the sum of first n terms of an A.P., prove that S30 = 3 [S20 – S10]
Sol. Let first term of A.P. = a and Common difference of A.P. = d
n
_ Sn = [2a + (n - 1) d ]
2

30
Now, S30 = [2a + (30 – 1) d]
2
= 15 [2a + 29d] = 30a + 435d
20
S20 = [2a + (20 - 1) d ] = 10 [2a + 19d] = 20a + 190d
2
10
S10 = [2a + (10 – 1) d] = 5 [2a + 9d] = 10a + 45d
2
R.H.S. = 3 (S20 – S10) = 3 [20a + 190d – (10a + 45d)]
= 3 [20a + 190d – 10a – 45d] = 3 [10a + 145d] = 30a + 435d = S30 = L.H.S.
Thus, LHS = RHS
28. A 21 m deep well with diameter 6 m is dug and the earth from digging is evenly spread to
é 22 ù
form a platform 27 m × 11 m. Find the height of the platform. êë Use π = 7 úû

Sol. Given, radius of well (r) = 3 m


and Depth of well (h) = 21 m
\ Volume of earth dug out = pr2h
22
= ´ 9 ´ 21 = 594 m2
7
Let the required height of platform = h
Length of platform = l = 27 m
Breadth of platform = b = 11 m
\ Volume of platform = 27 × 11 × h
Now, Volume of platform = Volume of earth dug out = 27 × 11 × h = 594
https://www.arundeepselfhelp.info/index.php?route=product/product&path=180&product_id=395

Arundeep’s Solved Papers 107 Mathematics 2015 (Delhi)

594 Then AS, AT, BS¢ and BT¢ are the required
Þ h = = 2m tangent.
27 ´ 11
Here the required height of platform be 2
metre S¢
S
29. A bag contains 25 cards numbered from
1 to 25. A card is drawn at random from
the bag. Find the probability that the
3 cm 2 cm
number on the drawn card is : A M 7 cm B

(i) divisible by 3 or 5
(ii) a perfect square number
T
Sol. Total number of all possible outcomes = 25 T¢
(i) Out of the given numbers, divisible by 3 or
5 are
31. Solve for x :
3, 5, 6, 9, 10, 12, 15, 18, 20, 21, 24, 25
3 4 29 1
Then the number of favourable outcomes = 12 + = ; x ¹ 1, - 1,
x + 1 x - 1 4x - 1 4
\ Required probability (that drawn card is
3 4 29
12 Sol. Given eqn. be, + =
divisible by 3 or 5) = x +1 x -1 4x - 1
25
3( x - 1) + 4 (x + 1) 29
(ii) Out of the given numbers, perfect square Þ =
are 1, 4, 9, 16 and 25. ( x + 1) ( x - 1) 4x - 1
\ Probability (a card drawn having no is a 3x - 3 + 4 x + 4 29
Þ =
5 1 x -1
2 4x - 1
perfect square) = =
25 5 7x + 1 29
Þ =
30. Draw a line segment AB of length 7 cm. x - 1 4x - 1
2
Taking A as centre, draw a circle of Þ (7x + 1) (4x – 1) = 29 (x2 – 1)
radius 3 cm and taking B as centre, draw Þ 28x2 – 7x + 4x – 1 = 29x2 – 29
another circle of radius 2 cm. Construct
tangents to each circle from the centre Þ 28x2 – 3x – 1 = 29x2 – 29
of the other circle. Þ 28x2 – 3x – 1 – 29x2 + 29 = 0
Sol. Steps of construction : Þ – x2 – 3x + 28 = 0
(i) Draw a line segment AB = 7 cm. Þ – (x2 + 3x – 28) = 0
(ii) With centre A and radius 3 cm and with centre
Þ x2 + 3x – 28 = 0
B and radius 2 cm, circles are drawn.
(iii) Bisect AB at M. Þ x2 + 7x – 4x – 28 = 0
(iv) With centre M and diameter AB, draw a circle Þ x (x + 4) – 4 (x + 7) = 0
which intersects the two circles at S¢, T¢ and Þ (x + 7) (x – 4) = 0
S, T respectively. Þ x+7=0 or x – 4 = 0
(v) Join AS, AT, BS¢ and BT¢. Þ x=–7 or x = 4
https://www.arundeepselfhelp.info/index.php?route=product/product&path=180&product_id=395

MATHEMATICS -2016 TERM I

Time allowed : 3 hours Maximum marks : 90

1. In DABC, D and E are points AC and ÐA + ÐC 180º -ÐB


=
BC respectively such that DE || AB. If 2 2
AD = 2x, BE = 2x – 1, CD = x + 1 and
ÐA + ÐC ÐB
CE = x – 1, then find the value of x. [1] = 90º -
2 2
Sol.
æ ÐA + ÐC ö æ ÐB ö
\ sin ç ÷ = sin ç 90º - ÷
è 2 ø è 2 ø

æ ÐA + ÐC ö ÐB
Þ sin ç ÷ = cos
è 2 ø 2

(A + C) B
Thus, sin = cos Hence proved.
2 2
3. If x = 3 sin q and y = 4 cos q, find the
DE || AB
AD BE
value of 16 x 2 + 9 y 2 . [1]
So, = [By B.P.T.]
CD EC Sol. x = 3 sin q
Þ x2 = 9 sin2 q
2x 2x - 1
=
x +1 x -1 x2
sin2 q = ...(i)
Þ 2x (x – 1) = (x + 1) (2x – 1) 9
Þ 2x2 – 2x = 2x2 + 2x – x – 1 and y = 4 cos q
Þ – 2x = x – 1 Þ y2 = 16 cos2 q
Þ 1 = 3x y2
\ cos2 q = ...(ii)
1 16
or x = Ans. On adding eq. (i) and eq. (ii) ; we have
3
2. In A, B and C are interior angles of x2 y 2
DABC, then prove that : sin2 q + cos2 q = +
9 16
(A + C) B
sin = cos . [1] x2 y 2
2 2 Þ 1 = +
Sol. In DABC 9 16
ÐA + ÐB + ÐC = 180º 16 x 2 + 9 y 2
ÐA + ÐC = 180º – ÐB Þ 1 =
144
Divide by 2 on both sides
i.e. 16x + 9y2 = 144
2

Arundeep’s Solved Papers 108 Mathematics 2016 (Term I)


https://www.arundeepselfhelp.info/index.php?route=product/product&path=180&product_id=395

Arundeep’s Solved Papers 109 Mathematics 2016 (Term I)

Þ 16 x 2 + 9 y 2 = 520521
144 Þ 52.0521 =
10000
\ 16 x 2 + 9 y 2 = 12 Ans. 2 10000
4. If empirical relationship between mean, 2 5000
median and mode is expressed as mean 2 2500
= k (3 median – mode), then find the 2 1250
value of k. [1]
5 625
Sol. Given, mean = k (3 median – mode)
5 125
As we know, mode = 3 median – 2 mean
\ mean = k [3 median – (3 median – 2 mean)] 5 25
mean = k [3 median – 3 median + 2 mean] 5
mean = 2k mean Yes, it is rational number.
Þ 2k mean – mean = 0 where q = 10000 = 24 × 54
i.e. mean [2k – 1] = 0 The given decimal expression is a
2k – 1 = 0 [since mean ¹ 0] terminating decimal as the factors of q
consist only 2 and 5. Ans.
Þ 2k = 0 + 1
7. Given the linear equation x – 2y – 6 = 0,
i.e. k = 1/2 Ans.
write another linear equation in these
SECTION – B two variables, such that the geometrical
representation of the pair so formed is :
5. Express 23150 as product of its prime
factors. Is it unique ? [2] (i) coincident lines
Sol. Prime factor of 23150 = 2 × 5 × 5 × 463 (ii) intersection lines [2]
As per the fundamental theorem of Sol. (i) Given, x – 2y – 6 = 0
Arithmetic every number has a unique For line to be coincident
factorisation.
a1 b1 c1
= =
2 23150 a2 b2 c2
5 11575
Thus one possible option will be
5 2315
Ans. 2x – 4y – 12 = 0
463 463
Here, a1 = 1, b1 = – 2, c1 = – 6
1
a2 = 2, b2 = – 4, c2 = – 12
6. State whether the real number 52.0521 a1 1 b1 - 2 1 c1 -6 1
is rational or not. If it is rational express = ; = = ; = =
a2 2 b2 - 4 2 c2 - 12 2
p
it in the form , where p, q are co-
q a1 b1 c1
prime, integers and q ¹ 0. What can you Þ a =b =c
2 2 2
say about prime factorisation of q ? [2]
and hence both lines are coincident lines.
Sol. Ans.
https://www.arundeepselfhelp.info/index.php?route=product/product&path=180&product_id=395

Arundeep’s Solved Papers 110 Mathematics 2016 (Term I)


(ii) Given, x – 2y – 6 = 0 2
é1 - tan A ù
Sol. L.H.S. = ê
ë1 - cot A úû
For intersecting lines

a1 b1
¹
a2 b2 2
é sin A ù
Thus, one possible option will be, ê1 - cos A ú
=ê ú
ê1 - cos A ú
2x – 7y – 13 = 0
êë sin A úû
Here, a1 = 1, b1 = – 2, c1 = – 6
a2 = 2, b2 = – 7, c2 = – 13
2
é cos A - sin A ù
a1 1 b1 - 2 2 ê ú
Here, a = 2 ; b = - 7 = 7 =ê cos A
ú
2 2 ê sin A - cos A ú
êë sin A úû
a1 b1
Þ a ¹b
2 2 2
é (cos A - sin A) sin A ù
=ê ú
ë - (cos A - sin A) cos A û
So, both lines representing intersecting
lines. Ans.
8. In an isosceles DABC right angled at B, 2
prove that AC2 = 2AB2. [2] é sin A ù
= ê-
Sol. In an isosceles DABC, AB = BC ...(i) ë cos A úû

[Q triangle is isosceles] = [– tan A]2

A = tan2 A = R.H.S. Hence Proved.

10. Given below is a cumulative frequency


distribution table. Corresponding to it,
make an ordinary frequency distribution
table. [2]

B C
x cf
In DABC by pythagoras theorem,
More than or equal to 0 45
AC2 = AB2 + BC2
More than or equal to 10 38
Þ AC2 = AB2 + AB2 [From (i)]
More than or equal to 20 29
i.e. AC2 = 2AB2 Hence Proved.
More than or equal to 30 17
9. Prove the following identity :
More than or equal to 40 11
2
é 1 - tan A ù More than or equal to 50 6
êë 1 - cot A úû = tan A : ÐA is acute. [2]
2
https://www.arundeepselfhelp.info/index.php?route=product/product&path=180&product_id=395

Arundeep’s Solved Papers 111 Mathematics 2016 (Term I)


Sol. Table of values is given as under : and g (x) = 2x + 1. [3]
Sol. Given, f (x) = 6x3+ 13x2
+x–2
C.I. Frequency and g (x) = 2x + 1,
By division algorithm, we have
0 – 10 07 (45 – 38)
10 – 20 09 (38 – 29) 2x + 1 ) 6x3 + 13x2 + x – 2 (3x2 + 5x – 2
20 – 30 12 (29 – 17) 6x3 + 3x 2
30 – 40 6(17 – 11) – –
10x2 + x – 2
40 – 50 5 (11 – 6) 10x2 + 5 x
50 – 60 – –
– 4x – 2
– 4x – 2
SECTION – C + +
11. Find LCM and HCF of 3930 and 1800 0
by prime factorisation method. [3]
quotient = 3x2 + 5x – 2, remainder = 0 Ans.
Sol. By prime factorization method,
13. If three zeroes of a polynomial
Factors of 3930 and 1800 are,
x4 – x3 – 3x2 + 3x are 0, 3 and - 3,
then find the fourth zero. [3]
2 1800
2 900 Sol. Let P (x) = x4 – x3 – 3x2 + 3x
2 3930 2 450 Given, 0, 3, - 3 are three zeroes, so
3 1965 3 225
5 655 3 75 x, x – 3 and x + 3 are factors of f(x)
131 131 5 25 [If a be the root of f(x) = 0
1 5 5 Then x – a be a factor of f(x).]
1
Here, x ( x + 3) ( x - 3) will also be the
factor of P (x).
So, 3930 = 2 × 3 × 5 × 131
1800 = 2 × 2 × 2 × 3 × 3 × 3 × 5 × 5 Or, x (x2 – 3) will be the factor of P (x).

then, HCF = product of the smallest power then by long division, we have
of each common prime factor in given
numbers = 2 × 3 × 5 = 30 x3 – 3x) x4 – x3 – 3x2 + 3x (x – 1
and, LCM = product of the greatest power x4 – 3x2
of each prime factor involved in given – +
numbers = 2 × 3 × 5 × 131 × 2 × 2 × 3 × 5 – x3 + 3x
– x3 + 3x
= 235800 Ans.
+ –
12. Using division algorithm, find the 0
quotient and remainder on dividing f (x)
by g (x) where f (x) = 6x3 + 13x2 + x – 2 \ quotient = (x – 1)
https://www.arundeepselfhelp.info/index.php?route=product/product&path=180&product_id=395

Arundeep’s Solved Papers 112 Mathematics 2016 (Term I)

( )(
Thus P(x) = q(x) (x) x - 3 x + 3 + 0 ) So u = 1 Þ
1
y
=1 Þ y = 1
Thus zeroes of P(x) are given by putting
P(x) = 0 1 1
So fourth zero be given by x – 1 = 0 and u = 6 Þ = 6, Þ x =
x 6
Þx=1
1
Hence four zeroes will be 1, 0, 3, - 3. Hence, x = and y = 1 Ans.
6
Ans.
15. DABC is a right angled triangle in which
14. Solve the following pair of equations by ÐB = 90º. D and E are any point on AB
reducing them to a pair of linear and BC respectively. Prove that
equations : [3]
AE2 + CD2 = AC2 + DE2. [3]
1 4
- =2 Sol. In DABC, ÐB = 60º and D, E are point of
x y AB, BC respectively.

1 3
+ =9
x y

1 4
Sol. Given, equations are ; - =2
x y

1 3
+ =9
x y
To prove :
1 1
Let = u, = u AC2 + DE2 = AE2 + CD2
x y
In DABC by using Pythagoras theorem,
Given equations reduces to,
AC2 = AB2 + BC2 ...(i)
So, u – 4u = 2 ...(i)
In DABE by using Pythagoras theorem
u + 3u = 9 ...(ii)
On solving eq. (i) and eq. (ii) ; we have AE2 = AB2 + BE2 ...(ii)
u – 4u = 2 In DBCD by Pythagoras theorem
u + 3u = 9 CD2 = BD2 + BC2 ...(iii)
– – – In DDBE by Pythagoras theorem
– 7u = – 7 DE2 = DB2 + BE2 ...(iv)
Þ u=1 Adding eq. (i) and eq. (iv)
Putting the value of u in eq. (i) ; we get AC2 + DE2 = AB2 + BC2 + BD2 + BE2
Þ u – 4u = 2
= AB2 + BE2 + BC2 + BD2
Þ u–4×1=2
AC2 + DE2 = AE2 + CD2
i.e. u–4=2
i.e. u=2+4 [From eq. (ii) and eq. (iii)]
Þ u=6 Hence Proved.
https://www.arundeepselfhelp.info/index.php?route=product/product&path=180&product_id=395

Arundeep’s Solved Papers 113 Mathematics 2016 (Term I)


16. In the given figure, RQ and TP are Thus their corresponding sides are
perpendicular to PQ, also TS ^ PR prove proportional.
that ST.RQ = PS.PQ. [3]
ST PS
= [By c.p.c.t]
QP RQ
Þ ST.RQ = PS.PQ Hence Proved.
2
17. If sec A = , find the value of
3

tan A 1 + sin A
+ [3]
cos A tan A
2
Sol. Given, sec A =
Sol. 3

In DABC, we have
AC2 = AB2 + BC2
In DRPQ Þ 22 = ( 3)2 + BC2
Ð1 + Ð2 + Ð4 = 180º Þ 4 = 3 + BC2
Ð1 + Ð2 + 90º = 180º Þ BC2 = 4 – 3
Ð1 + Ð2 = 180º – 90º Þ BC2 = 1
Ð1 = 90º – Ð2 ...(i) Þ BC = 1
Q TP ^ PQ 1 1
3
\ ÐTPQ = 90º So, tan A = ; cos A = ; sin A =
3 2 2
Þ Ð2 + Ð3 = 90º
Ð3 = 90º – Ð2 ...(ii) 1 1
1+
From eq. (i) and eq. (ii) tan A 1 + sin A = 3 + 2
\ + 1
Ð1 = Ð3 cos A tan A 3
Now in DRQP and DPST, we have 2 3
Ð1 = Ð3 [Proved above]
3
Ð4 = Ð5 [Each 90º]
= + 2 = 2 + 3 3 = 4 + 9 3 Ans.
2
So by AA axiom of similarity 3 1 3 2 6
DRQP ~ DPST 3
https://www.arundeepselfhelp.info/index.php?route=product/product&path=180&product_id=395

Arundeep’s Solved Papers 114 Mathematics 2016 (Term I)


18. Prove that : [3]
sec2 q – cot2 (90º – q) = cos2 (90º – q) + cos2 q.
Sol. To prove :
sec2 q – cot2 (90º – q) = cos2 (90º – q) + cos2 q
L.H.S. = sec2 q – cot2 (90º – q)
= sec2 q – [cot (90 – q)]2
= sec2 q – (tan q)2 [_ cot (90° – q) = tan q]
= sec q – tan q
2 2 [_ sec2 q = 1 + tan2 q]
=1
R.H.S. = cos2 (90° – q) + cos2 q
= [cos (90° – q)]2 + cos2 q
= (sin q)2 + cos2 q [_ cos (90° – q) = sin q]
= sin q + cos q
2 2

=1
Hence, L.H.S. = R.H.S. Hence Proved
19. For the month of February, a class teacher of Class IX has the following absentee record
for 45 students. Find the mean number of days, a student was absent.

Number of days of absent 0–4 4–8 8 – 12 12 – 16 16 – 20 20 – 24


Number of students 18 3 6 2 0 1
[3]
Sol. The table of values is given as under :

C.I. fi xi (mid-value) di = xi – A fi × d i
0–4 18 2 – 12 – 216
4–8 3 6 –8 – 24
8 – 12 6 10 –4 – 24
12 – 16 2 A = 14 0 00
16 – 20 0 18 4 00
20 – 24 1 22 8 08
Sfi = 30 Sfidi = – 256

Sfi di æ - 256 ö
Mean = A + = 14 + ç ÷ = 14 – 8.53 = 5.47 Ans.
Sfi è 30 ø
20. Find the missing frequency (x) of the following distribution, if mode is 34.5 :

Marks obtained 0 – 10 10 – 20 20 – 30 30 – 40 40 – 50
Number of students 4 8 10 x 8
[3]
https://www.arundeepselfhelp.info/index.php?route=product/product&path=180&product_id=395

Arundeep’s Solved Papers 115 Mathematics 2016 (Term I)


Sol. C.I. Frequency
0 – 10 4 Here max frequency = 10 which lies in 20 – 30
10 – 20 8 = f0 \ Modal class be 20 – 30
20 – 30 10 = f1 \ l = 20, f1 = 10 ; f0 = 8 ; f2 = x ; h = 10
30 – 40 x = f2
40 – 50 8

æ f1 - f0 ö
\ Mode = l + ç ÷h
è 2 f1 - f0 - f 2 ø

æ 10 - 8 ö æ 2 ö
Þ 34.5 = 20 + ç ÷ 10 Þ 34.5 = 20 + ç ÷ 10
è 20 - 8 - x ø è 12 - x ø

14.5 20 20
Þ = Þ 20 = 14.5 (12 – x) Þ = 12 – x
1 12 - x 14.5
40 40 348 - 40
Þ = 12 – x Þ x = 12 - Þ x =
29 29 29
308
Þ x =
29
Þ x = 10.62 Ans.
SECTION D
21. Prove that 5 is an irrational number.. Þ 5 is factor of q2
Hence show that 3 + 2 5 is also an Þ 5 is a factor of q.
irrational number. [4] Here 5 is a common factor of p, q which
contradicts the fact that p, q are co-prime.
Sol. Let 5 be a rational number..
Hence our assumption is wrong, 5 is an
p irrational number.
So, 5 =
q Now we have to show that 3 + 2 5 is an
On squaring both sides ; we have irrational number. So let us assume
p2 3 + 2 5 is a rational number..
5 =
q2
p
Þ 3+2 5 = ; where p, q Î I and q ¹ 0
Þ 5q2 = p2 q
Þ 5 is a factor of p2 and (p, q) = 1
Þ 5 is a factor of p. p
Now, again let p = 5c. Þ 2 5 = -3
q
Þ 5q2 = 25c2
p - 3q
Þ q2 = 5c2 Þ 2 5 = q
https://www.arundeepselfhelp.info/index.php?route=product/product&path=180&product_id=395

Arundeep’s Solved Papers 116 Mathematics 2016 (Term I)

p - 3q
Þ 5 =
2q

p - 3q p
2q
is in the rational form of q so 5 is a rational number but we have already proved that

5 is an irrational number so contradiction arises. Here our supposition is wrong that 3 + 2 5


is a rational number. So we can say that 3 + 2 5 is an irrational number.. Hence Proved.
22. Obtain all other zeroes or the polynomial x4 + 6x3 + x2 – 24x – 20, if two of its zeroes are
+ 2 and – 5. [4]
Sol. Given, 2 – 5 are the zeroes of polynomial
p (x) = x4 + 6x3 + x2 – 24x – 20 By long division method, we have
So (x – 2) and (x + 5) are factors of p (x)
Þ (x – 2) (x + 5) is also a factor of p (x) x2 + 3x – 10) x4 + 6x 3 + x 2 – 24x – 20(x2 + 3x + 2
So (x – 2) (x + 5) = x2 + 3x – 10 x4 +3x 3 – 10x2
– – +
So, by Euclid algorithm, we have
3x2 + 11 x2 – 24 x – 20
Dividend = Divisor × Quotient + Remainder
3x3 + 9x 2 – 30x
4 3 2 2 2
x + 6x + x – 24x – 20 = (x + 3x – 10) × (x + 3x + 2) + 0 – – +
= (x2 + 3x – 10) (x2 + 2x + x + 2) 2x2 + 6x – 20
= (x2 + 3x – 10) [x (x + 2) + 1 (x + 2)] 2x2 + 6x – 20
– – +
= (x2 + 3x – 10) (x + 2) (x + 1)
0
So other zeroes are – 2 and – 1. Ans.
[We know that if x = a be a root of f(x) = 0
Then x – a be a factor of f(x)]
23. Draw graph of following pair of linear equations :
y = 2 (x – 1)
4x + y = 4
Also write the coordinate of the points where these lines meets x-axis and y-axis. [4]
Sol. Given line be, y = 2 (x – 1) ...(1)
So,

x 1 2 3 4
y 0 2 4 6

Ploting all points (1, 0), (2, 2), (3, 4) on graph paper and join them by straight line to get the
graph for line (1).
and for, 4x + y = 4
or y = 4 – 4x ...(2)
https://www.arundeepselfhelp.info/index.php?route=product/product&path=180&product_id=395

Arundeep’s Solved Papers 117 Mathematics 2016 (Term I)

x 1 2 1/2
y 0 –4 2

Ploting all the points (1, 0), (2, –4) and (½, 2) on graph paper and join them by line to get a graph
for eqn. (2).

Y
7

)
6

–1
2(x
5

y=
4
3
2
1
X
– 4 –3 –2 –1 0 1 2 3 4 5 6
–1
–2
–3 y = 4 – 4x
–4

Co-ordinates of point where lines meets axes :


Line y = 2 (x – 1) : x-axis (i.e. y = 0) = (1, 0)
and y-axis (i.e. x = 0) = (0, – 2)
Line 4x + y = 4 : x-axis (i.e. y = 0) = (1, 0)
and y-axis (i.e. x = 0) = (0, 4) Ans.
24. A boat goes 30 km upstream and 44 km downstream in 10 hours. The same boat goes 40
km upstream and 55 km downstream in 13 hours. On this information some student guessed
the speed of the boat in still water as 8.5 km/h and speed of the stream as 3.8 km/h. Do you
agree with their guess ? Explain what do we learn from the incident ? [4]
Sol. Let the speed of boat = x km/hr.
Let the speed of stream = y km/hr.
Speed of boat in upstream = (x – y) km/hr.
Speed of boat in downstream = (x + y) km/hr.
30
Time taken to cover 30 km upstream = hrs.
x- y
https://www.arundeepselfhelp.info/index.php?route=product/product&path=180&product_id=395

Arundeep’s Solved Papers 118 Mathematics 2016 (Term I)


Time taken to cover 44 km downstream Þ 15u + 2 = 5 Þ 15u = 3
44 3 1
= hrs. Þ u = or u =
x+ y 15 5
According to question, we have
1
Total time taken = 10 hrs. Now, u =
11
30 44
+ = 10 ...(i) 1 1
x- y x+ y Þ =
x+ y 11
Now, Time taken to cover 55 km
Þ x + y = 11 ...(v)
55
downstream = hrs. 1
x+ y And u =
5
Time taken to cover 40 km upstream
1 1
=
40 Þ =
hrs. x- y 5
x- y
Þ x–y=5 ...(vi)
Given Total time taken = 3 hrs.
On adding eq. (v) and (vi) ; we have
40 55
+ = 13 ...(ii) x + y + x – y = 11 + 5
x- y x+ y Þ 2x = 16 or x = 8
Solving eq. (i) and eq. (ii) Put the value of x in eq. (v) ; we have
1 1 Þ 8 + y = 11 Þ y = 11 – 8
Let = u, = u.
x- y x+ y i.e. y = 3
Then eqn. (i) and (ii) reduces to, The speed of boat in still water = 8 km/hr.
30u + 44u = 10 The speed of stream = 3 km/hr.
40u + 55u = 13 We learns that the speed of boat is slow in
or 15u + 22u = 5 ...(iii) upstream and fast in downstream. Ans.
13 25. In an equilateral DABC, E is any point
8u + 11u = ...(iv)
5 1
on BC such that BE = BC. Prove that
Multiplying eq. (iii) by 8 and eq. (iv) by 4
15, we get 16AE2 = 13AB2. [4]
120u + 176v = 40 1
120u + 165v = 39 Sol. Given BE = BC
4 A
– – – Draw AD ^ BC
11v = 1
1
Þ u =
11
Putting the value of u in eq. (iii) ; we have
B C
15u + 22u = 5 E D
In DAED by pythagoras theorem,
1
Þ 15u + 22 ´ =5 AE2 = AD2 + DE2 ...(i)
11
https://www.arundeepselfhelp.info/index.php?route=product/product&path=180&product_id=395

Arundeep’s Solved Papers 119 Mathematics 2016 (Term I)


In DADB ; we have Sol. To prove : c (a + b) = ab
AB2 = AD2 + BD2 In DABD and DDXY ; we have
Þ AB2 = AE2 – DE2 + BD2 [From (i)] ÐB = ÐXYD [Each 90º]
= AE2 – DE2 + (BE + DE)2
ÐXDY = ÐADB [Common]
Þ AB2 = AE2 – DE2 + BE2 + DE2 + 2BE.DE
Þ AB2 = AE2 + BE2 + 2BE.DE So by AA axiom of similarity

2
DDAB ~ DDXY
æ BC ö BC
Þ AB2 = AE 2 + ç ÷ +2 (BD - BE)
è 4 ø 4
DY XY
BC 2 BC æ BC BC ö \ =
Þ AB2 = AE2 + + ç - ÷ DB AB
16 2 è 2 4 ø
c
AB2 AB é 2AB - AB ù Þ DY = (BD) ...(i)
Þ AB2 = AE 2 + + êë úû
a
16 2 4
In DBCD & DBYX ; we have
éQ DABC be an equilateral triangle ù ÐXYB = ÐD [Each 90º]
ê \ AB = BC = AC úû
ë ÐCBD = ÐXBY [Common]

AB2 AB2 AB So by AA axiom of similarity,


Þ AB2 = AE 2 + + ´
DBYX ~ DBDC
16 2 4
Thus their corresponding sides are
AB2 AB2
Þ AB2 - - = AE2 proportional.
16 8
BY XY
16AB2 - AB2 - 2AB2 Þ =
Þ = AE2 BD CD
16
Þ 16AB2 – 3AB2 = 16AE2 c
Þ BY = (BD) ...(ii)
i.e. 13AB2 = 16AE2 Hence Proved. b
26. In the figure, if ÐABD = ÐXYD = ÐCDB Adding eq. (i) and eq. (ii) ; we have
= 90º. AB = a, XY = c and CD = b, then
prove that c (a + b) = ab. [4] c c
Þ DY + BY = (BD) + (BD)
a b

éc cù
Þ BD = BD ê + ú
ëa bû

BD é cb + ca ù
Þ =ê
BD ë ab úû

c (a + b )
1 =
ab
c (a + b) = ab Hence Proved.
https://www.arundeepselfhelp.info/index.php?route=product/product&path=180&product_id=395

Arundeep’s Solved Papers 120 Mathematics 2016 (Term I)


27. In the DABC (see figure), ÐA = right x 5 x+5
angle, AB = x and BC = x + 5. = + = = 1 Ans.
x+5 x+5 x+5
Evaluate
sin C · cos C · tan C + cos2 C · sin A [4] cos B cos B
28. If = n and = m , then show
sin A cos A
that (m2 + n2) cos2 A = n2. [4]

cos B cos B
Sol. Given, n = ; m=
sin A cos A

cos 2 B cos 2 B
So, n2 = ; m2 =
sin 2 A cos 2 A
L.H.S. = (m2 + n2) cos2 A
Sol. In DABC, by pythagoras theorem ; we have
æ cos2 B cos 2 B ö 2
BC2 = AC2 + AB2 =ç + ÷ cos A
è cos2 A sin 2 A ø
( x + 5) 2 = ( x ) 2 + AC2
Þ x + 5 = x + AC2
(sin 2 A cos 2 B + cos 2 A cos 2 B)
= ´ cos2 A
cos2 A sin 2 A

cos 2 B (sin 2 A + cos 2 A)


=
sin 2 A

cos 2 B
=
sin 2 A
= n2 = R.H.S. Hence Proved.
29. Prove that :
5 = AC2 or AC = 5
2
sec A - 1 æ sin A ö
x 5 =ç ÷
\ sin C = ; cos C = ; sec A + 1 è 1 + cos A ø
x+5 x+5
= (cot A – cosec A)2 [4]
x
tan C = sec A - 1
5
Sol. L.H.S. =
sec A + 1
and sin A = sin 90º = 1
Then, sin C cos C tan C + cos2 C sin A
1 1 - cos A
-1
2 1 - cos A
x 5 x æ 5 ö = cos A = cos A =
= +ç ÷ ·1 1 1 + cos A 1 + cos A
x+5 x+5 5 è x+5ø +1
cos A cos A
https://www.arundeepselfhelp.info/index.php?route=product/product&path=180&product_id=395

Arundeep’s Solved Papers 121 Mathematics 2016 (Term I)

(1 - cos A) (1 + cos A)
= [on rationalization]
(1 + cos A) (1 + cos A)

1 - cos2 A sin 2 A
= =
(1 + cos A) 2 (1 + cos A) 2

2
æ sin A ö
=ç ÷ Hence Proved
è 1 + cos A ø

2
æ sin A ö
And, ç ÷
è 1 + cos A ø

éæ sin A ö (1 - cos A) ù é sin A (1 - cos A) ù 2


= êç ÷´ ú =
ëè 1 + cos A ø (1 - cos A) û êë 1 - cos2 A úû

2
é sin A (1 - cos A) ù
2
é1 - cos A ù
2
é 1 cos A ù
=ê úû = êë sin A úû = ê - ú
ë sin 2 A ë sin A sin A û
= (cosec A – cot A)2 = (– 1)2 [cot A – cosec A]2 = [cot A – cosec A]2 = R.H.S.
Hence Proved.
30. Following table shows marks (out of 100) of students in a class test :

Marks No. of students


More than or equal to 0 80
More than or equal to 10 77
More than or equal to 20 72
More than or equal to 30 65
More than or equal to 40 55
More than or equal to 50 43
More than or equal to 60 28
More than or equal to 70 16
More than or equal to 80 10
More than or equal to 90 8
More than or equal to 0
100

Draw a ‘more than type’ ogive. From the curve, find the median. Also, check the value of
the median by actual calculation. [4]
https://www.arundeepselfhelp.info/index.php?route=product/product&path=180&product_id=395

Arundeep’s Solved Papers 122 Mathematics 2016 (Term I)


Sol.

More than type C.I. No. of Students Frequency c.f.


More than or equal to 0 0 – 10 80 3 3
More than or equal to 10 10 – 20 77 5 8
More than or equal to 20 20 – 30 72 7 15
More than or equal to 30 30 – 40 65 10 25
More than or equal to 40 40 – 50 55 12 37
More than or equal to 50 50 – 60 43 15 52
More than or equal to 60 60 – 70 28 12 64
More than or equal to 70 70 – 80 16 06 70
More than or equal to 80 80 – 90 10 02 72
More than or equal to 90 90 – 100 8 08 80
More than or equal to 100 100 – 110 0 00
Taking lower limits x-axis and no. of students along y–axis. Draw the ordered pairs (0, 80),
(10, 77), (20, 72), (30, 65), (40, 55), (50, 43), (60, 28), (70, 16), (80, 10), (90, 8), (100, 0) on
graph paper and join them by free hand to give the required more than type ogive.
N 80
Here = = 40, So take a point P representing 40 on y-axis, draw a straight line || to x-axis
2 2
meeting ogive at Q. From Q draw a line ^ to x-axis meeting x-axis at R, Then abscissa of R
given the required median
\ Median = 52

P Q

Median by actual calculation :


https://www.arundeepselfhelp.info/index.php?route=product/product&path=180&product_id=395

Arundeep’s Solved Papers 123 Mathematics 2016 (Term I)

N 80 Sol. First convert the given table into C.I. Table.


Here N = 80 (even) \ = = 40
2 2
So c.f. which is just greater than 40 be 52 C.I. Frequency fi c.f.
So median class will be 50 – 60 0 – 10 4 (50-46) 4
and corresponding class interval be 10 – 20 6 (46-40) 10
50 – 60. 20 – 30 20 (40-20) 30
Thus median class be 50–60
30 – 40 10 (20-10) 40
Here, l = 50, h = 10, f = 15, c.f = 37,
40 – 50 7 (10-3) 47
é æN öù 50 – 60 3 50
ê ç - c. f ÷ú
= + ê ´ è2 øú åfi = N = 50
\ Median l h
êë f úû
N 50
é (40 - 37) ù Here, = = 25
= 50 + ê10 ´ 2 2
ë 15 úû and c.f. which just greater than 25 be 30
which lies in class interval 20 – 30.
3
= 50 + 10 ´ \ Median class be 20 – 30.
15
= 50 + 2 = 52
Hence Verified. Thus l = 20 ; f = 20 ; c.f. = 10, h = 10
31. From the following data find the median
é æN öù
age of 100 residents of a colony who took ê ç - c. f ÷ú
part in swachch bharat abhiyan : [4] \ Median = l + êh ´ è 2 ø
ú
êë f úû
Age (in yrs.) More
No. of residents
than or equal to
0 50 é (25 - 10) ù 15
= 20 + ê10 ú = 20 +
10 46 ë 20 û 2
20 40 = 27.5 Ans.
30 20
40 10
50 3
https://www.arundeepselfhelp.info/index.php?route=product/product&path=180&product_id=395

MATHEMATICS 2016 TERM II (OUTSIDE DELHI)


SET I
Time allowed : 3 hours Maximum marks : 90

General Instructions :
(i) All questions are compulsory.
(ii) The question paper consists of 31 questions divided into four sections – A, B, C and D.
(iii) Section A contains 4 questions of 1 mark each, Section-B contains 6 questions of 2 marks each.
Section-C contains 10 questions of 3 marks each and Section-D contains 11 questions of 4
marks each.
(iv) Use of calculators is not permitted.
1. In given figure, PQ is a tangent at a point Þ 30º + ÐPCA = 90º
C to a circle with centre O. If AB is a \ ÐPCA = 60º
diameter and ÐCAB = 30º, find ÐPCA.
2. For what value of k will k + 9, 2k – 1 and
2k + 7 are the consecutive terms of an
A.P. ?
Sol. Given that k + 9, 2k – 1 and 2k + 7 are in
A.P. [if a, b, c are in A.P then b – a = c – b]
Then (2k – 1) – (k + 9) = (2k + 7) – (2k – 1)
30°
Þ k – 10 = 8 Þ k = 18
3. A ladder, leaning against a wall, makes
an angle of 60º with the horizontal. If
the foot of the ladder is 2.5 m away from
Sol. Construction : Join AO. the wall, find the length of the ladder.
Given : PQ is tangent. AB is diameter Sol. Let AB be the ladder of length x and AC
ÐCAB = 30º. be the wall.
To Find : ÐPCA
Solution : In DAOC, AO = CO BC
In right DABC, = cos 60º
x
(Q Equal radii)
ÐCAO = ÐOCA (Q Angles opposite to 2.5 1
Þ = Þ x = 2 × 2.5 = 5 cm
equal sides are equal) x 2
or ÐCAB = ÐOCA A
But, ÐCAB = 30º So, ÐOCA = 30º ...(i)
Since, OC ^ PQ
(Q Tangent is perpendicular to radius at
point of contact) 60°
Þ ÐPCO = 90º B
2.5 m
C

Þ ÐOCA + ÐPCA = 90º


Thus, length of the ladder is 5 m.
Arundeep’s Solved Papers 124 Mathematics 2016 (Outside Delhi)
https://www.arundeepselfhelp.info/index.php?route=product/product&path=180&product_id=395

Arundeep’s Solved Papers 125 Mathematics 2016 (Outside Delhi)


4. A card is drawn at random from a well Let P (x, y), Q (x¢, y¢) are point of trisection.
shuffled pack of 52 playing cards. Find
the probability of getting neither a red Q
1 : 2P 2:1
card nor a queen.
A (2, – 2) B (– 7, 4)
Sol. Number of total possible outcomes when
one card is drawn = 52 Then using section formula
Number of favourable outcomes when card P divides AB in the ratio 1 : 2 internally
is neither red nor queen = 52 – 28 = 24 Coordinates of P are
[we have 26 red cards and 2 black queens]
æ 2 ´ 2 + 1( -7) (-2) (2) + 1(4) ö
\ Required probability ç , ÷
è 1+ 2 1+ 2 ø
Favourable outcomes
= or (– 1, 0)
Total possible outcomes
Q is mid point of PB. So using mid point
24 6 formula coordinates of Q are
= =
52 13 æ -1 - 7 0 + 4 ö
ç , ÷ or (– 4, 2)
5. If – 5 is a root of the quadratic equation è 2 2 ø
2x2 + px – 15 = 0 and the quadratic 7. In figure, a quadrilateral ABCD is drawn
equation p (x2 + x) + k = 0 has equal roots, to circumscribe a circle, with centre O,
find the value of k. in such a way that the sides AB, BC, CD
Sol. Q – 5 is the root of the quadratic equation and DA touch the circle at the points P,
2x2 + px – 15 = 0 Q, R and S respectively. Prove that
Þ 2 (– 5)2 + p (– 5) – 15 = 0 AB + CD = BC + DA.
[_ if a be a root of f(x) = 0 then f(a) = 0]
Þ 50 – 5p – 15 = 0 Þ 35 – 5p = 0
Þ 5p = 35 Þ p = 7
Now, given that equation p (x2 + x) + k = 0
has equal roots
i.e. 7 (x2 + x) + k = 0 has equal roots
i.e. 7x2 + 7x + k = 0 has equal roots
Þ 72 – 4 × 7 × k = 0 Sol. We know that tangents drawn to a circle
from an outer points are equal.
[Q For equal roots, D = 0, i.e. b2 – 4ac = 0
So, AP = AS, BP = BQ, CR = CQ and DR
Here a = 7, b = 7 and c = k] = DS.
7 Now, consider
Þ 7 (7 – 4k) = 0 Þ k=
4 AP + BP + CR + DR = AS+ BQ + CQ + DS
6. Let P and Q be the points of trisection of Þ AP + PB + CP + PD = AS + SD + BQ + QC
the line segment joining the points Þ AB + CD = AD + BC
A (2, – 2) and B (– 7, 4) such that P is Hence proved.
nearer to A. Find the coordinates of
8. Prove that the points (3, 0), (6, 4) and
P and Q.
(– 1, 3) are the vertices of a right angled
Sol. Let A (2, – 2), B (– 7, 4) be given points. isosceles triangle.
https://www.arundeepselfhelp.info/index.php?route=product/product&path=180&product_id=395

Arundeep’s Solved Papers 126 Mathematics 2016 (Outside Delhi)


Sol. Let the triangle be DABC as shown in Since an = 0
figure. \ 0 = a + (4 – 1) d
Þ 0 = a + 3d Þ a = – 3d
Now a25 = a + (25 – 1) d = a + 24d
= – 3d + 24d = 21d = 3 × 7d
Hence, a25 = 3 × a11
[Q Since a111 = a + (11 – 1) d
= – 3d + 10d = 7d]
Using distance formula, we have 10. In given figure, from an external point
P, two tangents PT and PS are drawn to
AB = (3 - 6)2 + (0 - 4)2
a circle with centre O and radius r. If
PO = 2r, show that ÐOTS = ÐOST = 30º.
= (-3) 2 + 4 2 = 9 + 16 = 25 = 5

BC = (6 + 1) 2 + (4 - 3) 2

= 72 + 12 = 49 + 1 = 50 = 25 ´ 2

=5 2

CA = (-1 - 3) 2 + (3 - 0) 2 Sol. Let ÐTOP = q


In right triangle OTP we have
= (-4) 2 + 32 = 16 + 9 = 25 = 5
OT r 1
Here, AB = AC Þ ABC is an isosceles \ cos q = = = = cos 60º
triangle OP 2r 2
Consider, AB2 + AC2 = (5)2 + (5)2 Þ q = 60º
= 25 + 25 = 50 Hence ÐTOS = 2 × 60 = 120º
and, BC2 = (5 2) 2 = 50 [Q ÐTOP = ÐPOS as angles opposite to
equal tangents are equal]
\ Here, AB2 + AC2 = BC2
Þ DABC is a right angled triangle. In DOTS, we have OT = OS

[Q In right D, using Pythagoras theorem [Q Equal radii]


(H)2 = (P)2 + (B)2] Þ ÐOTS = ÐOST
where H = hypotenuse, B = base, [Q Angle opposite to equal sides are equal]
P = perpendiculars]
In DOTS, we have
9. The 4th term of an A.P. is zero. Prove
that the 25th term of the A.P. is three ÐOTS + ÐOST + ÐTOS = 180º
times its 11th term. Þ 2ÐOST = 60º
Sol. Let a be first term and d be the common \ ÐOST = ÐOTS = 30º Hence proved.
difference of the given A.P. Then 11. In figure, O is the centre of a circle such
we know that an = a + (n – 1) d that diameter AB = 13 cm and AC = 12 cm.
https://www.arundeepselfhelp.info/index.php?route=product/product&path=180&product_id=395

Arundeep’s Solved Papers 127 Mathematics 2016 (Outside Delhi)


BC is joined. Find the area of the shaded region. (Take p = 3.14)
Sol. Here, BC2 = AB2 – AC2 = 169 – 144 = 25
\ BC = 5
\ Area of shaded region = Area of semicircle – Area of right triangle ABC
1 1
= ´ pr 2 - AC ´ BC
2 2
[Since diameter of semi-circle = AB = 13 cm]
2
1 æ 13 ö 1
= ´ 3.14 ç ÷ - ´ 12 ´ 5 = 66.33 – 30 = 36.33 cm2
2 è2ø 2
12. In figure, a tent is in the shape of a cylinder surmounted by a

2. 8
conical top of same diameter. If the height and diameter of

m
cylindrical part are 2.1 m and 3 m respectively and the slant
height of conical part is 2.8 m, find the cost of canvas needed to
make the tent if the canvas is available at the rate of ` 500/sq.
é 22 ù
metre. ê Use π = ú
ë 7û 2.1 m

3m

3
Sol. Given radius of cylindrical part = radius of conical part = r = m = 1.5 m
2
and height of cylindrical part = h = 2.1 m
\ Slant height of cone = l = 2.8 m
Area for canvas needed = curved surface area of cylinder + curved surface area of cone
= 2prh + prl

22 22 22 22
= 2´ ´ 1.5 ´ 2.1 + ´ 1.5 ´ 2.8 = [6.3 + 4.2] = ´ 10.5 = 33 m2
7 7 7 7
Thus required cost of canvas needed to make the tent at the rate of `500 per m2
= 33 × 500 = ` 16500
13. If the point P (x, y) is equidistant from the points A (a + b, b – a) and B (a – b, a + b). Prove
that bx = ay.
Sol. Given, PA = PB Þ PA2 = PB2
b–a
Applying distance formula, we have
Þ (a + b – x)2 + (b – a – y)2 = (a – b – x)2 + (a + b – y)2
Þ (a + b)2 + x2 – 2ax – 2bx + (b – a)2 + y2 – 2by + 2ay
https://www.arundeepselfhelp.info/index.php?route=product/product&path=180&product_id=395

Arundeep’s Solved Papers 128 Mathematics 2016 (Outside Delhi)


= (a – b)2 + x2 – 2ax + 2bx + (a +b)2
+ y2 – 2ay – 2by n
[2a + ( n - 1) d ]
Sn 2
Þ 4ay = 4bx Þ ay = bx or bx = ay =
Sn¢ n
[2a¢ + ( n - 1) d ¢]
Hence proved. 2
14. In figure, find the area of the shaded
region, enclosed between two concentric æ n - 1ö
circles of radii 7 cm and 14 cm where a+ç ÷d
è 2 ø 7n + 1
= =
é 22 ù æ n -1ö 4n + 27 ...(i)
ÐAOC = 40º. ê Use π = ú a¢ + ç ÷ d¢
ë 7û è 2 ø

tm a + ( m - 1) d
Since ¢ =
tm a¢ + ( m - 1) a¢

[Q Let tm, tm be mth terms of two A.P.’s]

n -1
So replacing by m – 1,
2
i.e. n = 2m – 1 in eqn. (i) ; we get
tm a + ( m - 1) d
=
¢
tm a¢ + ( m - 1) d ¢
Sol. Given outer radius of circle = R = 14 cm
Radius of inner circle = r = 7 cm
7 (2m - 1) + 1 14m - 6
Here q¢= 360° – q = 360° – 40° = 320° = =
4 (2m - 1) + 27 8m + 23
\ Area of shaded region
Thus, the ratio of their mth terms is
360º - q 14m – 6 : 8m + 23.
= ´ p (R 2 - r 2 )
360º 16. Solve for x :
320º 1 1 2
= ´ p [(14)2 - (7) 2 ] + = ,
360º ( x - 1) ( x - 2) ( x - 2) ( x - 3) 3
8 22 8 22 x ¹ 1, 2, 3
= ´ (196 - 49) = ´ ´ 147
9 7 9 7 Sol. Given eqn. be

1232 1 1 2
= = 410.67 cm2 + = ,
3 ( x - 1) ( x - 2) ( x - 2) ( x - 3) 3
15. If the ratio of the sum of first n terms of x ¹ 1, 2, 3
two A.P.’s is (7n + 1) : (4n + 27), find the
x - 3 + x -1 2
ratio of their mth terms. Þ =
( x - 1) ( x - 2) ( x - 3) 3
Sol. Let Sn and Sn¢ be the sum of n terms of two
A.P.’s Let a, a¢ and d, d¢ be first terms and (2 x - 4) 2
Þ ( x - 1) ( x - 2) ( x - 3) =
common difference of two A.P.’s. Then 3
https://www.arundeepselfhelp.info/index.php?route=product/product&path=180&product_id=395

Arundeep’s Solved Papers 129 Mathematics 2016 (Outside Delhi)

2 ( x - 2) 18. A sphere of diameter 12 cm, is dropped


2
Þ = in a right circular cylindrical vessel,
( x - 1) ( x - 2) ( x - 3) 3 partly filled with water. If the sphere is
Þ 3 = (x – 1) (x – 3) completely submerged in water, the
water level in the cylindrical vessel rises
Þ x2 – 4x + 3 = 3 [_ x – 2 ¹ 0]
5
Þ x2 – 4x = 0 by 3 cm. Find the diameter of the
9
Þ x (x – 4) = 0
cylindrical vessel.
Þ x = 0 or x = 4
12
17. A conical vessel, with base radius 5 cm Sol. Given radius of sphere = R = cm = 6 cm
and height 24 cm, is full of water. This 2
water is emptied into a cylindrical vessel We know that,
of base radius 10 cm. Find the height to
4
which the water will rise in the \ Volume of sphere = pR 3
3
é 22 ù
cylindrical vessel. êë Use π = 7 úû 4 4
= p ´ (6)3 = p ´ 216 cm3
3 3
Sol. Given base radius of conical vessel =r = 5 cm
Let radius of cylindrical vessel be r cm then,
and height of vessel = h = 24 cm volume of water in cylinder
and Volume of water in conical vessel = h
32
= pr2 h = pr 2 ´ cm3
1 2 9
= pr h
3 [Since rise in water level in cylindrical
1 22 5 32
= ´ ´ 5 ´ 5 ´ 24 cm3 vessel = h = 3
9
cm =
9
cm
3 7
Let height of water in cylindrical vessel be According to question, we have
h then volume of water in cylinder = pR2h volume of sphere = volume of cylinderical
vessel
Where R = radius of cylindrical vessel = 10 cm
4 32
22 p ´ 216 = p ´ r 2 ´
= × 10 × 10 × h cm3 3 9
7
4 ´ 216 ´ 9
A.T.Q., we have Þ r2 = = 27 × 3 = 81
3 ´ 32
Volume of water in conical vessel = Volume
of water in cylindrical vessel Þ r2 = 81 = 92
i.e. r = 9 cm
1 22 22 \ Required diameter of cylindrical vessel
Þ ´ ´ 5 ´ 5 ´ 24 = ´ 10 ´ 10 ´ h
3 7 7 = 2r = 18 cm
19. A man standing on the deck of a ship,
5 ´ 5 ´ 24
Þ h = = 2 cm which is 10 m above water level, observes
3 ´ 10 ´ 10 the angle of elevation of the top of a hill
\ Required height to which water rises in as 60º and the angle of depression of the
cylindrical vessel = 2 cm base of hill as 30º. Find the distance of
https://www.arundeepselfhelp.info/index.php?route=product/product&path=180&product_id=395

Arundeep’s Solved Papers 130 Mathematics 2016 (Outside Delhi)


the hill from the ship and the height of 3
the hill. \ P (exactly two heads) =
8
Sol. Let ED be the water level and BE = 10 m
Let CD be the hill of height h from water (ii) In case of at least two heads, outcomes are
level. HHT, HTH, THH and HHH.
Let AB = x 4 1
In right DDEB, we have \ P (at least two heads) = =
8 2
BE 10 1 (iii) In case of at least two tails, outcomes are
= tan 30º Þ =
DE x 3 TTH, THT, HTT and TTT.

Þ x = 10 3 m 4 1
\ P (at least two tails) = =
8 2
C 21. Due to heavy floods in a state, thousands
were rendered homeless. 50 schools
(h – 10) m collectively offered to the state
government to provide place and the
B A
hm canvas for 1500 tents to be fixed by the
government and decided to share the
whole expenditure equally. The lower
part of each tent is cylindrical of base
E radius 2.8 m and height 3.5 m, with
x D
conical upper part of same base radius
but of height 2.1 m. If the canvas used to
CA
Now, in DCAB, = tan 60º make the tents costs ` 120 per sq. m, find
x the amount shared by each school to set
up the tents. What value is generated by
Þ h - 10 = 3 [From (i)]
10 3 é 22 ù
the above problem ? êë Use π = 7 úû
Þ h – 10 = 30 Þ h = 40 m
Sol. Given radius of cylindrical part = radius of
So, distance of hill from ship = 10 3 m and
conical part = 2.8 m
the height of the hill = 40 m.
and h = height of cylindrical part = 3.5 m
20. Three different coins are tossed together.
Find the probability of getting H = height of conical part = 2.1 m
(i) exactly two heads Slant height of conical part = r 2 + H 2
(ii) at least two heads
(iii) at least two tails. = (2.8) 2 + (2.1)2 = 7.84 + 4.41
Sol. Possible outcomes when three coins are
tossed = 12.25 = 3.5 m
HHH, HHT, HTT, TTT, THH, TTH, HTH, \ Area of tents = Curved surface area of
THT cylindrical part + Curved surface area of
(i) Number of exactly two heads are HHT, conical part
HTH and THH. = 2prh + prl
https://www.arundeepselfhelp.info/index.php?route=product/product&path=180&product_id=395

Arundeep’s Solved Papers 131 Mathematics 2016 (Outside Delhi)

22 22 [Radius is perpendicular to the tangent at


= 2´ ´ 2.8 ´ 3.5 + ´ 2.8 ´ 3.5 the point of contact]
7 7

22
= 3´ ´ 2.8 ´ 3.5 = 92.4 m 2
7

OA = OB (radii) ...(ii)
2.1 m
OP is common ...(iii)
2.8 m
\ DOAP @DOBP(RHS axiom of congruency)
[from (i), (ii) and (iii)]
Hence, AP = BP (CPCT)
3.5 m
23. Draw a circle of radius 4 cm. Draw two
tangents to the circle inclined at an angle
2.8 m of 60º to each other.
Sol. Steps of construction :
1. Draw a circle of radius 4 cm with centre O.
\ Canvas required for 1500 tents
2. Take point A on circle. Join OA.
= 1500 × 92.4 = 138600 m2
3. Draw line AP perpendicular to radius OA.
Cost of 1500 tents = (1500 ×92.4) × 120
4. Draw ÐAOB = 120º at O.
= ` 16632000
5. Join A and B at P, to get 2 tangents. Here
[Q Making tents costs ` 120 per sq. m] ÐAPB = 60º.
16632000
Thus, share of each school =
50
= ` 332640
School authorities are concerned about
safety of children and their families.
22. Prove that the lengths of tangents drawn
from an external point to a circle are
equal.
Sol. Given : A circle C (O, r), P is a point outside 24. In given figure, two equal circles, with
the circle and PA and PB are tangents to a centres O and O¢, touch each other at X.
circle. OO¢ produced meets the circle with
To Prove : PA = PB centre O¢ at A. AC is tangent to the circle
with centre O at the point C. O¢D is
Construction : Draw OA, OB and OP.
perpendicular to AC. Find the value of
Proof : Consider triangles OAP and OBP.
DO¢
ÐOAP = ÐOBP = 90º ...(i) .
CO
https://www.arundeepselfhelp.info/index.php?route=product/product&path=180&product_id=395

Arundeep’s Solved Papers 132 Mathematics 2016 (Outside Delhi)


Þ (3x + 4) (x + 4) = 4 (x2 + 3x + 2)
Þ 3x2 + 16x + 16 = 4x2 + 12x + 8
Þ x2 – 4x – 8 = 0
on comparing with ax2 + bx + c = 0
We have a = 1 ; b = –4 and c = –8
Sol. AC is tangent to the circle with centre O. Then by quadratic formula, we have
In DADO¢and DACO,
ÐADO¢ = ÐACO (each 90º) -b ± b2 - 4ac
x=
ÐDAO = ÐCAO (common) 2a

4 ± 16 + 32
Þ x =
2

r r r 4±4 3
Þ x = = 2±2 3
2

\ By AA axiom of similarity 26. The angle of elevation of the top Q of a


vertical tower PQ from a point X on the
DADO¢ ~ DACO ground is 60º. From a point Y, 40 m
Thus their corresponding sides are vertically above X the angle of elevation
proportional. of the top Q of tower is 45º. Find the
height of the tower PQ and the distance
AO¢ DO¢
\ = PX. (Use
AO CO 3 = 1.73)

[Q corresponding parts of similar triangle] Sol. Let height of tower PQ be h.


Let z be the distance between X and P.
AO = AO¢ + O¢X + XO = r + r + r = 3r
Q XPRY is a rectangle.
DO¢ r \ RP = XY = 40 m and PX = YR = z
Þ =
CO 3r
PQ
DO¢ 1 In right DQPX, = tan 60º
= PX
\
CO 3
h h
25. Solve for x : Þ = 3 Þ =z
z 3
1 2 4
+ = , x ¹ – 1, – 2, – 4
x+1 x+ 2 x+4
1 2 4
Sol. Given eqn. be, + = (h – 40) m
x +1 x + 2 x+4
x + 2 + 2x + 2 4 R
h
Þ =
( x + 1) ( x + 2) x+4
40 m
3x + 4 4
Þ =
( x + 1)( x + 2) ( x + 4) z
https://www.arundeepselfhelp.info/index.php?route=product/product&path=180&product_id=395

Arundeep’s Solved Papers 133 Mathematics 2016 (Outside Delhi)


( x - 1)
In right DQRY,
QR
= tan 45º \ Sx–1 = × [2 + x – 2]
YR 2
x ( x - 1)
h - 40 =
Þ = 1 Þ h – 40 = z ...(ii) 2
z
Now, A.P. of total number of houses
From (i) and (ii), we get following x is :
h (x + 1) + (x + 2) + ... + 49
3
= h – 40 \ n = 49 – (x + 1) + 1 = 49 – x
n
Þ h = h 3 - 40 3 \ Sum of these numbers, S n = [a + l ],
2
Þ h 3 - h = 40 3 where l be the last term
(49 - x)
Þ h ( 3 - 1) = 40 3 S49 – x = [ x + 1 + 49]
2
40 3 40 3 ( 3 + 1) (49 - x)
Þ h = = = ( x + 50)
( 3 - 1) ( 3 - 1) ( 3 + 1) 2
According to question, we have
40 (3 + 3)
Þ h = x ( x - 1) (49 - x) ( x + 50)
2 =
2 2
Þ h = 20 (3 + 1.73) = 20 × 4.73 = 94.6 m
...(iii) Þ x2 – x = 49x + 2450 – x2 – 50x
So, height of the tower PQ = 94.6 m Þ 2x2 = 2450
and the distance PX = 94.6 – 49 = 54.6 m Þ x2 = 1225 = (35)2 Þ x = 35
[From (ii) and (iii)] Justification :
27. The houses in a row are numbered Now, A.P. of numbers before house
consecutively from 1 to 49. Show that numbered x = 1 + 2 + .... + 34
there exists a value of X such that sum
34 34
of numbers of houses preceding the \ S34 = [a + l ] = ´ [1 + 34] = 17 × 35
house numbered X is equal to sum of the 2 2
numbers of houses following X. = 595
Sol. The A.P. of numbers of houses preceding Now, A.P. of numbers following house
house numbered x is : numbered x = 36 + 37 ... + 49
1 + 2 + 3 + ... + (x – 1) 14
\ S¢ = [36 + 49] = 7 × 85 = 595
n 2
\ Sum, Sn = [2a + ( n - 1) d ],
2 Hence, for value of x = 35, the sum of
where a ® first term numbers of houses preceding house
numbered x is equal to sum of numbers of
d ® common difference
houses following x.
( x - 1) 28. In fig. given below the vertices of DABC
= [2 × 1 + (x – 1 – 1) × 1]
2 are A (4, 6), B (1, 5) and C (7, 2). A line-
https://www.arundeepselfhelp.info/index.php?route=product/product&path=180&product_id=395

Arundeep’s Solved Papers 134 Mathematics 2016 (Outside Delhi)


segment DE is drawn to intersect the sides AB and AC at D and E respectively such that
AD AE 1
= = . Calculate the area of DADE and compare it with area of DABC.
AB AC 3

AD 1
Sol. Given : =
AB 3
3AD = AB
\ 3AD = AD + DB Þ 2AD = DB
2 2
AD 1
=
DB 2

AE 1
Similarly, =
EC 2
Thus D divides AB in the ratio 1 : 2 and E divides AC in the ratio 1 : 2.
Then by using section formula, we have
æ 1(1) + 2 (4) 1(5) + 2 (6) ö
Coordinates of D are ç , ÷
è 1+ 2 1+ 2 ø
æ 17 ö
i.e. ç 3, ÷
è 3ø
æ 1(7) + 2 (4) 1(2) + 2 (6) ö
Coordinates of E are ç , ÷
è 1+ 2 1+ 2 ø
æ 14 ö
i.e. ç 5, ÷
è 3ø
1
Area of DADE = |x (y – y ) + x2 (y3 – y1) + x3 (y1 – y2)|
2 1 2 3
1 é æ 17 14 ö æ 14 ö æ 17 ö ù
= ê 4 ç - ÷ + 3 ç - 6÷ + 6 ç 6 - ÷ú
2 ë è 3 3ø è 3 ø è 3 øû

1 é æ -4 ö æ 1 öù 1 é 5ù 5
= ê 4+3 ç ÷ + 5 ç ÷ú = êë4 - 4 + 3 úû = 6 sq. units
2 ë è 3 ø è 3 øû 2

1
Area of DABC = |[4 (5 – 2) + 1 (2 – 6) + 7 (6 – 5)]|
2
1 1 1 15
= |[4 × 3 + (– 4) + 7 × 1]| = |[12 – 4 + 7]| = ´ 15 = sq. units
2 2 2 2

5
Area ( DADE) 5 15 5 12 2
Hence, = 6 = ¸ = ´ =
Area (DABC) 15 6 12 6 15 3
12
https://www.arundeepselfhelp.info/index.php?route=product/product&path=180&product_id=395

Arundeep’s Solved Papers 135 Mathematics 2016 (Outside Delhi)


29. A number x is selected at random from pr q
the numbers 1, 2, 3 and 4. Another = r tan q + r sec q – r +
180
number y is selected at random from the
numbers 1, 4, 9 and 16. Find the é pq ù
= r ê tan q + sec q - 1 +
probability that product of x and y is less ë 180 úû
than 16.
31. A motor boat whose speed is 24 km/h in
Sol. x can be any one of 1, 2, 3 or 4 and y can be still water takes 1 hour more to go 32 km
any one of 1, 4, 9 or 16. upstream than to return downstream to
Total number of cases of xy = 4 × 4 = 16 the same spot. Find the speed of the
Number of cases when product is less than stream.
16 are 1 × 1, 1 × 4, 1 × 9, 2 × 1, 2 × 4, 3 × Sol. Let the speed of the stream be x km/h
1, 3 ×4, 4 × 1, i.e. 8 cases. Upstream case :
\ Total no. of favourable cases = 8 Speed of boat = (24 – x) km/h
\ Required probability Time taken for going 32 km upstream
32
Number of favourable cases = 8 = 1 = hours
= 24 - x
Total number of cases 16 2
Downstream case :
30. In figure, is shown a sector OAP of a
Speed of boat = (24 + x) km/h
circle with centre O, containing Ðq. AB
is perpendicular to the radius OA and Time taken for going 32 km downstream
meets OP produced at B. Prove that the 32
perimeter of shaded region is = hours
24 + x
é pq ù
r êtan q + sec q + - 1ú . According to question, we have
ë 180 û
32 32
- =1
24 - x 24 + x

é 24 + x - 24 + x ù
Þ 32 ê ú =1
ë (24 - x ) (24 + x) û

é Distance ù
êQ Time = Speed ú
ë û

» = q ´ 2pr = pr q æ 2x ö
Sol. Length are AP Þ 32 ç ÷ =1
360° 180° è 576 - x 2 ø
AB Þ 64x = 576 – x2 Þ x2 + 64x – 576 = 0
Now = tan q Þ AB = r tan q
r Þ x2 + 72x – 8x – 576 = 0
OB Þ x(x + 72) – 8(x + 72) = 0
= sec q Þ OB = r sec q
r Þ (x + 72) (x – 8) = 0
\ PB = OB – r = r sec q – r Þ x = 8, x = – 72
Þ x=8 (as speed cannot be negative)
»
\ Perimeter of shaded region =AB + PB + AP
Hence, speed of the stream is 8 km/hr.
https://www.arundeepselfhelp.info/index.php?route=product/product&path=180&product_id=395

Arundeep’s Solved Papers 136 Mathematics 2016 (Outside Delhi)

SET-II [UNCOMMON QUESTIONS TO SET-I]


10. Solve for x : 111 × (5) – 99 (– 3) = 555 + 297 = 852
2 x + 9 + x = 13. 19. If the roots of the quadratic equation
(a – b) x2 + (b – c) x + (c – a) = 0 are
Sol. Given eqn. be 2 x + 9 + x = 13 equal, prove that 2a = b + c.
Þ 2 x + 9 = (13 – x) Sol. Given quadratic eqn. be
On squaring both sides, we get (a – b) x2 + (b – c) x + (c – a) = 0
Þ 2x + 9 = (13 – x)2 On comparing with ax2 + bx + c = 0
Þ 2x + 9 = 169 – 26x + x2 we have ‘a’ = a – b ;
Þ x2 – 28x + 160 = 0
‘b’ = b – c and ‘c’ = c – a
Þ x2 – 20x – 8x + 160 = 0
For equal roots, discriminant, D = 0
Þ x(x – 20) – 8 (x – 20) = 0
Þ b2 – 4ac = 0
Þ (x – 20) (x – 8) = 0
Þ x = 20 or x = 8 Þ (b – c)2 – 4 (a – b) (c – a) = 0
Þ x=8 Þ b2 – 2bc + c2– 4 (ac – a2 – bc + ab) = 0
[as x = 20 does not satisfy the given equation Þ b2 – 2bc + c2 – 4ac + 4a2 + 4bc – 4ab = 0
Then L.H.S. is positive while R.H.S. is Þ 4a2 + b2 + c2 – 4ab + 2bc – 4ac = 0
negative.]
18. The digits of a positive number of three Þ (2a – b – c)2 = 0
digits are in A.P. and their sum is 15. The Þ 2a – b – c = 0
number obtained by reversing the digits Þ 2a = b + c
is 594 less than the original number. Find
the number. 20. From a pack of 52 playing cards, Jacks,
Queens and Kings of red colour are
Sol. Let the required numbers in A.P. are a – d,
removed. From the remaining, a card is
a, a + d respectively.
drawn at random. Find the probability
Now, a – d + a + a + d = 15 that drawn card is :
[Q Sum of digits = 15] (i) a black King.
Þ 3a = 15 Þ a = 5
(ii) a card of red colour.
According to question, number is
(iii) a card of black colour.
100 (a – d) + 10a + a + d, i.e. 111a – 99d
Number on reversing the digits is Sol. Removed red colour cards = 3 × 2 = 6
100 (a + d) + 10a + a – d, i.e. 111a + 99d [Since there are 2 red jacks, 2 red queens
and 2 red kings]
Now, as per given condition in question,
(111a – 99d) – (111a + 99d) = 594 \ No. of remaining cards = 52 – 6 = 46
Þ – 198d = 594 (i) Number of black kings = 2 = No. of
favourable cases
Þ d=–3
\ Digits of number are 2 1
\ P (a black king) = =
[5 – (– 3), 5, (5 + (– 3)] = 8, 5, 2 46 23
\ Required number is (ii) Number of red colour cards = 26
https://www.arundeepselfhelp.info/index.php?route=product/product&path=180&product_id=395

Arundeep’s Solved Papers 137 Mathematics 2016 (Outside Delhi)


\ Remaining no. of red colour cards = 26 – 6 = 20 3. Along LP cut off LA = 3 cm.
= No. of favourable cases 4. Join BA and CA. Then DABC so obtained
is the required DABC.
20 10
\ P (a card of red colour) = = 5. Draw an acute angle CBY and cut 4 equal
46 23 lengths as
(iii) Number of black cards = 26 BA1 = A1A2 = A2A3 = A3A4 and join CA4.
= No. of favourable cases 6. Now draw a line through A3 parallel to CA4
intersecting BC at C¢.
26 13
\ P (a black colour card) = = 7. Draw a line through C¢ and parallel to AC
46 23
intersecting AB at A¢. Thus, A¢BC¢ is the
28. Draw an isosceles DABC in which required triangle.
BC = 5.5 cm and altitude AL = 3 cm. 29. Prove that tangent drawn at any point
Then construct another triangle whose of a circle is perpendicular to the radius
3 through the point of contact.
sides are of the corresponding sides
4 Sol. Given : A circle C (O, r) and a tangent AB
of DABC. at a point P.
Sol. To prove : OP ^ AB
Construction : Take any point Q other than
P on the tangent AB.
P
Join OQ, intersecting circle at R.
Proof : We have, OP = OR [Radii]
OQ = OR + RQ
A¢ \ OQ > OR Þ OQ > OP
[Q OR = OP = radius]

A Thus, OP < OQ i.e. OP is shorter than any


other segment joining O to any point of AB.
A
But among all line segments, joining point
O to point on AB, shortest one is
perpendicular from O on AB.
Steps of construction : Hence, OP ^ AB
1. Draw a line segment BC = 5.5 cm.
30. As observed from the top of a light house,
2. Construct AP the perpendicular bisector of 100 m high above sea level, the angles of
BC meeting BC at L. depression of a ship, sailing directly
https://www.arundeepselfhelp.info/index.php?route=product/product&path=180&product_id=395

Arundeep’s Solved Papers 138 Mathematics 2016 (Outside Delhi)


towards it, changes from 30º to 60º. Find 31. A rectangular park is to be designed
the distance travelled by the ship during whose breadth is 3 m less than its length.
the period of observation. Its area is to be 4 square metres more
than the area of a park that has already
(Use 3 = 1.73) been made in the shape of an isosceles
Sol. Let AB be the tower of height 100 m. triangle with its base as the breadth
of the rectangular park and of altitude
and C and D are the two positions of ships
12 m. Find the length and breadth of the
s.t BC = y and CD = x.
rectangular park.
AB Sol. Let length of rectangular park be x m and
In right DABC, = tan 60º Breadth be (x – 3) m
BC
Given and base of isosceles triangle
A = (x – 3) m
and Altitude of triangle = 12 m

x–3

y x D
x

100 100 According to question, we have


Þ = 3 Þy = ...(i) Area of rectangular park = Area of isosceles
y 3
triangle + 4
AB 1
In right DABD, = tan 30º Þ x (x – 3) = ( x - 3) ´ 12 + 4
BD 2
100 1 Þ x (x – 3) = 6 (x – 3) + 4
Þ y+x = Þ x2 – 3x = 6x – 18 + 4
3
Þ x2 – 9x + 14 = 0
Þ x + y = 100 3 Þ (x – 7) (x – 2) = 0
Þ x = 7 or x = 2
Þ x = 100 3 - y

100 300 - 100 200


Þ x = 100 3 - = =
3 3 3
12
...[From (i)]

200 3 200 ´ 1.73 (x – 3)


Þ x = = = 115.33 m
3 3
But x = 2 is rejected otherwise breadth will
The required distance travelled by the ship is be –ve which is not possible.
115.33 m.
\ Length of rectangular park is 7 m and
breadth be 4 m.
https://www.arundeepselfhelp.info/index.php?route=product/product&path=180&product_id=395

Arundeep’s Solved Papers 139 Mathematics 2016 (Outside Delhi)

SET-III [UNCOMMON QUESTIONS TO SET-I and Set II]

10. Solve for x : So, number of favourable cases = 3


6 x + 7 - (2 x - 7) = 0. \ Required probability
Number of favourable cases 3
Sol. Given eqn. be, 6 x + 7 - (2 x - 7) = 0 = =
Total possible cases 100
Þ 6 x + 7 = 2x – 7 19. Three consecutive natural numbers are
On squaring both sides, we get such that the square of the middle
Þ 6x + 7 = (2x – 7)2 number exceeds the difference of the
Þ 6x + 7 = 4x2 – 28x + 49 squares of the other two by 60. Find the
Þ 4x2 – 34x + 42 = 0 numbers.
Þ 2x2 – 17x + 21 = 0 Sol. Let the three consecutive natural numbers
Þ 2x2 – 14x – 3x + 21 = 0 are x – 1, x and x + 1.
Þ 2x (x – 7) – 3 (x – 7) = 0 According to given condition, we have
Þ (2x – 3) (x – 7) = 0 x2 – [(x + 1)2 – (x – 1)2] = 60
Þ x2 – [(x + 1 – x + 1) (x + 1 + x – 1)] = 60
3
Þ x = 7 or x = Þ x2 – 4x – 60 = 0
2
Þ x2 – 10x + 6x – 60 = 0
Þ x=7
Þ x (x – 10) + 6 (x – 10) = 0
é 3 ù Þ (x + 6) (x – 10) = 0
ê as x = 2 does not satisfy the equation, in this ú
ê ú Þ x = 10 (x = – 6, rejected)
ë case L.H.S. is positive while R.H.S. is negative û
Hence, the required numbers are 9, 10 and
18. There are 100 cards in a bag on which 11.
numbers from 1 to 100 are written. A
20. The sums of first n terms of three
card is taken out from the bag at random.
arithmetic progressions are S1, S2 and S3
Find the probability that the number on
respectively. The first term of each A.P. is
the selected card :
1 and their common differences are 1, 2
(i) is divisible by 9 and is a perfect square. and 3 respectively. Prove that S1 + S3 = 2S2.
(ii) is a prime number greater than 80.
Sol. Here, sum of n terms of AP is
Sol. Total possible cases = 100
n
(i) Favourable cases when number is a perfect Sn = [2a + (n - 1) d ]
square and is divisible by 9 are 9, 36 and 2
81.
n
So, number of favourable cases = 3 \ S1 = [2 + ( n - 1)1]
2
\ Required probability
n (n + 1)
Number of favourable cases 3 = [Q where a = 1, d = 1]
= = 2
Total possible cases 100
n
(ii) Favourable cases the prime numbers greater S2 = [2 + ( n - 1) 2]
than 80 are 83, 89 and 97 2
https://www.arundeepselfhelp.info/index.php?route=product/product&path=180&product_id=395

Arundeep’s Solved Papers 140 Mathematics 2016 (Outside Delhi)

n 29. From a point on the ground, the angle of


= (2n) = n2 [Q where a = 1, d = 2] elevation of the top of a tower is observed
2
to be 60º. From a point 40 m vertically
n above the first point of observation, the
and S3 = [2 + ( n - 1) 3] angle of elevation of the top of the tower
2
is 30º. Find the height of the tower and
n its horizontal distance from the point of
= [2 + 3n - 3] [Here a = 1, d = 3]
2 observation.
Sol. Let h be the height of the tower AD and x
n
= [3n - 1] be the horizontal distance from the point
2 of observation.
Now, consider S1 + S3
Q BDEC is a rectangle.
n n \ CB = ED = x and CE = BD = 40 m
= [(n + 1) + 3n - 1] = ´ 4n
2 2
AB
In right DABC, tan 30º =
4n2 BC
= = 2n 2 = 2S2
2 1 AB
28. Two pipes running together can fill a Þ =
3 x
1
tank in 11 minutes. If one pipe takes 5 Þ x = AB 3 ...(i)
9
minutes more than the other to fill the AD
tank separately, find the time in which Now, in right DAED, tan 60º =
DE
each pipe would fill the tank separately.
h
Sol. Let one pipe takes x minutes to fill the tank. Þ 3 =
DE
Then, another pipe takes x + 5 to fill the
tank. h
Þ x= ...(ii)
According to question, we have 3
1 1 9 æ 1 100 ö
+ = çQ 11 = ÷ A
x x + 5 100 è 9 9 ø
Þ 100 [x + 5 + x] = 9 [x2 + 5x]
Þ 200x + 500 = 9x2 + 45x h
C B
Þ 9x2 – 155x – 500 = 0
Þ 9x2 – 180x + 25x – 500 = 0
Þ 9x (x – 20) + 25 (x – 20) = 0 C
E x
Þ (9x + 25) (x – 20) = 0
25 From equation (i) and (ii), we get
x = 20 or x = - (Rejected)
9 h
AB 3 =
Hence, one pipe takes 20 minutes and 3
another pipe takes 25 minutes to fill the
[Q AB + 40 = h Þ AB = h – 40]
tank.
https://www.arundeepselfhelp.info/index.php?route=product/product&path=180&product_id=395

Arundeep’s Solved Papers 141 Mathematics 2016 (Outside Delhi)

h Steps of construction :
Þ 3 ( h - 40) =
3 1. Draw a line segment AB of length 7 cm.
Þ 3 (h – 40) = h Then using A as centre and distance 5 cm
Þ 3h – 120 = h draw an arc. Also draw an arc using B as
Þ 2h = 120 centre and with distance 6 cm, which
Þ h = 60 m intersect earlier drawn arc at C. Join AC
Hence the required height of tower be and BC.
60 metre. 2. Draw an acute angle BAZ and cut AZ as
h AA1 = A1A2 = A2A3 = A3A4 = A4A5 and
60
From (ii), x = Þ x= join BA5.
3 3
3. Through A4 draw a line parallel BA 5
60 3 intersecting AB at B¢.
Þ x=
3 4. Through B¢ draw a line parallel to BC
Þ x = 20 3 intersecting AC at C¢.
Þ x = 34.641 m DAB¢C¢ is the required triangle.
Thus, required horizontal distance from 31. A number x is selected at random from
point of observation be 34.641 metre. the numbers 1, 4, 9, 16 and another
30. Draw a triangle with sides 5 cm, 6 cm number y is selected at random from the
and 7 cm. Then draw another triangle numbers 1, 2, 3, 4. Find the probability
4 that the value of xy is more than 16.
whose sides are of the corresponding
5 Sol. x can be 1, 4, 9 or 16 and y can be 1, 2, 3 or
sides of first triangle. 4.
Sol. Total number of cases of xy = 4 × 4 = 16.
Number of cases when xy is more than 16
are (9 × 2), (9 × 3), (9 × 4), (16 × 2),
C (16 × 3), (16 × 4), i.e. 6 cases.

6c 6 3
C¢ m P (value of xy more than 16) = =
16 8
5 cm

A B
7 cm B¢
A1
A2
A3
A4
A5

Z
https://www.arundeepselfhelp.info/index.php?route=product/product&path=180&product_id=395

MATHEMATICS 2016 TERM II (DELHI)


SET I
Time allowed : 3 hours Maximum marks : 90

General Instructions :
(i) All questions are compulsory.
(ii) The question paper consists of 31 questions divided into four sections – A, B, C and D.
(iii) Section A contains 4 questions of 1 mark each, Section-B contains 6 questions of 2 marks each.
Section-C contains 10 questions of 3 marks each and Section-D contains 11 questions of 4
marks each.
(iv) Use of calculators is not permitted.
1. From an external point P, tangents PA Þ ÐAOB + 40º + 40º = 180º
and PB are drawn to a circle with centre Þ ÐAOB = 180º – 80º = 100º
O. If ÐPAB = 50º, then find ÐAOB.
2. In Figure, AB is a 6 m high pole and CD
Sol. Given, ÐPAB = 50º is a ladder inclined at an angle of 60º to
ÐPAB + ÐOAB = 90º [Q angle the horizontal and reaches up to a point
between radius OA and tangent PA is 90º] D of pole. If AD = 2.54 m, find the length
of the ladder. (use 3 = 1.73)

Þ 50º + ÐOAB = 90º


Þ ÐOAB = 90º – 50º = 40º
Sol. BD = AB – AD
Now, PA = PB [Q tangents from an = 6 m – 2.54 m = 3.46 m
external point are same]
BD
Þ ÐPBA = ÐPAB In right DDBC, = sin 60º
CD
Þ ÐPBA = 50º
ÐPBA + ÐOBA = 90º 3.46 3
[\ angle between radius OB and tangent Þ =
CD 2
PB is 180º]
Þ 50º + ÐOBA = 90º 2 ´ 3.46 2 ´ 3.46
Þ CD = = =2×2=4m
Þ ÐOBA = 90º – 50º = 40º 3 1.73
Now in DAOB we have Hence, length of the ladder is 4 m.
ÐAOB + ÐABO + ÐBAO = 180º 3. Find the 9th term from the end (towards
[Q sum of angles in triangle is 180º] the first term) of the A.P. 5, 9, 13, ..., 185.
Arundeep’s Solved Papers 142 Mathematics 2016 (Delhi)
https://www.arundeepselfhelp.info/index.php?route=product/product&path=180&product_id=395

Arundeep’s Solved Papers 143 Mathematics 2016 (Delhi)


Sol. Reversing the given A.P., we get Þ 4a + 9b + 42 = 0 ...(ii)
185, 181, 174, ..., 9, 5 a (– 3)2 + 7 (– 3) + b = 0
Now, first term (a) = 185 [Q x = – 3 is the root of equation (i)]
Common difference, (d) = 181 – 185 = – 4
Þ 9a + b – 21 = 0 ...(iii)
We know that nth term of an A.P. is given
Putting the value of b from (iii) in (ii), we
by a + (n – 1) d
get
Ninth term a9 = a + (9 – 1) d
4a + 9 [21 – 9a] + 42 = 0
= 185 + 8 × (– 4) = 185 – 32 = 153
Þ 4a + 189 – 81a + 42 = 0
4. Cards marked with number 3, 4, 5, ...,
50 are placed in a box and mixed Þ 231– 77a = 0
thoroughly. A card is drawn at random Þ 77a = 231
from the box. Find the probability that Þ a=3
the selected card bears a perfect square Putting a = 3 in (iii), we get
number.
27 + b = 21
Sol. Total no. of possible outcomes when one
Þ b=–6
card is drawn = 48
Hence, a = 3, b = – 6
When the number on drawn card is a perfect
square, total favourable cases are 4, 9, 16, 6. Find the ratio in which y-axis divides the
25, 36, 49, i.e. = 6 line segment joining the points A (5, – 6)
P (perfect square number) and B (– 1, – 4). Also find the coordinates
of the point of division.
Number of total possible outcomes
= Sol. Let the point on y-axis be P (0, y) and
Number of favourable outcomes \ Co-ordinates of P is given by using section
6 1 formula, we have
= =
48 8 æ - k + 5 -4 k - 6 ö
ç , ÷
2 è k +1 k +1 ø
5. If x = and x = – 3 are roots of the Since point P lies on y-axis
3
quadratic equations ax2 + 7x + b = 0, find \ x-coordinate of P = 0
the values of a and b. -k + 5
Sol. Given quadratic equation is Þ =0
k +1
ax2 + 7x + b = 0 ...(i)
é 2 ù
êëQ x = 3 is the root of equation (i ) úû
1

2
æ2ö æ2ö 5-k
a ç ÷ +7 ç ÷+b = 0 Þ =0Þk=5
è3ø è3ø k +1
[_ if a be a root of f(x) = 0 then f(a) = 0] Hence the required ratio is 5 : 1.
4 14 ( - 4) (5) + (1) ( - 6)
Þ a+ +b =0 Thus, y-coordinate of P =
9 3 5 +1
4a + 42 + 9b -13
Þ =0 =
9 3
https://www.arundeepselfhelp.info/index.php?route=product/product&path=180&product_id=395

Arundeep’s Solved Papers 144 Mathematics 2016 (Delhi)

æ -13 ö Þ (2 y - 2) 2 + ( y + 5) 2 = (2 y + 3) 2 + ( y - 6) 2
Hence point on y-axis is ç 0, ÷
è 3 ø
[Q using Distance formula]
7. In given figure, a circle is inscribed in a
DABC, such that it touches the sides AB, on Squaring both sides we get
BC and CA at points D, E and F 4y2 + 4 – 8y + y2 + 10y + 25 = 4y2 + 9
respectively. If the lengths of sides AB, + 12y + y2 – 12y + 36
BC and CA are 12 cm, 8 cm and 10 cm
Þ 2y + 29 = 45
respectively, find the lengths of AD, BE
and CF. Þ 2y = 45 – 29 = 16
Þ y=8
C
\ 2y = 16
Hence coordinates of P are (16, 8)
9. How many terms of the A.P. 18, 16, 14,
F E
... be taken so that their sum is zero ?
Sol. Let the number of terms of given A.P. taken
A D B for sum to be zero be n.
Then, sum of n terms (Sn) = 0 (Given)
Sol. Given, AB = 12 cm, CA = 10 cm, BC = 8 cm
\ First term (a) = 18
Let AD = AF = x [Q Tangent drawn from
and Common difference (d) = – 2
external point to circle are equal]
\ DB = BE = 12 – x and CF = CE = 10 – x n
Therefore, Sn = [2a + (n - 1) d ]
BC = BE + EC Þ 8 = 12 – x + 10 – x 2
Þ 8 = 22 – 2x Þ 2x = 14
n
Þ 0 = [2 × 18 + (n – 1) (– 2)]
C 2
Þ 0 = 38 – 2n
10 –
–x

Þ n = 19
10

F E \ Hence, required sum of 19 terms is 0.


12 –

x 10. If given figure, AP and BP are tangents


x

A x D 12 – x B
to a circle with centre O, such that
AP = 5 cm and ÐAPB = 60º. Find the
Þ x=7 length of chord AB.
\ AD = 7 cm, BE = 5 cm and CF = 3 cm
8. The x-coordinate of a point P is twice its
y-coordinate. If P is equidistant from
Q (2, – 5) and R (– 3, 6), find the
coordinates of P.
Sol. Let the required point be (2y, y).
Let Q (2, – 5) and R (– 3, 6) are given
points.
Now, PQ = PR
https://www.arundeepselfhelp.info/index.php?route=product/product&path=180&product_id=395

Arundeep’s Solved Papers 145 Mathematics 2016 (Delhi)


Sol. In DAPB we have Hence, the area of shaded region = Area of
AP = BP square – Area of two semicircle AOB and
Þ ÐPAB = ÐPBA COD
= 196 – 154 = 42 cm2
[Q Tangents from an external point are
Therefore, area of four shaded parts (i.e.
equally inclined to segment joining centre
X, Y, W, Z) = (2 × 42) cm2 = 84 cm2
to point].
12. In Figure, a decorative block, made up
Let ÐPAB = x,
of two solids – a cube and a hemisphere.
then in DAPB, x + x + 60º = 180º The base of the block is a cube of side
[Q sum of angles of D is 180º] 6 cm and the hemisphere fixed on the top
has a diameter of 3.5 cm. Find the total
Þ 2x = 180º – 60º = 120º
i.e. x = 60º é 22 ù
surface area of the block. ê Use π = ú
As all three angles of DAPB are 60º. So ë 7û
DAPB is an equilateral triangle.
Hence AP = BP = AB = 5 cm
11. In figure, ABCD is a square of side 14
cm. Semi-circles are drawn with each
side of square as diameter. Find the area
é 22 ù
of the shaded region. ê Use π = ú
ë 7û

A B
Sol. Given Side of the cube = a = 6 cm
X
Total surface area of cube = 6 × (side)2 = 6a2
= 6 × (6)2 = 216 cm2
Z W
3.5
Given radius of hemisphere = r = cm
O 2
Y
Area covered on the face of cube by circular
D C part of hemisphere = pr2
Sol. Area of the square ABCD = 14 × 14 22 3.5 3.5
= ´ ´ cm 2
= 196 cm2 7 2 2
14 Curved surface area of hemisphere
Since radius of semi-circle =r = cm = 7cm = 2 × p × r2
2
22 3.5 3.5
1 = 2´ ´ ´ cm 2
Area of semicircle AOB = ´ pr 2 7 2 2
2
So, Total surface area of the block = Surface
1 22 æ 14 ö
= ´ ´ 7 ´ 7 = 77 cm2 çQ r = ÷ area of cube – Area of circular face of
2 7 è 2ø hemisphere
Similarly, area of semicircle DOC = 77 cm2 + Curved surface area of hemisphere
https://www.arundeepselfhelp.info/index.php?route=product/product&path=180&product_id=395

Arundeep’s Solved Papers 146 Mathematics 2016 (Delhi)


1
= 216 -
22 3.5 3.5
´ ´ + 2´
22 3.5 3.5
´ ´ = [0 (1 – 3) + 2 (3 + 1) + 0 (– 1 – 1)]
7 2 2 7 2 2 2

1
= 216 +
22 3.5 3.5
´ ´ = ´ 8 = 4 sq. units
7 2 2 2
= 216 + 9.625 = 225.625 cm2 æ 2 + 0 1+ 3ö
Coordinates of F are ç , ÷
13. In figure ABC is a triangle coordinates è 2 2 ø
of whose vertex A are (0, – 1). D and E i.e. (1, 2)
respectively are the mid-points of the
sides AB and AC and their coordinates [Q Using mid-point formula]
are (1, 0) and (0, 1) respectively. If F is
1
the mid-point of BC, find the areas of and Area of DDEF = |1 (1 – 2) + 0 (2 – 0)
DABC and DDEF. 2
+ 1 (0 – 1)|
A (0, –1)
1
= |– 1 + 0 – 1|
2
1
(1, 0) D E (0, 1) = × |– 2| = |– 1| = 1 sq. units
2
14. In figure, two arcs PAQ and PBQ are
F shown. Arc PAQ is a part of circle with
B C
centre O and radius OP while arc PBQ
Sol. Let coordinates of B are (x, y). Then using is a semicircle drawn on PQ as diameter
mid point formula we have with centre M. If OP = PQ = 10 cm, show
that area of shaded region is
x+0
=1Þx=2 æ pö
2 25 ç 3 - ÷ cm 2 .
è 6ø
y -1
=0Þy=1
2
Coordinates of B are (2, 1)
Let coordinates of C are (p, q)
Since E(0, 1) be the mid point of AC.
p+0
\ =0Þp=0
2
q -1
and =1Þq=3
2
Coordinates of C are (0, 3) Sol. OP = OQ = 10 cm (radii of circle)
\ Area of DABC PQ = 10 cm (given)
1 So, DOPQ is an equilateral triangle
= [x (y – y ) + x2 (y3 – y1) + x3 (y1 – y2)]
2 1 2 3 \ ÐPOQ = 60º
https://www.arundeepselfhelp.info/index.php?route=product/product&path=180&product_id=395

Arundeep’s Solved Papers 147 Mathematics 2016 (Delhi)


Area of segment PAQM = Area of sector Similarly, S17 = 289
OPAQ – Area of equilateral DOPQ
17
Þ [2a + (17 - 1) d ] = 289
60° 3 2
= ´ p ´ 10 ´ 10 - ´ 10 ´ 10
360° 4 Þ 2a + 16d = 34 Þ a + 8d = 17 ...(ii)
Solving (i) and (ii), we get
q a = 1 and d = 2
[_ area of sector = pr2 × ]
360°
n
Now Sn = [2a + (n - 1) d ]
æ 100p 100 3 ö 2
=ç - ÷ cm 2
è 6 4 ø n
= [2 ´ 1 + (n - 1) 2]
2
pr 2 1
and Area of semicircle = = ´ p´ 5´5
2 2 n
= [2 + 2n - 2] = n × n = n2
2
25
= p cm 2 16. Solve for x :
2
2x 1 3x + 9
\ Area of the shaded region + + = 0,
x - 3 2 x + 3 ( x - 3) (2 x + 3)
25 æ 100p 100 3 ö
= p-ç - ÷ 3
2 è 6 4 ø x ¹ 3, -
2
25p 50p Sol. Given eqn. be
= - + 25 3
2 3 2x 1 3x + 9
+ + =0
75p - 100p x - 3 2 x + 3 ( x - 3) (2 x + 3)
= + 25 3
6
2 x (2 x + 3) + ( x - 3) + (3 x + 9)
25p Þ ( x - 3) (2 x + 3)
=0
= 25 3 -
6 Þ 4x2 + 6x + x – 3 + 3x + 9 = 0
Þ 4x2 + 10x + 6 = 0
æ pö
= 25 ç 3 - ÷ cm 2 Þ 2x2 + 5x + 3 = 0
è 6ø
Þ 2x2 + 2x + 3x + 3 = 0
15. If the sum of first 7 terms of an A.P. is 49 Þ 2x (x + 1) + 3 (x + 1) = 0
and that of its first 17 terms is 289, find
Þ (x + 1) (2x + 3) = 0
the sum of first n terms of the A.P.
Þ x + 1 = 0 or 2x + 3 = 0
Sol. Let a be the first term and d be the common
difference of an A.P. -3 æ -3ö
Þ x = – 1 or x = çQ given, x ¹ ÷
Given : S7 = 49, 2 è 2 ø
n -3
where Sn = [2a + (n - 1) d ] Q It is given that x ¹ .
2 2
7 Hence, solution of the given equation
Þ [2a + (7 - 1) d ] = 49 be x = – 1.
2
17. A well of diameter 4 m is dug 21 m deep.
Þ 2a + 6d = 14 Þ a + 3d = 7 ...(i) The earth taken out of it has been spread
https://www.arundeepselfhelp.info/index.php?route=product/product&path=180&product_id=395

Arundeep’s Solved Papers 148 Mathematics 2016 (Delhi)


evenly all around it in the shape of a circular and height of solid cylinder = h cm
ring of width 3 m to form an embankment. Here r + h = 37 ...(i)
Find the height of the embankment. Total surface area of cylinder = 2pr (h + r)
Sol. Radius of the well = r = 2m, = 2prh + 2pr2
Height of the well = h = 21 m Þ 2pr (h + r) = 1628
Volume of the earth dug out = pr2h Þ 2pr × 37 = 1628 [using (i)]
22
= p × 2 × 2 × 21 m3 =
22
´ 2 ´ 2 ´ 21 Þ 2´ ´ r ´ 37 = 1628
7 7
= 264 m3 1628 ´ 7
\ External radius of embankment = Radius Þ r = = 7 cm
2 ´ 22 ´ 37
of well + width of embankment Given, r + h = 37
=2+3=5m Þ 7 + h = 37 Þ h = 37 – 7 = 30 cm
[_ given width of embankment = 3m] Hence, volume of cylinder = pr2h
22
= ´ 7 ´ 7 ´ 30 = 4620 cm3
7
3
19. The angles of depression of the top and
bottom of a 50 m high building from the
2
top of a tower are 45º and 60º
respectively. Find the height of the tower
and the horizontal distance between the
tower and the building. (use 3 = 1.73)
Volume of embankment
Sol. Let CD is the building of height 50 m and
= ( pr12 h - pr22 h) m3 AB be the tower.
= [p × (5)2 × h – p (2)2 h] m3 Let horizontal distance between the tower
= (p × 25 × h – p × 4 × h) m3 and building is BC is x metre.
Q BCDE is a rectangle
æ 22 ö
= ç ´ 21 ´ h ÷ m3 = 66 h m3 So, ED = BC and BE = CD
è 7 ø Also, ED = x and BE = 50 m
As per given condition, we have Let AE = y
Volume of earth dug out = Volume of
embankment A
264m3 = 66h m3 60°
45°

264 ym
\ Height of embankment, h = = 4m
66
18. The sum of the radius of base and height 45°
D E
of a solid right circular cylinder is 37 cm.
If the total surface area of the solid
cylinder is 1628 sq. cm, find the volume
50 m
é 22 ù
of the cylinder. ê Use π = ú
ë 7û
60°
Sol. Given base radius of solid cylinder = r cm C xm B
https://www.arundeepselfhelp.info/index.php?route=product/product&path=180&product_id=395

Arundeep’s Solved Papers 149 Mathematics 2016 (Delhi)

y y Favourable cases when both numbers are


Now, in DAED, = tan 45º Þ = 1 prime are (2, 2), (2, 3), (2, 5), (3, 2), (3, 3),
x x
(3, 5), (5, 2), (5, 3), (5, 5), i.e. 9 outcomes
Þ y=x ...(i)
P (a prime number on each dice)
AB
Now, in DABC, = tan 60º Favourable cases
BC =
Total cases
AE + EB y + 50
Þ = 3 Þ = 3 9 1
BC x = =
36 4
Þ x + 50 = 3x [Q y = x, using (i)] (ii) Favourable cases when sum of numbers are
9 or 11 are (3, 6), (4, 5), (5, 4), (5, 6),
Þ 3x - x = 50 Þ ( 3 - 1) x = 50
(6, 3), (6, 5), i.e. 6 outcomes
50 P (a total of 9 or 11)
Þ x =
3 -1 Favourable outcomes
=
Total possible outcomes
50 ( 3 + 1)
Þ x =
( 3 - 1) ( 3 + 1) 6 1
= =
36 6
50 ( 3 + 1) 50 ( 3 + 1)
= = 21. A passenger, while boarding the plane,
3 -1 2 slipped from the stairs and got hurt. The
pilot took the passenger in the emergency
Þ x = 25 ( 3 + 1) = 25 (1.73 + 1)
clinic at the airport for treatment. Due
= 25 × 2.73 = 68.25 m to this, the plane got delayed by half an
\ Height of the tower = 50 + y hour. To reach the destination 1500 km
= 50 + 68.25 (Q x = y) away in time, so that the passengers
could catch the connecting flight, the
= 118.25 m speed of the plane was increased by 250
Horizontal distance between the tower and km/hour than the usual speed. Find the
the building = x = 68.25 m usual speed of the plane.
20. In a single throw of a pair of different What value is depicted in this question ?
dice, what is the probability of getting Sol. Let the usual speed of plane be x km/hr.
(i) a prime number on each dice ?
Time taken to cover 1500 km with usual
(ii) a total of 9 or 11 ?
1500
Sol. (i) Total possible cases when two dice are speed = hours.
thrown together = 6 × 6 = 36 x
\ Simple space = {(1, 1), (1, 2)....(1, 6) When the speed of plane is increased, then
new speed = (x + 250) km/hr.
(2, 1), (2, 2) ......(2, 6)
(3, 1), (3, 2) .... (3, 6) Time taken to cover 1500 km with the new
(4, 1), (4, 2) .... (4, 6) 1500
speed (x + 250) km/hr =
(5, 1), (5, 2) ..... (5, 6) x + 250
(6, 1), (6, 2) ...... (6, 6) According to question, we have
https://www.arundeepselfhelp.info/index.php?route=product/product&path=180&product_id=395

Arundeep’s Solved Papers 150 Mathematics 2016 (Delhi)

1500 1500 1 ÐOAP = ÐOBP = 90º ...(i)


= + [Radius is perpendicular to the tangent at
x x + 250 2
the point of contact]
1500 1500 1 OA = OB (radii) ...(ii)
Þ - =
x x + 250 2 OP is common ...(ii)
1500 x + 1500 ´ 250 - 1500 x 1 \ DOAP @ DOBP
Þ =
x ( x + 250) 2 (RHS axiom of congruency)
[from (i), (ii) and (iii)]
1500 ´ 250 1
Þ = Hence, AP = BP (CPCT)
x 2 + 250 x 2
23. Draw two concentric circles of radii 3 cm
Þ x2 + 250x = 750000 and 5 cm. Construct a tangent to smaller
Þ x2 + 250x – 750000 = 0 circle from a point on the larger circle.
Þ x2+ 1000x – 750x – 750000 = 0 Also measure its length.
Þ x (x + 1000) – 750 (x + 1000) = 0 Sol. Now after measuring, PA and PB comes out
to be 4 cm.
Þ (x + 1000) (x – 750) = 0
Þ x + 1000 = 0 or x – 750 = 0
Þ x = – 1000 or x = 750
Þ x = 750 (Q Speed cannot be negative)
Hence, the usual speed of plane is 750 km/hr. A
In this question, pilot’s caring behaviour
towards passengers is shown as well as
other side pilot is cautious and alert for his P O
M
duty to reach destination point at scheduled
time.
22. Prove that the lengths of tangents drawn
B
from an external point to a circle are
equal.
Sol. Given : A circle C (O, r), P is a point outside
the circle and PA and PB are tangents to a
circle. Steps of construction of tangents :
1. Take point O. Draw 2 concentric circles of
M
radii 3 cm and 5 cm respectively.
2. Locate point P on the circumference of
A O larger circle.
3. Join OP and bisect it. Let M be mid-point
of OP.
N 4. Taking M as centre and MP as radius, draw
To Prove : PA = PB an arc intersecting smaller circle at A and
Construction : Draw OA, OB and OP. B.
Proof : Consider triangles OAP and OBP. Join PA and PB. Thus, PA, PB are required
tangents.
https://www.arundeepselfhelp.info/index.php?route=product/product&path=180&product_id=395

Arundeep’s Solved Papers 151 Mathematics 2016 (Delhi)


24. In given figure, O is the centre of a circle of radius 5 cm. T is a point such that OT = 13 cm
and OT intersects circle at E. If AB is a tangent to the circle at E, find the length of AB,
where TP and TQ are two tangents to the circle.

P
A

O 5 cm E T

Sol. In DOPT ; OP2 + PT2 = OT2 [Q Pythagoras theorem]

PT = OT 2 - OP 2 = 169 - 25 = 12 cm
and TE = OT – OE = 13 – 5 = 8 cm
Let PA = AE = x [tangent from outer point A]
In DTEA, TE2 + EA2 = TA2 [Q Pythagoras theorem]
Þ (8)2 + (x)2 = (12 – x)2 Þ 64 + x2 = (12 – x)2
Þ 64 + x2 = 144 + x2 – 24x Þ 80 = 24x Þ x = 3.3 cm
Thus AB = 2 × 3.3 cm = 6.6 cm [Q AE = EB, as AB is tangent to circle at E]
25. Find x in terms of a, b and c :

a b 2c
+ = , x ¹ a, b, c
x-a x-b x-c

a b 2c
Sol. Given eqn. be + =
x-a x -b x-c

a ( x - b) + b ( x - a) 2c ( ax + bx - 2ab) 2c
Þ= = Þ =
( x - a ) ( x - b) x-c ( x - ax - bx + ab)
2
x-c
Þ (ax + bx – 2ab) (x – c) = 2c (x2 – ax – bx + ab)
Þ ax2 + bx2 – 2abx – acx – bcx + 2abc = 2cx2 – 2acx – 2bcx + 2abc
Þ (a + b – 2c) x2 + (ac + bc – 2ab) x = 0
Þ x {(a + b – 2c) x + (ac + bc – 2ab)} = 0

2ab - ac - bc
Þ x = 0 or x =
a + b - 2c
26. A bird is sitting on the top of a 80 m high tree. From a point on the ground, the angle of
elevation of the bird is 45º. The bird flies away horizontally in such a way that it remained
https://www.arundeepselfhelp.info/index.php?route=product/product&path=180&product_id=395

Arundeep’s Solved Papers 152 Mathematics 2016 (Delhi)


at a constant height from the ground. After 2 seconds, the angle of elevation of the bird
from the same point is 30º. Find the speed of flying of the bird. (Take 3 = 1.732)
Sol. Let BC be the tree of height 80 metre
After 2 seconds, position of bird is at point E.
Let CE = x
BC
In DCBA, = tan 45º
AB
E C
80 80
Þ = 1 Þ y = 1 Þ y = 80 m
AB
ED
In DEDA, = tan 30º
AD 80 m 80 m
80 1
Þ =
AB + BD 3
45° 30°
80 1
Þ = D x B y A
80 + x 3
Þ 80 3 = 80 + x [Q AB = 80 m]

Þ x = 80 3 - 80 Þ x = 80 ( 3 - 1)
Þ x = 80 (1.732 – 1) Þ x = 80 × 0.732 Þ x = 58.56 m Þ BD = x = 58.56 m
distance (BD) 58.56
So, the speed of flying of the bird = = = 29.28 m/s
Time 2
27. A thief runs with a uniform speed of 100 m/minute. After one minute a policeman runs
after the thief to catch him. He goes with a speed of 100 m/minute in the first minute and
increases his speed by 10 m/minute every succeeding minute. After how many minutes the
policeman will catch the thief.
Sol. Let total time be (n – 1) minutes in which the police catch the thief.
Since thief ran 1 minute before police start running.
Q Time taken by thief before he was caught = (n – 1 + 1) = n minute
Then total distance covered by thief = (100 × n) metres
Total distance covered by policeman in (n – 1) minute
= 100 + 110 + 120 + .... + (n – 1) terms

( n - 1) é n ù
=
2
[200 + (n – 2) 10] êëQ Sn = 2 [2a + (n - 1) d ]úû

According to question,
Total distance covered by thief in ‘n’ minutes
= total distance covered by policeman in (n – 1) minute
https://www.arundeepselfhelp.info/index.php?route=product/product&path=180&product_id=395

Arundeep’s Solved Papers 153 Mathematics 2016 (Delhi)

( n - 1)
100n = [200 + (n – 2) 10]
2
Þ 200n = (n – 1) [200 + 10n – 20] Þ 200n = (n – 1) (10n + 180)
Þ 200n = 10n2 + 180n – 10n – 180 Þ 10n2 – 30n – 180 = 0
Þ n2 – 3n – 18 = 0 Þ n2 – 6n + 3n – 18 = 0
Þ n (n – 6) + 3 (n – 6) = 0 Þ (n – 6) (n + 3) = 0
Þ n = 6 or n = – 3 (rejected)
Hence, time taken by policeman to catch the thief is (6 – 1), i.e., 5 minutes.
28. Prove that the area of a triangle with vertices (t, t – 2), (t + 2, t + 2) and (t + 3, t) is independent
of t.
Sol. Given vertices of triangle are
{t, t – 2), {t + 2, t + 2), {t + 3, t}
Let (x1, y1), (x2, y2), (x3, y3) are vertices of the triangle.
1
Then Area of the triangle = |x (y – y ) + x2 (y3 – y1) + x3 (y1 – y2)|
2 1 2 3
1 1
= |t (t + 2 – t) + (t + 2) {t – t + 2} + (t + 3) {t – 2 – t – 2}| = |2t + 2t + 4 – 4t – 12|
2 2
1
= × |– 8| = 4 sq. units,
2
Hence, area of triangle is independent of t.
29. A game of chance consists of spinning an arrow on a circular board, 3 2
divided into 8 equal parts, which comes to rest pointing at one of
4 1
the numbers 1, 2, 3, ..., 8 which are equally likely outcomes. What
is the probability that the arrow will point at (i) an odd number
5 8
(ii) a number greater than 3 (iii) a number less than 9 ?
6 7
Sol. (i) Total possible outcomes when the arrow points at one of the numbers
are 8.
Favourable outcomes when the required number is odd are 1, 3, 5, 7, i.e. 4 outcomes.
\ P (an odd number)
No. of favourable outcomes 4 1
= = =
Total no. of possible outcomes 8 2
(ii) Favourable outcomes when the required number is more than 3 are 4, 5, 6, 7, 8, i.e. 5 outcomes.
P (a number is more than 3)
No. of favourable outcomes
=
Total no. of possible outcomes
(iii) Favourable outcomes when the required number is less than 9 are 1, 2, 3, 4, 5, 6, 7, 8 i.e. 8
outcomes.
https://www.arundeepselfhelp.info/index.php?route=product/product&path=180&product_id=395

Arundeep’s Solved Papers 154 Mathematics 2016 (Delhi)

No. of favourable outcomes 8


P(number is less than 9) = = =1
Total no. of possible outcomes 8
30. An elastic belt is placed around the rim of a pulley of radius 5 cm. From one point C on the
belt, the elastic belt is pulled directly away from the centre O of the pulley until it is at P, 10
cm from the point O. Find the length of the belt that is still in contact with the pulley. Also
find the shaded area. (use p = 3.14 and 3 = 1.73)

5 cm q
C
O P

Sol. Given : AO = 5 cm and OP = 10 cm


In right DAOP, we have
Base AO 5 1
cos q = = = =
Hypotenuse OP 10 2
Þ q = 60º
ÐAOB = q¢ = 2 × 60 = 120º
360º - q¢
Þ Length of AOB = ´ 2pr (Q q¢ = 120º)
360º
240° 2
= ´ 2 ´ 3.14 ´ 5 = × 10 × 3.14 = 20.93 cm
360° 3
Hence, length of belt in contact = 20.93 cm
Now, in right DOAP, we have
AP
\ tan q =
AO
AP
Þ tan 60º = Þ AP = 5 3 cm
5
1 1
Area of (DOAP + DOBP) = × AO × AP + × OB × PB
2 2
1 1
= ´ 5´ 5 3 + ´ 5´ 5 3 (Q AP = BP and OA = OB)
2 2
= 25 3 cm2 = 25 × 1.73 cm2 = 43.25 cm2


Thus, Area of sector OACB = ´ pr 2
360
https://www.arundeepselfhelp.info/index.php?route=product/product&path=180&product_id=395

Arundeep’s Solved Papers 155 Mathematics 2016 (Delhi)


120° 1
= × 3.14 × 5 × 5 = × 3.14 × 25 = 26.16 cm2
360° 3
\ Area of shaded region = Area of (DOAP + DOBP) – Area of sector OACB
= 43.25 – 26.16 = 17.09 cm2
31. A bucket open at the top is in the form of a frustum of a cone with a capacity of 12308.8
cm3. The radii of the top and bottom circular ends are 20 cm and 12 cm respectively. Find
the height of the bucket and the area of metal sheet used in making the bucket. (use p =
3.14)
Sol. Let R, r and V be the upper radius, lower radius and volume of the frustum respectively then
R = 20 cm, r = 12 cm
and V = 12308.8 cm3
1
\ Volume of frustum of cone = × p [(R2 + r2 + Rr)] h
3
1
Þ 12308.8 = × 3.14 [400 + 144 + 240] h
3
12308.8 ´ 3
h = Þ h = 15 cm
3.14 ´ 784

Now, l (slant height) = (R – r ) 2 + h2 = (20 - 12) 2 + 152

= 64 + 225 = 289 = 17 cm2


Total area of metal sheet used = Curved Surface Area of cone + Area of base
= p (R + r) l + pr2
= p (20 + 12) × 17 + p × 12 × 12 [Q lower radius, r = 12 cm]
= p × 32 × 17 + 144p = 688p = 688 × 3.14 = 2160.32 cm2

SET-II [UNCOMMON QUESTIONS TO SET-I]

10. How many terms of the A.P. 27, 24, 21, .... should be taken so that their sum is zero ?
Sol. In the given A.P.,
Here, first term (a) = 27
and Common difference (d) = 24 – 27 = – 3
Given Sum of n terms (Sn) = 0
n
We know that, Sn = [2a + (n - 1) d ]
2
n
0 = [2 × 27 + (n – 1) (– 3)]
2
Þ 54 – 3n + 3 = 0 Þ 3n = 57 Þ n = 19
Thus, the sum of 19 terms of the given A.P. is zero.
https://www.arundeepselfhelp.info/index.php?route=product/product&path=180&product_id=395

Arundeep’s Solved Papers 156 Mathematics 2016 (Delhi)


18. Solve for x :
x+1 x-2 2x + 3
+ =4- ; x ¹ 1, – 2, 2
x -1 x + 2 x-2
Sol. Given eqn. be,
x +1 x - 2 2x + 3 x + 1 x - 2 2x + 3
+ = 4- Þ + + =4
x -1 x + 2 x-2 x -1 x + 2 x - 2

( x + 1) ( x + 2) ( x - 2) + ( x - 2) 2 ( x - 1) + (2 x + 3) ( x - 1) ( x + 2)
Þ =4
( x - 1) ( x + 2) ( x - 2)
Þ (x + 1) (x2 – 4) + (x – 1) (x2 + 4 – 4x) + (2x + 3) (x2 + x – 2) = 4 (x – 1) (x2 – 4)
Þ x3 – 4x + x2 – 4 + x3 + 4x – 4x2 – x2 – 4 + 4x + 2x3 + 2x2 – 4x + 3x2 + 3x – 6
= 4 (x3 – 4x – x2 + 4)
Þ x3 + x2 – 4x – 4 + x3 – 5x2 + 8x – 4 + 2x3 + 5x2 – x – 6 = 4 (x3 – x2 – 4x + 4)
Þ 4x3 + x2 + 3x – 14 = 4x3 – 4x2 – 16x + 16
Þ 5x2 + 19x – 30 = 0 Þ 5x2 + 25x – 6x – 30 = 0
Þ 5x (x + 5) – 6 (x + 5) = 0 Þ (x + 5) (5x – 6) = 0
6
Þ x + 5 = 0 or 5x – 6 = 0 Þ x = – 5 or x =
5
Thus, solutions of given equation are
6
x = – 5 and x =
5
19. Two different dice are thrown together. Find the probability of :
(i) getting a number greater than 3 on each die.
(ii) getting a total of 6 or 7 of the numbers on two dice.
Sol. (i) When two dice are thrown together total possible outcomes = 6 × 6 = 36
Sample space = {(1, 1), (1, 2)....(1, 6)
(2, 1), (2, 2) ......(2, 6)
(3, 1), (3, 2) .... (3, 6)
(4, 1), (4, 2) .... (4, 6)
(5, 1), (5, 2) ..... (5, 6)
(6, 1), (6, 2) ...... (6, 6)}
Favourable outcomes when both dice have number more than 3 are (4, 4), (4, 5), (4, 6), (5, 4),
(5, 5), (5, 6), (6, 4), (6, 5), (6, 6), i.e. 9 outcomes.
Number of favourable outcomes
\ Required P (a number greater than 3 on each due) =
Number of total possible outcomes

9 1
= =
36 4
(ii) Favourable outcomes when sum of the numbers appearing on the dice is 6 or 7 are, i.e. (1, 5),
https://www.arundeepselfhelp.info/index.php?route=product/product&path=180&product_id=395

Arundeep’s Solved Papers 157 Mathematics 2016 (Delhi)


(1, 6), (2, 4), (2, 5), (3, 3), (3, 4), (4, 2), from bottom of tower respectively.
(4, 3), (5, 1), (5, 2), (6, 1). \ BD = 4 m and BC = 9 m
Thus total no. of favourable cases = 11.
AB
11 In right DABD, = tan 60º
BC
\ Required P (a total of 6 or 7) =
36
h
20. A right circular cone of radius 3 cm, has Þ
a cur ved sur face ar ea of 47.1 cm2. Find 4 = 3
the volume of the cone. (use p = 3.14) Þ h =4 3
Sol. Given Radius of cone (r) = 3 cm
\ Curved surface area of cone = prl = 47.1 cm2
A
47.1 47.1
\ l = = = 5 cm,
pr 3.14 ´ 3
hm

60° 30°
a C
B 4m D
l 9m
h

Hence the required height of tower be 4 3


r metre.
where l = slant height of cone 29. Construct a triangle ABC in which
BC = 6 cm, AB = 5 cm and ÐABC = 60º.
We know that, h = l 2 - r 2 Then construct another triangle whose
3
= 52 - 32 = 25 - 9 = 16 = 4 sides are times the corresponding
4
1 sides of DABC.
\ Required volume of cone = ´ pr 2 h
3 Sol. Steps of construction :
1 1. Draw DABC with side BC = 6 cm,
= × 3.14 × 3 × 3 × 4 AB = 5 cm, ÐABC = 60º.
3
= 3.14 × 3 × 4 = 37.68 cm3 2. Draw ray BX making an acute angle with
BC on opposite side of vertex A.
28. The angles of elevation of the top of a
tower from two points at a distance of 4 3. Locate 4 points P 1 , P 2 , P 3 , P 4 on line
m and 9 m from the base of the tower segment BY.
and in the same straight line with it are 4. Join P 4 C and draw a line through P 3 ,
60º and 30º respectively. Find the height parallel to P4C intersecting BC at C¢.
of the tower.
5. Draw a line through C¢ parallel to AC
Sol. Let AB be the tower of height ‘h’. intersecting AB at A¢.
Let D and C are the two points on ground
DA¢BC¢ is the required triangle.
which is at a distance of 4 m and 9 metre
https://www.arundeepselfhelp.info/index.php?route=product/product&path=180&product_id=395

Arundeep’s Solved Papers 158 Mathematics 2016 (Delhi)


Now, (a + b)2
= a2 + b2 + 2ab
2
(35) = 625 + 2ab [using (1)]
Þ 1225 – 625 = 2ab
Þ 2ab = 600 Þ ab = 300

c=
b

25
cm
90°
30. The perimeter of a right triangle is 60 C a B
cm. Its hypotenuse is 25 cm. Find the
area of the triangle. 1
\ Required area DABC = × base × height
Sol. Here, Perimeter of triangle, a + b + c = 60 2
[Given]
1
Let c be the hypotenuse of triangle DABC = ab
2
a + b + 25 = 60
a + b = 35 cm ...(1) 1
= × 300 = 150 cm2
Using Pythagoras theorem, we have 2
a2 + b2 = (25)2 = 625
SET-III [UNCOMMON QUESTIONS TO SET-I and Set II]
10. How many terms of the A.P. 65, 60, 55, major defects. A shirt is taken out of the
... be taken so that their sum is zero ? box at random. What is the probability
Sol. In the given A.P., that
First term (a) = 65 (i) Ramesh will buy the selected shirt ?
and Common difference (d) = 60 – 65 = – 5 (ii) ‘Kewal’ will buy the selected shirt ?
Sum of n terms (Sn) = 0 Sol. (i) When one shirt is taken out, then
n number of total possible outcomes = 100
Since, Sn = [2a + (n - 1) d ] Ramesh will purchase when shirt is good,
2
\ Favourable outcomes = number of good
n shirts = 88
0 = [2 ´ 65 + (n - 1) (- 5)]
2 P (Ramesh buys shirt)
0 = 130 – 5n + 5 Number of favourable outcomes
Þ – 5n = – 135 Þ n = 27 =
Number of total possible outcomes
\ Hence, required sum of 27 terms is zero.
18. A box consists of 100 shirts of which 88 88 22
= =
are good, 8 have minor defects and 4 100 25
have major defects. Ramesh, a (ii) Kewal will buy shirt if a shirt is not having
shopkeeper will buy only those shirts major defect.
which are good but ‘Kewal’ another Number of favourable outcomes = Number
shopkeeper will not buy shirts with of shirts without major defect = 96
https://www.arundeepselfhelp.info/index.php?route=product/product&path=180&product_id=395

Arundeep’s Solved Papers 159 Mathematics 2016 (Delhi)

96 24
\ P (Kewal buys a shirt) = =
100 25
æ a a + bö
19. Solve the following quadratic equation for x : x 2 + ç + ÷ x+1=0
èa+b a ø

æ a a +bö
Sol. Given eqn. be, x 2 + ç + ÷ x +1 = 0
èa+b a ø

æ a a + bö æ a öæ a + b ö
Þ x2 + ç + ÷ x+ç ÷ç ÷ =0
èa+b a ø è a + b øè a ø

æ a ö æa +bö æ a ö æa + bö
Þ x2 + ç ÷x+ç ÷x +ç ÷·
è a + b ø çè a ÷ø
=0
è a + b ø è a ø

é a ù æa +bö é a ù æ a ö æ a +bö
Þ x êx + +ç ÷ êë x + a + b úû = 0 Þ çx+ ÷ çx + ÷ =0
ë a + b úû è a ø è a +bø è a ø

a a+b æ a ö æa +bö
Þ x+ = 0 or x + =0 Þ x = -ç ÷ or x = - ç ÷
a+b a è a + b ø è a ø
20. A toy is in the form of a cone of base radius 3.5 cm mounted on a hemisphere of base
diameter 7 cm. If the total height of the toy is 15.5 cm, find the total surface area of the toy.
é 22 ù
êë Use π = 7 úû
Sol. Here, given that
base radius of cone = base radius of hemisphere = 3.5 m = r
h = height of cone = 15.5 – 3.5 = 12 cm
Also, slant height of cone, l = h 2 + r 2

= (12) 2 + (3.5) 2 cm = 144 + 12.25 cm = 156.25 cm = 12.5 cm


l
22 h
\ Curved Surface Area of cone = prl = ´ 3.5 ´ 12.5 = 137.5 cm2
7 15.5 cm
\ Curved surface area of hemisphere = 2pr2 3.5 cm

22 3.5 cm
= 2´ ´ (3.5)2 = 77 cm2
7
Hence, Total Surface Area of toy = Surface area of hemisphere + Curved Surface Area of cone
= 77 + 137.5 = 214.5 cm2
28. The sum of three numbers in A.P. is 12 and sum of their cubes is 288. Find the numbers.
Sol. Let the three numbers in A.P. are a – d, a, a + d
Then a – d + a + a + d = 12 [Q Given that, S3 = 12]
https://www.arundeepselfhelp.info/index.php?route=product/product&path=180&product_id=395

Arundeep’s Solved Papers 160 Mathematics 2016 (Delhi)


Þ 3a = 12 Þ a = 4 30. The time taken by a person to cover
Also, (a – d)3 + a3 + (a + d)3 = 288 1
150 km was 2 hours more than the
[Q Sum of their cubes = 288] 2
Þ (4 – d)3 + (4)3 + (4 + d)3 = 288 time taken in the return journey. If he
returned at a speed of 10 km/hour more
Þ 64 – 48d + 12d2 – d3 + 64 + 64 + 48d + than the speed while going, find the speed
12d2 + d3 = 288 per hour in each direction.
Þ 24d2 + 192 = 288 Sol. Let t1 and t2 be the time taken in going and
Þ 24d2 = 96 returning respectively.
Þ d2 = 4 Þ d = ± 2 Also, u and u + 10 be the speed in going
For d = 2, the numbers will be 2, 4, 6. For and returning respectively.
d = – 2, numbers will be 6, 4, 2. Distance
Hence, required numbers are 2, 4, 6. As, Velocity =
Time
29. Prove that the tangent at any point of a
circle is perpendicular to the radius Distance
Þ Time =
through the point of contact. Velocity
Sol. Given : A circle C (O, r) and a tangent AB
150 150
at a point P. \ t1 = and t2 =
To prove : OP ^ AB u u + 10
Construction : Take any point Q other than 1
P on the tangent AB. According to question, t1 – t2 = 2
2
Join OQ, intersecting circle at R.
150 150 5
Proof : We have, OP = OR [Radii] - =
u u + 10 2

é1 1 ù 5
Þ 150 ê - ú =
O ë u u + 10 û 2

1 1 5
Þ - =
P υ υ + 10 2 ´ 150
A M N B
1 1 1
Þ - =
OQ = OR + RQ u u + 10 60
\ OQ > OR Þ OQ > OP
υ + 10 - υ 1
[Q OR = OP = radius] Þ =
υ 60
Thus, OP < OQ, i.e., OP is shorter than any
Þ 60 [u + 10 – u] = u (u + 10)
other segment joining O to any point of AB.
Þ u2 + 10u – 600 = 0
But among all line segments, joining point
O to point on AB, shortest one is Þ u2 + 30u – 20u – 600 = 0
perpendicular from O on AB. Þ u (u + 30) – 20 (u + 30) = 0
Hence, OP ^ AB. Þ (u + 30) (u – 20) = 0
https://www.arundeepselfhelp.info/index.php?route=product/product&path=180&product_id=395

Arundeep’s Solved Papers 161 Mathematics 2016 (Delhi)


Þ u = 20 or u = – 30 (rejected)
Hence, velocity in going 20 km/hour and in returning be (u + 10) i.e. 30 km/hour.
31. Draw a triangle ABC with BC = 7 cm, ÐB = 45º and ÐA = 105º. Then construct a triangle
4
whose sides are times the corresponding sides of DABC.
5
Sol. Given, ÐB = 45º, ÐA = 105º
\ Sum of all interior angles in D = 180º
\ ÐA + ÐB + ÐC = 180º
Þ ÐC = 30º

X
Y


P1
P2
P3
P4
P5
Z

Steps of construction :
1. Draw a line segment BC = 7 cm
2. at B, construct ÐCBX = 45° and at C, construct ÐBCY = 30°
3. Let BX and CY intersects at A.
4. Join AB and AC we get the DABC.
5. Draw a ray BZ making an acute angle with BC on opposite side of vertex A.
6. Locate 5 points P1, P2, P3, P4, P5 on BZ, s.t BP1 = P1 P2 = P2P3 = P3P2 = P4P5
7. Join P5C. Draw line through P4 parallel to P5C intersecting BC at C¢.
8. Through C¢, draw line parallel to AC intersecting AB at A¢.
DA¢BC¢ is the required triangle.
Pages 162 – 1371 are missing
Click on the link to purchase complete book

https://www.arundeepselfhelp.info/index.php?ro
ute=product/product&path=180&product_id=395
9 789388 395434

You might also like